Exam 1 & 2

Pataasin ang iyong marka sa homework at exams ngayon gamit ang Quizwiz!

A client recovering from abdominal surgery refuses analgesia, saying that he is "fine, as long as he doesn't move." Which nursing diagnosis should be a priority? 1) Deficient Knowledge (pain control measures) 2) Ineffective Health Maintenance 3) Risk for Ineffective Airway Clearance 4) Impaired Physical Mobility

1) Deficient Knowledge (pain control measures)

A client who had abdominal surgery 4 hours ago is receiving a continuous epidural infusion of an analgesic. Which of the following observations indicates the nurse should monitor the client closely? 1) Drowsy; drifts off to sleep before completing a sentence 2) Respirations = 18/minute 3) Drowsy; easily aroused 4) Pain rating 1-2/10

1) Drowsy; drifts off to sleep before completing a sentence

Which of the following is the purpose of assessing? 1) Establish a database of client responses to their health status. 2) Identify client strengths and problems. 3) Develop an individualized plan of care. 4) Implement care, prevent illness, and promote wellness.

1) Establish a database of client responses to their health status. Rationale: Assessing provides a database of the client's physiological and psychosocial responses to his or her health status. Client strengths and problems (option 2) are identified in the diagnosing phase of the nursing process, a care plan is established (option 3) in the planning phase, and care, prevention, and wellness promotion (option 4) are part of the implementing phase.

A collaborative (multidisciplinary) problem is indicated instead of a nursing or medical diagnosis 1) If both medical and nursing interventions are required to treat the problem. 2) When independent nursing actions can be utilized to treat the problem. 3) In cases where nursing interventions are the primary actions required to treat the problem. 4) When no medical diagnosis (disease) can be determined.

1) If both medical and nursing interventions are required to treat the problem. Rationale: A collaborative (multidisciplinary) problem is indicated when both medical and nursing interventions are needed to prevent or treat the problem. If nursing care alone (whether that care involves independent or dependent nursing actions) can treat the problem, a nursing diagnosis is indicated. If medical care alone can treat the problem, a medical diagnosis is indicated.

Which of the following demonstrates appropriate use of guidelines in implementing nursing interventions? Select all that apply. 1) No interventions should be carried out without the nurse having clear rationales. 2) Always follow the physician's orders exactly, without variation. 3) Encourage all clients to be as dependent as desired and allow the nurse to perform care for them. 4) When possible, give the client options in how interventions will be implemented. 5) Each intervention should be accompanied by client teaching.

1) No interventions should be carried out without the nurse having clear rationales. 4) When possible, give the client options in how interventions will be implemented. 5) Each intervention should be accompanied by client teaching. Rationale: Nurses should always have clear rationales for their actions, clients should be given options whenever possible, and client teaching is a constant, integral part of implementing. Primary care provider orders must be critically evaluated and modified to meet individual client needs (option 2). Clients may have nurses provide needed care but should take care of themselves whenever possible since dependency has its own complications (option 3).

How can the nurse best assess a client's style of learning? 1. Ask the client how he or she learns best 2. Use a variety of teaching strategies 3. Observe the client's interactions with others 4. Ask a family member

1. Ask the client how he or she learns best- Individuals learn in various ways, and the individual knows how learning has occurred in the past

After auscultating the abdomen, the nurse should report which finding to the primary care provider? 1. Bruit over the aorta. 2. Absence of bowel sounds for 60 seconds. 3. Continuous bowel sounds over the ileocecal valve. 4. A completely irregular pattern of bowel sounds.

1. Bruit over the aorta.

Which statement made by the patient indicates a need for further teaching on sleep hygiene? 1) "I' m going to do my exercises before I eat dinner." 2) "I'll have a glass of wine at bedtime to relax." 3) "I set my alarm to get up at the same time every morning." 4) "I moved my computer to the den to do my work."

2) "I'll have a glass of wine at bedtime to relax."

Proper administration of an otic medication to a 2-year-old client includes which of the following? 1) Pull the ear straight back. 2) Pull the ear down and back. 3) Pull the ear up and back. 4) Pull the ear straight upward.

2) Pull the ear down and back. Rationale: To straighten the ear canal in children less than 3 years of age, the ear must be pulled down and back. In individuals over 3 years of age, the ear is pulled up and back.

Because of significant concerns about financial problems a middle-aged client complains of difficulty sleeping. Which outcome would be the most appropriate for the nursing care plan? "By day 5, the client will: 1) Sleep 8 to 10 hours per day." 2) Report falling asleep within 20 to 30 minutes." 3) Have a plan to pay all the bills." 4) Decrease worrying about financial problems and will keep busy until bedtime."

2) Report falling asleep within 20 to 30 minutes."

The nurse completed the following assessment: 63-year-old female patient has had abdominal pain for 6 days. She reportsnot having a bowel movement for 4 days, whereas she normally has a bowel movement every 2 to 3 days. She has not been hospitalized in the past. Her abdomen is distended. She reports being anxious about upcoming tests. Her temperature was 37° C, pulse 82 and regular, blood pressure 128/72. Which of the following data form a cluster, showing a relevant pattern? (Select all that apply.) 1. Vital sign results 2. Abdominal distention 3. Age of patient 4. Change in bowel elimination pattern 5. Abdominal pain 6. No past history of hospitalization

2. Abdominal distention 4. Change in bowel elimination pattern 5. Abdominal pain

Which learning activity reflects Bloom's affective domain? 1. Administering an injection 2. Accepting the loss of a limb 3. Inserting a catheter 4. Learning how to read

2. Accepting the loss of a limb

Which condition places the client at the greatest risk for developing an infection? 1. Implantation of a prosthetic device 2. Burns over more than 20% of the body 3. Presence of an indwelling urinary catheter 4. More than 2 puncture sites from laparoscopic surgery

2. Burns over more than 20% of the body, Burns more than 20% of the client's total body surface are generally considered major burn injuries. When the skin is damaged by a burn the underlying tissue is left unprotected and the individual is at risk for infection. The greater the extent and deeper the depth of the burn, the higher the risk is for infection. Prosthetic devices are surgically implanted under sterile conditions to minimize risk of infection. Indwelling urinary catheters are implanted under sterile conditions and are considered closed systems where sterile technique is maintained. Laparoscopic surgery is also performed using sterile technique.

Which of the following is an expected finding during assessment of the older adult? 1. Facial hair becomes finer and softer. 2. Decreased peripheral, color, and night vision. 3. Increased sensitivity to odors. 4. Respiratory rate and rhythm are irregular at rest.

2. Decreased peripheral, color, and night vision.

Which of the following are examples of collaborative problems? (Select all that apply.) 1. Nausea 2. Hemorrhage 3. Wound infection 4. Fear

2. Hemorrhage 3. Wound infection

The nurse enters a patient's room and finds that the patient was incontinent of liquid stool. The patient has recurrent redness in the perineal area, and there is concern that he is developing a pressure ulcer. The nurse cleanses the patient, inspects the skin, and applies a skin barrier ointment to the perineal area. She calls the ostomy and wound care specialist and asks that he visit the patient to recommend skin care measures. Which of the following describe the nurse's actions? (Select all that apply.) 1. The application of the skin barrier is a dependent care measure. 2. The call to the ostomy and wound care specialist is an indirect care measure. 3. The cleansing of the skin is a direct care measure. 4. The application of the skin barrier is a direct care measure.

2. The call to the ostomy and wound care specialist is an indirect care measure. 3. The cleansing of the skin is a direct care measure. 4. The application of the skin barrier is a direct care measure. The call to the ostomy and wound care specialist is an indirect care measure involving collaborative care. Cleansing the skin is an independent direct care measure. Applying the skin barrier is an independent nursing measure involving direct care

Which statement made by an older adult best demonstrates understanding of taking a sleep medication? 1) "I'll take the sleep medicine for 4 or 5 weeks until my sleep problems disappear." 2) "Sleep medicines won't cause any sleep problems once I stop taking them." 3) "I'll talk to my health care provider before I use an over the- counter sleep medication." 4) "I'll contact my health care provider if I feel extreme sleepy in the mornings."

3) "I'll talk to my health care provider before I use an over the- counter sleep medication."

Which statement made by a mother being discharged to home with her newborn infant indicates a need for further teaching? 1) "I won't put the baby to bed with a bottle." 2) "For the first few weeks we're putting the cradle in our room." 3) "My grandmother told me that babies sleep better on their stomachs." 4) "I know I'll have to get up during the night to feed the baby when he wakes up."

3) "My grandmother told me that babies sleep better on their stomachs."

During the review of systems in a nursing history, a nurse learns that the patient has been coughing mucus. Which of the following nursing assessments would be best for the nurse to use to confirm a lung problem? (Select all that apply.) 1. Family report 2. Chest x-ray film 3. Physical examination with auscultation of the lungs 4. Medical record summary of x-ray film findings

3. Physical examination with auscultation of the lungs 4. Medical record summary of x-ray film findings

Which client statement indicates a need for further teaching regarding treatment for hypokalemia? 1) "I will use avocado in my salads." 2) "I will be sure to check my heart rate before I take my digoxin." 3) "I will take my potassium in the morning after eating breakfast." 4) "I will stop using my salt substitute."

4) "I will stop using my salt substitute." Rationale: Salt substitutes contain potassium. The client can still use it within reason. Option 1: Avocado is higher in potassium than most foods. Option 2: Hypokalemia can potentiate digoxin toxicity and checking the pulse will help the client avoid this. Option 3: It is important to take potassium with food to avoid gastric upset.

Which statement by the client indicates that the preoperative teaching regarding gallbladder surgery has been effective? 1) "I cannot eat or drink anything after midnight." 2) "I'm not going to cough after surgery because it might open my incision." 3) "I might have a stroke if I stop taking my anticoagulant." 4) "The nurse showed me how to contract and relax my calf muscles."

4) "The nurse showed me how to contract and relax my calf muscles." Rationale: Option 1 is incorrect because of the ASA guidelines for preoperative fasting. Option 2 is incorrect because clients are taught how to cough and also how to splint their incision to prevent complications. Option 3 is incorrect because anticoagulants are discontinued a few days before surgery to avoid excessive bleeding postoperatively.

The nurse is to administer 0.5 mL of a medication by intramuscular injection to an older emaciated client. Which is the most appropriate for the nurse to use? 1) A tuberculin syringe, #25-#27 gauge, 1/4- to 5/8-inch needle 2) Two 3-mL syringes, #20-#23 gauge, 1 1/2-inch needle 3) 2-mL syringe, #25 gauge, 5/8-inch needle 4) 2-mL syringe, #20-#23 gauge, 1-inch needle

4) 2-mL syringe, #20-#23 gauge, 1-inch needle Rationale: If the nurse goes by the amount of the medication (0.5 mL) only, the deltoid muscle would be the site. However, knowing and assessing the client is critical. The muscles of an older, emaciated client will most likely be diminished or atrophied. The nurse should consider the ventrogluteal site because that site will have the most muscle mass.

A client reports to the nurse that she has been taking barbiturate sleeping pills every night for several months and now wishes to stop taking them. Which statement is the most appropriate advice for the nurse to provide the client? 1) Take the last pill on a Friday night so disrupted sleep can be compensated on the weekend. 2) Continue to take the pills since sleeping without them after such a long time will be difficult and perhaps impossible. 3) Discontinue taking the pills. 4) Continue taking the pills and discuss tapering the dose with the primary care provider.

4) Continue taking the pills and discuss tapering the dose with the primary care provider.

A client is admitted to the hospital for hypocalcemia. Nursing interventions relating to which system would have the highest priority? 1) Renal 2) Cardiac 3) Gastrointestinal 4) Neuromuscular

4) Neuromuscular

After being admitted directly to the surgery unit, a 75-year old client who had elective surgery to replace an arthritic hip was discharged from the postanesthesia recovery unit. The client has been on the orthopedic floor for several hours. Which type of planning will be least useful during the first shift on the orthopedic unit? 1) Initial 2) Ongoing 3) Discharge 4) Strategic

4) Strategic Rationale: Strategic planning is an ongoing process focused on organizational change rather than individual clients so it is least useful and not relevant in this case. The client requires initial planning because he has just arrived on the orthopedic unit for the first time (option 1). Of the three types of planning that need to be done at this time, initial is the highest priority since he has just had surgery. The client also requires the ongoing type of planning necessary to determine the care appropriate for this shift (option 2). Discharge planning needs to start on admission to ensure adequate client preparation for management of health needs outside the health agency (option 3).

The nurse writes an expectedoutcome statement in measurable terms. An example is: 1. Patient will be pain free. 2. Patient will have less pain. 3. Patient will take pain medication every 4 hours. 4. Patient will report pain acuity less than 4 on a scale of 0 to 10.

4. Patient will report pain acuity less than 4 on a scale of 0 to 10.

The operating room (OR) and postanesthesia care unit (PACU) are high-risk environments for patients with a latex allergy. Which safety measures to prevent a latex reaction should the nurse implement? (Select all that apply.) A. Screening patients about food allergies known to have a cross-reactivity to latex such as kiwis and bananas B. Having a latex allergy cart available at all times C. Communicating with the operating room (OR) team as soon as 24 to 48 hours in advance of the surgery when a latex-sensitive patient is identified D. Scheduling the latex-sensitive patient for the last operative case of the day

A, B, C

The nurse is planning to teach the student nurse how to assess the hydration status of an older adult. Which techniques are appropriate for this situation? (Select all that apply.) A. Inspect the lips and mucous membranes to determine if they are moist. B. Pinch the skin on the back of the hand to see if the skin tents. C. Check the patient's pulse and blood pressure. D. Weigh the patient daily.

A, C, D

The nurse is teaching a program on healthy nutrition at the senior community center. Which points should be included in the program for older adults? (Select all that apply.) A. Avoid grapefruit and grapefruit juice, which impair drug absorption. B. Increase the amount of carbohydrates for energy. C. Take a multivitamin that includes vitamin D for bone health. D. Cheese and eggs are good sources of protein. E. Limit fluids to decrease the risk of edema.

A, C, D

Which of the following is the most effective way to break the chain of infection? A) Hand hygiene B) Wearing gloves C) Placing patients in isolation D) Providing private rooms for patients

A. Hand hygiene

A patient's surgical wound has become swollen, red, and tender. You note that the patient has a new fever and leukocytosis. What is the best immediate intervention? A) Notify the health care provider and use surgical technique to change the dressing. B) Reassure the patient and recheck the wound later. C) Notify the health care provider and support the patient's fluid and nutritional needs. D) Alert the patient and caregivers to the presence of an infection to ensure care after discharge.

A. Notify the health care provider and support the patient's fluid and nutritional needs.

Your ungloved hands come in contact with the drainage from your patient's wound. What is the correct method to clean your hands? A) Wash them with soap and water. B) Use an alcohol-based hand cleaner. C) Rinse them and use the alcohol-based hand cleaner. D) Wipe them with a paper towel.

A. Wash them with soap and water.

A 62-year-old woman is being discharged home with her husband after surgery for a hip fracture from a fall at home. When providing discharge teaching about home safety to this patient and her husband, the nurse knows that: A. A safe environment promotes patient activity. B. Assessment focuses on environmental factors only. C. Teaching home safety is difficult to do in the hospital setting. D. Most accidents in the older adult are caused by lifestyle factors.

A. A safe environment promotes patient activity.

If a nurse experiences a problem reading a physician's medication order, the most appropriate action will be to A. Call the physician to verify order. B. Call the pharmacist to verify order. C. Consult with other nursing staff to verify. D. Withhold the medication until physician makes rounds.

A. Call the physician to verify order

A nurse is caring for a newly admitted client who has a documented history of falls. Which of the following is the priority action by the nurse? A. Complete a fall-risk assessment. B. Educate the client and family on fall risks. C. Complete a physical assessment. D. Survey the client's belongings.

A. Complete a fall-risk assessment.

Abnormal sound or murmur heard while auscultating an organ, gland, or artery. A. Atrophied B. Auscultation C. Borborygmi D. Bruit

D

A patient returns to your postoperative unit following surgery for right shoulder rotator cuff repair. The licensed practical nurse (LPN) reports that she had difficulty obtaining the patient's heart rate from his right radial pulse. What is your best response? a) Assess the patient's apical pulse to obtain the heart rate. b) Obtain the heart rate from right and left radial sites. c) Obtain the heart rate using the oximeter probe. d) Perform a complete assessment of all pulses.

d) Perform a complete assessment of all pulses. When an LPN reports that one pulse is difficult to obtain, first you need to assess the patient yourself and compare the quality of all pulses.

A nursing diagnosis of Ineffective Peripheral Tissue Perfusion would be validated by which one of the following? Bounding radial pulse. Irregular apical pulse. Carotid pulse stronger on the left side than the right. Absent posterior tibial and pedal pulses.

Your Answer: Absent posterior tibial and pedal pulses. Rationale: The posterior tibial and pedal pulses in the foot are considered peripheral and at least one of them should be palpable in normal individuals. Option 1: A bounding radial pulse is more indicative that perfusion exists. Options 2 and 3: Apical and carotid pulses are central and not peripheral. Cognitive Level: Analyzing. Client Need: Health Promotion and Maintenance. Nursing Process: Diagnosing.

The nurse teaches a patient taking a benzodiazepine that this group of medications causes which symptom of a sleep problem? 1) Nocturia 2) Hyperactivity 3) Grogginess and feeling hung over 4) Increased sleep time

3) Grogginess and feeling hung over

A client who describes his pain as 6 on a scale of 1 to 10 is classified as having which of the following? 1) Mild pain 2) Mild to moderate pain 3) Moderate to severe pain 4) Very severe pain

3) Moderate to severe pain

When planning care for pain control of older clients, which principles should the nurse apply? Select all that apply. 1) Pain is a natural outcome of the aging process. 2) Pain perception increases with age. 3) The client may deny pain. 4) The nurse should avoid use of opioids. 5) The client may describe pain as an "ache" or "discomfort."

3) The client may deny pain. 5) The client may describe pain as an "ache" or "discomfort."

The client at greatest risk for postoperative wound infection is: 1. A 3-month-old infant postoperative from pyloric stenosis repair 2. A 78-year-old postoperative from inguinal hernia repair 3. An 18-year-old drug user postoperative from removal of a bullet in the leg 4. A 32-year-old diabetic postoperative from an appendectomy

3. An 18-year-old drug user postoperative from removal of a bullet in the leg; All are at risk for infection. Answer 3 is at greatest risk, because the bullet is unclean, and a drug user is at great risk for immune deficiency.

A nurse reviews data gathered regarding a patient's pain symptoms. The nurse compares the defining characteristics for acute pain with those for chronic pain and in the end selects acute pain as the correct diagnosis. This is an example of the nurse avoiding an error in: 1. Data collection. 2. Data clustering. 3. Data interpretation. 4. Making a diagnostic statement.

3. Data interpretation.

A 74 year old client who takes multiple medications tells the nurse, "I have no idea what that little yellow pill is for." What is the best nursing diagnosis for this client? 1. Deficient Knowledge 2. Health-Seeking Behavior 3. Deficient Knowledge (Medication Information) 4. Noncompliance

3. Deficient Knowledge (Medication Information)

You find that your newly assigned client has very shiny skin on their legs, has little or no leg hair, and the client reports that their skin damages easily. You would suspect that these signs and symptoms are related to: 1. Overuse of caustic products to strip the leg hair. 2. Chronic neurological pathology. 3. Impaired peripheral arterial circulation. 4. Inherited reduction in sweat glands and hair follicles.

3. Impaired peripheral arterial circulation; Shiny skin on the legs, reduction in or absence of leg hair, and skin that damages easily is often related to impaired peripheral arterial circulation.

A blowing sound caused by turbulence in a narrowed section of a blood vessel: A. Bruit B. Atherosclerosis C. Occlusion D. Syncope

A

Exaggeration of the posterior curvature of the thoracic spine. A. Kyphosis B. Lordosis C. Malignancy D. Murmurs

A

Frontal or ventral surface face down: A. Pronation B. Supination C. Flexion D. Extension E. Hyperextension

A

Which of the following is not a characteristic of Delirium? A. Fever B. Confusion C. Disorientation D. Restlessness

A

Which of these patients do you expect will need teaching regarding dietary sodium restriction?

A 65-year-old Recently Diagnosed with Heart Failure

The primary reason that family members should be included when the nurse teaches the patient preoperative exercises is so they can: A. Coach and encourage the patient after surgery. B. Demonstrate to the patient at home. C. Relieve the nurse by getting the patient to do the exercises every 2 hours. D. Practice with the patient while he or she is waiting to be taken to the operating room.

A Coach and encourage the patient after surgery.

After a surgical patient has been given preoperative sedatives, which safety precaution should a nurse take? A. Reinforce to the patient to remain in bed or on the stretcher B. Raise the side rails and keep the bed or stretcher in the high position C. Determine if the patient has any allergies to latex D. Obtain informed consent immediately after sedative administration

A Reinforce to the patient to remain in bed or on the stretcher

Obesity places patients at an increased surgical risk because of which of the following factors? (Select all that apply.) A. Risk for bleeding is increased. B. Ventilatory capacity is reduced. C. Fatty tissue has a poor blood supply. D. Metabolic demands are increased.

A, B, C

As a nurse prepares to provide morning care and treatments, it is important to question a patient about a latex allergy before which intervention? (Select all that apply.) A. Applying adhesive tape to anchor a nasogastric tube B. Inserting a rubber Foley catheter into the patient's bladder C. Providing oral hygiene using a standard toothbrush and toothpaste D. Giving an injection using plastic syringes with rubber-coated plungers E. Applying a transparent wound dressing

A, B, D

A patient is admitted through the emergency department for multisystem trauma following a motorcycle crash with multiple orthopedic injuries. He goes to surgery for repair of fractures. He is postoperative day 3 from an open reduction internal fixation of bilateral femur fractures and external fixator to his unstable pelvic fracture. Interventions that are necessary for prevention of venous thromboembolism in this high-risk postsurgical patient include: (Select all that apply.) A. Intermittent pneumatic compression stockings. B. Vitamin K therapy. C. Subcutaneous heparin or enoxaparin (Lovenox). D. Continuous heparin drip with a goal of an international normalized ratio (INR) 5 times higher than baseline

A, C

A nurse educator is presenting a module on basic first aid for newly licensed home health nurses. The nurse educator evaluates the teaching as effective when the newly licensed nurse states the client who has heat stroke will have which of the following? A. Hypotension B. Bradycardia C. Clammy skin D. Bradypnea

A. Hypotension

6. A nurse is teaching a patient about the Speak Up Initiatives. Which information should the nurse include? a. The nurse is the center of the health care team. b. If you still do not understand, ask again. c. Ask a nurse to be your advocate or supporter. d. Inappropriate medical tests are the most common mistakes.

ANS: B If you still do not understand, ask again is part of the S portion of the Speak Up Initiatives. Speak up if you have questions or concerns. You (the patient) are the center of the health care team, not the nurse. Ask a trusted family member or friend to be your advocate (advisor or supporter), not a nurse. Medication errors are the most common health care mistakes, not inappropriate medical tests.

List five aspects of the skin that the nurse assesses during a routine examination.

Answers should include: color, turgor, temperature, moisture, lesions, odor, and edema.

Which of the following assessments do you perform routinely when an older adult patient is receiving intravenous 0.9% NaCl?

Auscultate Dependent Portions of Lungs

Abnormal condition of one or both upper eyelids in which the eyelid droops; caused by weakness of the levator muscle or paralysis of the third cranial nerve. A. Olfaction B. Ptosis C. Ectropion D. Entropion

B

Increased lumbar curvature. A. Kyphosis B. Lordosis C. Malignancy D. Murmurs

B

When a nurse is performing surgical hand asepsis, the nurse must keep hands: A) Below elbows. B) Above elbows. C) At a 45-degree angle. D) In a comfortable position.

B. Above elbows.

If an infectious disease can be transmitted directly from one person to another, it is a: A) Susceptible host. B) Communicable disease. C) Port of entry to a host. D) Port of exit from the reservoir.

B. Communicable disease.

You are caring for a 65-year-old patient 2 days after surgery and helping him walk down the hallway. The surgeon has ordered exercise as tolerated. Your assessment indicates that the patient's heart rate at baseline is 88. After walking approximately 30 yards down the hallway, the heart rate is 110. What should be your next action? A. Stop exercise immediately and have him sit in a nearby chair. B. Ask him how he feels; determine if there is any discomfort or shortness of breath; and, if not, continue exercise. C. Tell him that he needs to walk further to reach a heart rate of 120. D. Have him walk slower; he has reached his maximum.

B. Ask him how he feels; determine if there is any discomfort or shortness of breath; and, if not, continue exercise.

While examining Mr. Parker, the nurse notes a circumscribed elevation of skin filled with serous fluid on his upper lip. The lesion is 0.4mc in diameter. This type of lesion is called a: A. Macule B. Nodule C. Vesicle D. Pustule

C

Identify the interval when a patient progresses from nonspecific signs to manifesting signs and symptoms specific to a type of infection. A) Illness stage B) Convalescence C) Prodromal stage D) Incubation period

C. Prodromal stage

The nurse would call the primary care provider immediately for which laboratory result? Hgb = 16 g/dL for a male client. Hct = 22% for a female client. WBC = 9 x 10³/mL³ Platelets = 300 x 10³/mL³

Hct = 22% for a female client.

A patient who is comatose is admitted to the hospital with an unknown history. Respirations are deep and rapid. Arterial blood gas levels on admission are pH, 7.20; PaCO2, 21 mm Hg; PaO2, 92 mm Hg; and HCO3-, 8. You interpret these laboratory values to indicate:

Metabolic Acidosis

The health care provider's order is 1000 mL 0.9% NaCl with 20 mEq K+ intravenously over 8 hours. Which assessment finding causes you to clarify the order with the health care provider before hanging this fluid?

Oliguria

Which is the most effective nursing action for controlling the spread of infection? Thorough hand hygiene. Wearing gloves and masks when providing direct client care. Implementing appropriate isolation precautions. Administering broad-spectrum prophylactic antibiotics.

Thorough hand hygiene. Rationale: Since the hands are frequently in contact with clients and equipment, they are the most obvious source of transmission. Regular and routine hand hygiene is the most effective way to prevent movement of potentially infective materials. PPE (gloves and masks) is indicated for situations requiring standard precautions (option 2). Isolation precautions are used for clients with known communicable diseases (option 3). Routine use of antibiotics is not effective and can be harmful due to the incidence of superinfection and development of resistant organisms (option 4). Cognitive Level: Applying. Client Need: Safe, Effective Care Environment. Nursing Process: Implementation.

The licensed practice nurse (LPN) provides you with the change-of-shift vital signs on four of your patients. Which patient do you need to assess first? a) 84-year-old man recently admitted with pneumonia, RR 28, SpO2 89% b) 54-year-old woman admitted after surgery for fractured arm, BP 160/86 mm Hg, HR 72 c) 63-year-old man with venous ulcers from diabetes, temperature 37.3°C (99.1°F), HR 84 d) 77-year-old woman with left mastectomy 2 days ago, RR 22, BP 148/62

a) 84-year-old man recently admitted with pneumonia, RR 28, SpO2 89% Oxygen saturation is low, indicating a problem with ventilation or diffusion, which is related to the respiratory rate

The overall goal of nursing care during the intraoperative phase is the client's _____

safety

The nurse is caring for a patient experiencing dysphagia. Which interventions help decrease the risk of aspiration during feeding? (Select all that apply.) A. Sit the patient upright in a chair. B. Give liquids at the end of the meal. C. Place food in the strong side of the mouth. D. Provide thin foods to make it easier to swallow. E. Feed the patient slowly, allowing time to chew and swallow. F. Encourage patient to lie down to rest for 30 minutes after eating.

A, C, E

The nurse is teaching a patient with poor arterial circulation about checking blood flow in the legs. Which information should the nurse include? (Select all that apply.) A. A normal pulse on the top of the foot indicates adequate blood flow to the foot. B. To locate the dorsalis pedis pulse, take the fingers and palpate behind the knee C. When there is poor arterial blood flow, the leg is generally warm to the touch. D. Loss of hair on the lower leg indicates a long-term problem with arterial blood flow.

A, D

The nurse is administering a sustained-release capsule to a new patient. The patient insists that he cannot swallow pills. What is the nurse's next best course of action? A. Ask the prescriber to change the order B. Crush the pill with a mortar and pestle C. Hide the capsule in a piece of solid food D. Open the capsule and sprinkle it over pudding

A. Rationale: Enteric-coated or sustained-release capsules should not be crushed; the nurse needs to contact the prescriber to change the medication to a form that is liquid or can be crushed.

A client has wound that is healing by secondary intention. To best support the healing of the wound, the nurse should expect the practitioner's order to state, "Clean wound with:"

"Clean wound with normal saline and apply a wet-to-damp dressing"; Cleaning with normal saline will not damage fibroblasts. Wet-to-damp dressings allow epidermal cells to migrate more rapidly across the wound surface than dry dressings, thereby facilitating wound healing.

Following a gastroscopy, a client asks for something to eat. The nurse correctly responds: "I will first check your gag reflex." "I will first listen for bowel sounds." "I will first have you cough and deep-breathe." "I will first listen to your lungs."

"I will first check your gag reflex."

A certified nursing assistant is collecting a 24-hour urine specimen from a client. Which statement by the assistant indicates that the specimen collection will need to be restarted? "I used a container from the lab that has a preservative in it." "The client voided in it right away, and I wrote the time on the container." "I have the container in a plastic bucket with ice in it." "I told the client that every single urination must be put in the container. If one is missed, call one of us."

"The client voided in it right away, and I wrote the time on the container."

A client is to obtain a clean-catch urine specimen. Which statement by the client demonstrates a lack of understanding regarding the procedure? "I should use all of the towelettes in the kit and use each only once." "Urinate into the cup as soon as I start to go." "I don't have to fill the cup. Just get an ounce or two." "Put the cover on right away, without touching the inside of the cover or the cup."

"Urinate into the cup as soon as I start to go."

After teaching a client and family strategies to prevent infection prevention, which statement by the client would indicate effective learning has occurred? "We will use antimicrobial soap and hot water to wash our hands at least three times per day." "We must wash or peel all raw fruits and vegetables before eating." "A wound or sore is not infected unless we see it draining pus." "We should not share toothbrushes but it is OK to share towels and washcloths."

"We must wash or peel all raw fruits and vegetables before eating." Rationale: Raw foods touched by human hands can carry significant infectious organisms and must be washed or peeled. Antimicrobial soap is not indicated for regular use and may lead to resistant organisms. Hand hygiene should occur as needed. Hot water can dry and harm skin, increasing the risk of infection (option 1). Clients should learn all the signs of inflammation and infection (e.g., redness, swelling, pain, heat) and not rely on the presence of pus to indicate this (option 3). People should not share washcloths or towels (option 4). Cognitive Level: Analyzing. Client Need: Safe, Effective Care Environment. Nursing Process: Evaluation.

The patient reports vivid dreaming to the nurse. Through understanding of the sleep cycle, the nurse recognizes that vivid dreaming occurs during which sleep phase? 1) REM sleep 2) Stage 1 NREM sleep 3) Stage 4 NREM sleep 4) Transition period from NREM to REM sleep

1) REM sleep

During admission to a hospital unit, the client tells the nurse that her sleep tends to be very light and that it is difficult for her to get back to sleep if she's awakened at night. Which interventions should the nurse implement? Select all that apply. 1) Remind colleagues to keep their conversation to a minimum at night. 2) Encourage the client's family members to bring in a radio to play soft music at night. 3) Deliver necessary medications and procedures at 1.5- or 3-hour intervals between 11 PM and 6 AM. 4) Encourage the client to ask family members to bring in a fan to provide white noise. 5) Increase the temperature in the room.

1) Remind colleagues to keep their conversation to a minimum at night. 3) Deliver necessary medications and procedures at 1.5- or 3-hour intervals between 11 PM and 6 AM. 4) Encourage the client to ask family members to bring in a fan to provide white noise.

Which of the following nursing diagnoses contains the proper components? 1) Risk for Caregiver Role Strain related to unpredictable illness course. 2) Risk for Falls related to tendency to collapse when having difficulty breathing. 3) Impaired Communication related to stroke. 4) Sleep Deprivation secondary to fatigue and a noisy environment

1) Risk for Caregiver Role Strain related to unpredictable illness course. Rationale: States the relationship between the stem (caregiver role strain) and the cause of the problem. Option 2: The diagnostic statement says the same thing as the related factor (falls and collapse). Option 3: It is inappropriate to use medical diagnoses such as stroke within a nursing diagnosis statement. Option 4 is vague. The statement must be specific and guide the plan of care (fatigue may be a result of sleep deprivation and does not direct intervention).

Which of the following is true regarding the relationship of implementing to the other phases of the nursing process? 1) The findings from the assessing phase are reconfirmed in the implementing phase. 2) After implementing, the nurse moves to the diagnosing phase. 3) The nurse's need for involvement of other health care team members in implementing occurs during the planning phase. 4) Once all interventions have been completed, evaluating can begin.

1) The findings from the assessing phase are reconfirmed in the implementing phase. Rationale: During implementing, the nurse also assesses and compares with the initial assessment. Evaluating follows implementing (option 2), mobilization of other health care teams is a part of implementing (option 3), and evaluating occurs during or immediately after each intervention, not waiting for all interventions to be completed (option 4).

A new nursing graduate's first job requires 12-hour night shifts. Which strategy will make it easier for the graduate to sleep during the day and remain awake at night? 1) Wear dark wrap-around sunglasses when driving home in the morning, and sleep in a darkened bedroom. 2) Exercise on the way home to avoid having to stand around waiting for equipment at the gym. 3) Drink several cups of strong coffee or 16 oz of caffeinated soda when beginning the shift. 4) Try to stay in a brightly lit area when working at night.

1) Wear dark wrap-around sunglasses when driving home in the morning, and sleep in a darkened bedroom.

Which of the following items are used to perform wound care irrigation? Select all that apply. 1. Clean gloves 2. Sterile gloves 3. Refrigerated irrigating solution 4. 60-mL syringe

1, 2, and 4; To irrigate a wound, the nurse uses clean gloves to remove the old dressing and to hold the basin collecting the irrigating fluid plus sterile gloves to apply the new dressing. A 60-mL syringe is the correct size to hold the volume of irrigating solution plus deliver safe irrigating pressure. The irrigation fluid should be at room or body temperature-- certainly not refrigerated.

A major characteristic of the nursing process is which of the following? 1) A focus on client needs. 2) Its static nature. 3) An emphasis on physiology and illness. 4) Its exclusive use by and with nurses.

1) A focus on client needs. Rationale: The nursing process focuses on client needs. It is dynamic rather than static (option 2), emphasizes client responses rather than physiology and illness (option 3), and is collaborative rather than used exclusively by nurses (option 4).

In the case in which a client is vulnerable to developing a health problem, the nurse chooses which type of nursing diagnosis status? 1) A risk nursing diagnosis. 2) A wellness nursing diagnosis. 3) A health promotion nursing diagnosis. 4) An actual nursing diagnosis.

1) A risk nursing diagnosis. Rationale: A risk nursing diagnosis is appropriate when the evidence for the problem indicates that a condition exists that makes the client vulnerable to a problem. A wellness diagnosis is used when enhancement of the client's already healthy responses is indicated (option 2). Health promotion diagnoses are used when the client seeks to increase well-being but need not currently be well (option 3). An actual diagnosis is used when the client already exhibits the problem (option 4).

The nurse is to administer a tuberculin test to a 22-year-old male who is 6 feet tall and weighs 180 pounds. Which is the most appropriate for the nurse to use? 1) A tuberculin syringe, #25-#27 gauge, 1/4- to 5/8-inch needle 2) Two 3-mL syringes, #20-#23 gauge, 1 1/2-inch needle 3) 2-mL syringe, #25 gauge, 5/8-inch needle 4) 2-mL syringe, #20-#23 gauge, 1-inch needle

1) A tuberculin syringe, #25-#27 gauge, 1/4- to 5/8-inch needle Rationale: A tuberculin test is given by intradermal injection. A tuberculin syringe is used because the dosage will most likely be 0.1 mL. A short, fine needle is needed to avoid entering the subcutaneous tissue. The needle should have a short bevel and usually be between #25 and #27 gauge. The needle should be between 1/4 to 5/8 inch long.

Which of the following principles does the nurse use in selecting interventions for the care plan? 1) Actions should address the etiology of the nursing diagnosis. 2) Always select independent interventions when possible. 3) There is one best intervention for each goal/outcome. 4) Interventions should be "doing," not just "monitoring."

1) Actions should address the etiology of the nursing diagnosis. Rationale: Interventions should address the etiology of the nursing diagnosis. Both independent and dependent interventions should be selected if appropriate (option 2) and several interventions may be needed for a single outcome (option 3). Both action and assessment-type interventions can be used (option 4).

A college student was referred to the campus health service because of difficulty staying awake in class. What should be included in the nurse's assessment? Select all that apply. 1) Amount of sleep he usually obtains during the week and on weekends. 2) How much alcohol he usually consumes. 3) Onset and duration of symptoms. 4) Whether or not his classes are boring. 5) What medications, including herbal remedies, he is taking.

1) Amount of sleep he usually obtains during the week and on weekends. 3) Onset and duration of symptoms. 5) What medications, including herbal remedies, he is taking.

A health care provider writes the following order for an opioidnaive patient who returned from the operating room following a total hip replacement. "Fentanyl patch 100 mcg, change every 3 days." Based on this order, the nurse takes the following action: 1) Calls the health care provider, and questions the order 2) Applies the patch the third postoperative day 3) Applies the patch as soon as the patient reports pain 4) Places the patch as close to the hip dressing as possible

1) Calls the health care provider, and questions the order

The intake and output (I & O) record of a client with a nasogastric tube that has been attached to suction for 2 days shows greater output than input. Which nursing diagnoses are most applicable? Select all that apply. 1) Deficient Fluid Volume 2) Risk for Deficient Fluid Volume 3) Impaired Oral Mucous Membranes 4) Impaired Gas Exchange 5) Decreased Cardiac Output

1) Deficient Fluid Volume 3) Impaired Oral Mucous Membranes 5) Decreased Cardiac Output

The primary purpose of the evaluating phase of the care planning process is to determine whether 1) Desired outcomes have been met. 2) Nursing activities were carried out. 3) Nursing activities were effective. 4) Client's condition has changed

1) Desired outcomes have been met. Rationale: The desired outcomes and indicator statements reflect the parameters by which success will be measured. The goal can be met even if the nursing activities were not carried out or were ineffective. Although the desired outcome, by definition, indicates a change in the client's condition (behavior, knowledge, or attitude), only specific changes (desired outcomes) reflect the success of the care plan.

The school nurse is teaching health-promoting behaviors that improve sleep to a group of high school students. Which points should be included in the education? (Select all that apply.) 1) Do not study in your bed. 2) Go to sleep each night whenever you feel tired. 3) Turn off your cell phone at bedtime. 4) Avoid drinking coffee or soda before bedtime. 5) Turn on the television to help you fall asleep.

1) Do not study in your bed. 3) Turn off your cell phone at bedtime. 4) Avoid drinking coffee or soda before bedtime.

The client's postoperative orders state "diet as tolerated." The client has been NPO. The nurse will advance the client's diet to clear liquids based on which assessment? Select all that apply. 1) Does not complain of nausea or vomiting. 2) Pain level is maintained at a rating of 2-3 out of 10. 3) States passing flatus. 4) Ambulates with minimal assistance. 5) Expresses feeling "hungry."

1) Does not complain of nausea or vomiting. 3) States passing flatus.

During a well-child visit, a mother tells the nurse that her 4- year old daughter typically goes to bed at 10:30 PM and awakens each morning at 7 AM. She does not take a nap in the afternoon. Which is the best response by the nurse? 1) Encourage the mother to consider putting her daughter to bed between 8 and 9 PM. 2) Reassure the mother that it is normal for 4-year-olds to resist napping, but encourage her to insist that she rest quietly each afternoon. 3) Recommend that her daughter be allowed to sleep later in the morning. 4) Reassure her that her daughter's sleep pattern is normal and that she has outgrown her need for an afternoon nap.

1) Encourage the mother to consider putting her daughter to bed between 8 and 9 PM.

The nurse is gathering a sleep history from a patient who is being evaluated for obstructive sleep apnea. Which common symptoms does the patient most likely report? (Select all that apply.) 1) Headache 2) Early wakening 3) Excessive daytime sleepiness 4) Difficulty falling asleep 5) Snoring

1) Headache 3) Excessive daytime sleepiness 5) Snoring

The client with a fractured pelvis requests that family members be allowed to stay overnight in the hospital room. Before determining whether or not this request can be honored, the nurse should consult which of the following? 1) Hospital policies 2) Standardized care plans 3) Orthopedic protocols 4) Standards of care

1) Hospital policies Rationale: Policy and procedure documents provide data about how certain situations are handled. Standardized care plans (option 2) and standards of care (option 4) are written for groups of clients with similar medical or nursing diagnoses. They generally do not address questions such as hospital routines and nonmedical client needs. Note: Even hospital policies are not absolute. Each situation must be analyzed and responded to individually. Orthopedic protocols (option 3) would address elements specifically associated with the surgery, not whether the family slept in the room.

Which of the following behaviors is most representative of the nursing diagnosis phase of the nursing process? 1) Identifying major problems or needs. 2) Organizing data in the client's family history. 3) Establishing short-term and long-term goals. 4) Administering an antibiotic.

1) Identifying major problems or needs. Rationale: Identifying problems/needs is part of a nursing diagnosis. For example, a client with difficulty breathing would have Impaired Gas Exchange related to constricted airways as manifested by shortness of breath (dyspnea) as a nursing diagnosis. Organizing the family history is part of the assessment phase. Establishing goals is a part of the planning phase. Administering an antibiotic is part of the implementation phase.

When caring for an obese client 4 to 5 days post-surgery, who has nausea and occasional vomiting and is not keeping fluids down well, which of the following would you be most concerned about? 1. Post surgical hemorrhage and anemia 2. Wound dehiscence and evisceration 3. Impaired skin integrity and decubitus ulcers 4. Loss of motility and paralytic illeus

2. Wound dehiscence and evisceration; Wound dehiscence is most likely to occur 4 to 5 days postoperatively, and risk factors include obesity, poor nutrition, multiple trauma, failure of suturing, excessive coughing, vomiting, and dehydration.

Which of the following are allowed on a full liquid diet? 1. Scrambled eggs 2.Chocolate Pudding 3.Tomatoe Juice 4.Hard Candy 5.Mashed Potatoes 6.Cream of Wheat cereal 7.Oatmeal cereal 8.Fruit "smoothies"

2.Chocolate Pudding 3.Tomatoe Juice 4.Hard Candy 6.Cream of Wheat cereal 8.Fruit "smoothies"

The nurse selects the nursing diagnosis of Risk for Impaired Skin Integrity related to immobility, dry skin, and surgical incision. Which of the following represents a properly stated outcome/goal? The client will 1) Turn in bed q2h. 2) Report the importance of applying lotion to skin daily. 3) Have intact skin during hospitalization. 4) Use a pressure-reducing mattress.

3) Have intact skin during hospitalization. Rationale: The goal or outcome should state the opposite of the nursing diagnosis stem, and thus healthy intact skin is the reverse condition of impaired skin integrity. Turning in bed, applying lotion, and using a special mattress are all interventions that may result in achieving the goal (options 1, 2, and 4).

During the transduction phase of nociception, which method of pain control is most effective? 1) Tricyclic antidepressants 2) Opioids 3) Ibuprofen 4) Distraction

3) Ibuprofen Rationale: During the transduction phase, tissue injury triggers the release of biochemical mediators such as prostaglandin. Ibuprofen works by blocking the production of prostaglandin. The coanalgesic medication in option 1 would affect the modulation phase because coanalgesics inhibit the reuptake of norepinephrine and serotonin, which increases the modulation phase that helps inhibit painful ascending stimuli. Opioids block the release of neurotransmitters, particularly substance P, which stops the pain at the spinal level that occurs during the transmission phase (option 2). Distraction is best used during the perception phase when the client becomes conscious of the pain. Distraction (e.g., music, guided imagery, TV) can help direct the client's attention away from the pain (option 4).

The care plan calls for administration of a medication plus client education on diet and exercise for high blood pressure. The nurse finds the blood pressure extremely elevated. The client is very distressed with this finding. Which nursing skill of implementing would be needed most? 1) Cognitive 2) Intellectual 3) Interpersonal 4) Psychomotor

3) Interpersonal Rationale: This client needs psychosocial support rather than skills related to knowledge (options 1 and 2) or hands-on activity (option 4).

The care plan includes a nursing intervention "4/2/11 Measure client's fluid intake and output. F. Jenkins, RN." What element of a proper nursing intervention has been omitted? 1) Action verb 2) Content 3) Time 4) None

3) Time Rationale: Although there may be standard policies or routines for measuring intake and output, the nursing intervention should specify if this is to be done "routinely" or at specific intervals (e.g., q4h). The nurse is also aware, however, that critical thinking indicates that the intake and output should be monitored more frequently than ordered if assessment reveals abnormal findings.

During the implementation step of the nursing process, a nurse reviews and revises the nursing plan of care. Place the following steps of review and revision in the correct order: 1. Review the care plan. 2. Decide if the nursing interventions remain appropriate. 3. Reassess the patient. 4. Compare assessment findings to validate existing nursing diagnoses.

3, 1, 4, 2 After reassessing a patient, the nurse reviews the care plan and compares assessment data to validate the nursing diagnoses and determine whether the nursing interventions remain the most appropriate for the clinical situation. If the patient's status has changed and the nursing diagnosis and related nursing interventions are no longer appropriate, the nurse modifies the nursing care plan.

Your client has a pressure ulcer over the sacral area that is believed to be due to shearing force. The client's family asks you to explain shearing force. You would be most accurate if you tell the family that shearing force involves: 1. A tearing of the muscle tissue due to a considerable downward force. 2. A sudden break in skin integrity due to being pulled against the bed linens. 3. A superficial skin fold getting pinched, and tissues irritated by the pressure. 4. Superficial skin surface relatively unmoving in relation to the bed surface.

3. A superficial skin fold getting pinched, and tissues irritated by the pressure; Shearing force is a combination of friction and pressure with skin surface unmoving in relation to the bed surface, while deeper tissue attached to the skeleton tends to move with the body.

A primary care provider admitted a client experiencing hypertensive crisis because of failure to take his prescribed medications. To determine learning needs, which client assessment by the nurse would have the highest priority? 1. Age 2. Perception of the effects of hypertension 3. Ability to purchase needed medication 4. Support system

3. Ability to purchase needed medication

When does implementation begin as the fourth step of the nursing process? 1. During the assessment phase 2. Immediately in some critical situations 3. After the care plan has been developed 4. After there is mutual goal setting between nurse and patient

3. After the care plan has been developed Implementation begins after the nurse has developed the plan of care. Even in emergent situations, a nurse assesses a situation quickly, considers options, and then implements nursing measures. Goal setting is part of planning.

Which is the best method of helping a client newly diagnosed with diabetes to learn the dietary requirements associated with the disease? 1. Provide a videotape that addresses the dietary requirements associated with the disease 2. Ask a nutritionist to visit the client to present information and handouts about the diabetic diet 3. Ask the client to make a list of her favorite foods and how to work them into her diet 4. Have the client attend a group meting for clients with diabetes to discuss their adaptation to this chronic health condition

3. Ask the client to make a list of her favorite foods and how to work them into her diet

A nurse caring for a patient with pneumonia sits the patient up in bed and suctions his airway. After suctioning, the patient describes some discomfort in his abdomen. The nurse auscultates the patient's lung sounds and gives him a glass of water. Which of the following is an evaluative measure used by the nurse? 1. Suctioning the airway 2. Sitting patient up in bed 3. Auscultating lung sounds 4. Patient describing type of discomfort

3. Auscultating lung sounds Auscultation was the measure used to determine if the suctioning of the airway was effective. Suctioning and sitting the patient up are interventions. The nurse did not ask the patient or evaluate the nature of the pain.

What type of interview techniques does the nurse use when asking these questions, "Do you have pain or cramping?" "Does the pain get worse when you walk?" (Select all that apply.) 1. Active listening 2. Open-ended questioning 3. Closed-ended questioning 4. Problem-oriented questioning

3. Closed-ended questioning 4. Problem-oriented questioning

Two nurses are having a discussion at the nurses' station. One nurse is a new graduate who added, "Patient needs improved bowel function related to constipation" to a patient's care plan. The nurse's colleague, the charge nurse says, "I think your diagnosis is possibly worded incorrectly. Let's go over it together." A correctly worded diagnostic statement is: 1. Need for improved bowel function related to change in diet. 2. Patient needs improved bowel function related to alteration in elimination. 3. Constipation related to inadequate fluid intake. 4. Constipation related to hard infrequent stools.

3. Constipation related to inadequate fluid intake.

A physician writes an order to apply a wrist restraint to a patient who has been pulling out a surgical wound drain. Place the following steps for applying the restraint in the correct order. 1. Explain what you plan to do. 2. Wrap a limb restraint around wrist or ankle with soft part toward skin and secure. 3. Determine that restraint alternatives fail to ensure patient's safety. 4. Identify the patient using proper identifier. 5. Pad the patient's wrist.

3. Determine that restraint alternatives fail to ensure patient's safety. 4. Identify the patient using proper identifier. 1. Explain what you plan to do. 5. Pad the patient's wrist. 2. Wrap a limb restraint around wrist or ankle with soft part toward skin and secure.

A nurse is preparing for changeofshift rounds with the nurse who is assuming care for his patients. Which of the following statements or actions by the nurse are characteristics of ineffective handoff communication? 1. This patient is anxious about his pain after surgery; you need to review the information I gave him about how to use a patientcontrolled analgesia (PCA) pump this evening. 2. The nurse refers to the electronic care plan in the electronic health record (EHR) to review interventions for the patient's care. 3. During walking rounds the nurse talks about the problem the patient care technicians created by not ambulating the patient. 4. The nurse gives her patient a pain medication before report so there is likely to be no interruption during rounding.

3. During walking rounds the nurse talks about the problem the patient care technicians created by not ambulating the patient.

A client is admitted to the Emergency Department after a motorcycle accident that resulted in the client's skidding across a cement parking lot. Since the client was wearing shorts, there are large areas on the legs where the skin is ripped off. This wound is best described as: 1. Abrasion 2. Unapproximated 3. Laceration 4. Eschar

3. Laceration; Laceration best describes the wound, because skin is ripped off. An abrasion is a scrape. Unapproximated is a general term for a wound that is not closed. Eschar is a scab-like covering over a wound.

A clinic nurse assesses a patient who reports a loss of appetite and a 15-pound weight loss since 2 months ago. The patient is 5 feet 10 inches tall and weighs 135 pounds (61.2 kg). She shows signs of depression and does not have a good understanding of foods to eat for proper nutrition. The nurse makes the nursing diagnosis of imbalanced nutrition: less than body requirements related to reduced intake of food. For the goal of, "Patient will return to baseline weight in 3 months," which of the following outcomes would be appropriate? (Select all that apply.) 1. Patient will discuss source of depression by next clinic visit. 2. Patient will achieve a calorie intake of 2400 daily in 2 weeks. 3. Patient will report improvement in appetite in 1 week. 4. Patient will identify food protein sources.

3. Patient will report improvement in appetite in 1 week. With the related factor of reduced intake of food, the outcomes should focus on behaviors that reflect an increase in intake. Thus achieving an increase in calories and an improved appetite for food would be appropriate. The patient's depression probably contributes to the loss of appetite, but being able to discuss the source of depression is not an outcome for improving her baseline weight. Being able to identify protein sources would improve any knowledge deficit the patient might have but would not help her gain weight

A nursing student is talking with one of the staff nurses who works on a surgical unit. The student's care plan is to include nursing-sensitive outcomes for the nursing diagnosis of acute pain. A nursing-sensitive outcome suitable for this diagnosis would be: 1. Patient will achieve pain relief by discharge. 2. Patient will be free of a surgical wound infection by discharge. 3. Patient will report reduced pain severity in 2 days. 4. Patient will describe purpose of pain medicine by discharge.

3. Patient will report reduced pain severity in 2 days. An example of a nursing-sensitive outcome is one that is influenced and sensitive to nursing interventions. Such is the case with "reduction in pain severity." The patient achieving pain relief by discharge is a goal. The patient being free of a surgical wound infection by discharge is a medical outcome. The patient describing the purpose of pain medication by discharge is an outcome for a knowledge problem but not for the diagnosis of acute pain.

A patient is being discharged today. In preparation the nurse removes the intravenous (IV) line from the right arm and documents that the site was "clean and dry with no signs of redness or tenderness." On discharge the nurse reviews the care plan for goals met. Which of the following goals can be evaluated with what you know about this patient? 1. Patient expresses acceptance of health status by day of discharge. 2. Patient's surgical wound will remain free of infection. 3. Patient's IV site will remain free of phlebitis. 4. Patient understands when to call physician to report possible complications.

3. Patient's IV site will remain free of phlebitis. To achieve the goal of preventing phlebitis the nurse evaluates for signs of phlebitis, which include redness or inflammation. The outcome for this goal would be stated as, "IV site will show no signs of inflammation to discharge."

A nurse is orienting a new graduate nurse to the unit. The graduate nurse asks, "Why do we have standing orders for cases when patients develop life-threatening arrhythmias? Is not each patient's situation unique?" What is the nurse's best answer? 1. Standing orders are used to meet our physician's preferences. 2. Standing orders ensure that we are familiar with evidence based guidelines for care of arrhythmias. 3. Standing orders allow us to respond quickly and safely to a rapidly changing clinical situation. 4. Standing orders minimize the documentation we have to provide.

3. Standing orders allow us to respond quickly and safely to a rapidly changing clinical situation. Standing orders are preprinted documents containing orders for the conduct of routine therapies, monitoring guidelines, and/or diagnostic procedures for specific patients with identified clinical problems. They are common in critical care settings and other specialized practice settings in which patients' needs change rapidly and require immediate attention.

Which outcome allows you to measure a patient's response to care more precisely? 1. The patient's wound will appear normal within 3 days. 2. The patient's wound will have less drainage within 72 hours. 3. The patient's wound will reduce in size to less than 4 cm (1 1/2 inches) by day 4. 4. The patient's wound will heal without redness or drainage by day 4.

3. The patient's wound will reduce in size to less than 4 cm (1 1/2 inches) by day 4.

An older Asian client has mild dysphagia from a recent stroke. The nurse plans the client's meals based on the need to: 1.Have at least one serving of thick dairy (e.g., pudding, ice cream) per meal 2.Eliminate the beer usually ingested every evening 3.Include as many of the client's favorite foods as possible 4.Increase the calories from lipids to 40%

3.Include as many of the client's favorite foods as possible

Under what circumstances is it considered acceptable practice for the nurse to document a nursing activity before it is carried out? 1) When the activity is routine (e.g., raising the bed rails). 2) When the activity occurs at regular intervals (e.g., turning the client in bed). 3) When the activity is to be carried out immediately (e.g., a stat medication). 4) It is never acceptable

4) It is never acceptable. Rationale: It is never acceptable practice for the nurse to document a nursing activity before it is carried out. This would be very unsafe because many things can cause an activity to be postponed or canceled and prior charting would be inaccurate, misleading, and potentially dangerous. In a few situations, it may be permissible to chart frequent or routine activities some time following the activities such as at the end of a shift or after a particular interval (e.g., every 4 hours) rather than immediately following the activity

Which of the following would be true regarding use of the observing method of data collection? 1) When observing, the nurse uses only the visual sense. 2) Observing is done only when no other nursing interventions are being performed at the same time. 3) Data should be gathered as it occurs, rather than in any particular order. 4) Observed data should be interpreted in relation to other sources of collected data.

4) Observed data should be interpreted in relation to other sources of collected data. Rationale: Interpreting collected data is necessary to help validate its accuracy. Observing includes the senses of smell, hearing, and touch in addition to vision (option 1). Using priority setting, observing must often be performed simultaneously with other activities (option 2). A systematic approach to observing data helps ensure nothing is missed and the nurse pays attention to the most important data first (option 4).

An element of quality improvement, rather than quality assurance, is which of the following? 1) Focus is on individual outcomes. 2) Evaluates organizational structures. 3) Aims to confirm that quality exists. 4) Plans corrective actions for problems.

4) Plans corrective actions for problems. Rationale: Quality improvement plans corrective actions for problems. QI focuses on process rather than outcomes (option 1), client care rather than structure (option 2), and aims for improvement rather than confirmation of quality (option 3).

When written properly, NOC outcomes and indicators 1) Do not require customization. 2) Address several nursing diagnoses. 3) Are broad statements of desired end points. 4) Reflect both the nurse's and the client's values.

4) Reflect both the nurse's and the client's values. Rationale: NOC outcomes should reflect both the nurse's and the client's values of what is trying to be achieved. The outcomes still must be customized (option 1), but address only one nursing diagnosis at a time (option 2). Outcomes are narrow/specific end points, not broad (option 3).

One of the primary advantages of using a three-part diagnostic statement such as the problem-etiology-signs/symptoms (PES) format includes which of the following? 1) Decreases the cost of health care. 2) Improves communication between nurse and client. 3) Helps the nurse focus on health and wellness elements. 4) Standardizes organization of client data.

4) Standardizes organization of client data. Rationale: The PES format assists with comprehensive and accurate organization of client data. More efficient planning may or may not reduce health care costs. Nursing diagnostic statements should be confirmed with the client but using PES does not ensure this. PES statements can be wellness or illness focused.

When teaching a patient about transcutaneous electrical nerve stimulation (TENS), which information do you include? 1) TENS works by causing distraction. 2) TENS therapy does not require a health care provider's order. 3) TENS requires an electrical source for use. 4) TENS electrodes are applied near or directly on the site of pain.

4) TENS electrodes are applied near or directly on the site of pain.

Which nursing measure best promotes sleep in a school-age child? 1) Encourage evening exercise 2) Offer a glass of hot chocolate before bedtime 3) Make sure that the room is dark and quiet 4) Use quiet activities consistently before bedtime

4) Use quiet activities consistently before bedtime

The nurse checks the intravenous (IV) solution that is infusing into the patient's left arm. The IV solution of 9% NS is infusing at 100 mL/hr as ordered. The nurse reviews the nurses' notes from the previous shift to determine if the dressing over the site was changed as scheduled per standard of care. While in the room, the nurse inspects the condition of the dressing and notes the date on the dressing label. In what ways did the nurse evaluate the IV intervention? (Select all that apply.) 1. Checked the IV infusion location in left arm 2. Checked the type of IV solution 3. Confirmed from nurses' notes the time of dressing change and checked label 4. Inspected the condition of the IV dressing

1. Checked the IV infusion location in left arm 3. Confirmed from nurses' notes the time of dressing change and checked label 4. Inspected the condition of the IV dressing Evaluation often reveals changes that are not obvious. Changes are often subtle and occur over a period of time

Proper technique for performing a wound culture includes what? 1. Cleansing the wound prior to obtaining the specimen. 2. Swabbing for the specimen in the area with the largest collection of drainage. 3. Removing crusts or scabs with sterile forceps and then culturing the site beneath. 4. Waiting 8 hours following a dose of antibiotic to obtain the specimen.

1. Cleansing the wound prior to obtaining the specimen; Wound culture specimens should be obtained from a cleaned area of the wound. Microbes responsible for infection are more likely to be found in viable tissue. Collected drainage contains old and mixed organisms. An appropriate specimen can be obtained without causing the client the discomfort of debriding. The nurse does not generally debride a wound to obtain a specimen. Once systemic antibiotics have been begun, the interval following a does will not significantly affect the concentration of wound organisms.

Before consulting with a physician about a patient's need for urinary catheterization, the nurse considers the fact that the patient has urinary retention and has been unable to void on her own. The nurse knows that evidence for alternative measures to promote voiding exists, but none has been effective, and that before surgery the patient was voiding normally. This scenario is an example of which implementation skill? 1. Cognitive 2. Interpersonal 3. Psychomotor 4. Consultative

1. Cognitive Thinking and anticipating how to approach implementation involve a cognitive implementation skill. The nurse considers the rationale for an intervention and evidence in nursing science that supports that intervention or alternatives.

Which of the following statements correctly describe the evaluation process? (Select all that apply.) 1. Evaluation is an ongoing process. 2. Evaluation usually reveals obvious changes in patients. 3. Evaluation involves making clinical decisions. 4. Evaluation requires the use of assessment skills.

1. Evaluation is an ongoing process. 3. Evaluation involves making clinical decisions. 4. Evaluation requires the use of assessment skills.

For each of the following interventions, note which are direct and which are indirect nursing interventions. Place a D for direct or I for indirect in the space provided 1. A nurse checks the monthly performance improvement report on fall occurrences on a unit. _________ 2. A nurse discusses with the patient exercise restrictions to follow on return home. _________ 3. A nurse consults with a dietitian about a patient's therapeutic diet food choices. ________ 4. A nurse administers a tube feeding. _______ 5. A nurse assists a colleague in applying a complex dressing to a patient's wound. _______

1. I 2. D 3. I 4. D 5. I

A client needs to learn to self-administer insulin injections. Which statements reflect possible low literacy skills? Select all that apply 1. I will read the information later- I'm too tired right now 2. I've watched my brother give his own shots. I know how to do it. 3. Just show my wife 4. Do you have a video showing how I should give myself the shot? 5. I don't understand this one section in the handout

1. I will read the information later- I'm too tired right now 2. I've watched my brother give his own shots. I know how to do it. 3. Just show my wife 4. Do you have a video showing how I should give myself the shot?

A patient signals the nurse by turning on the call light. The nurse enters the room and finds the patient's drainage tube disconnected, 100 mL of fluid in the intravenous (IV) line, and the patient asking to be turned. Which of the following does the nurse perform first? 1. Reconnect the drainage tubing 2. Inspect the condition of the IV dressing 3. Improve the patient's comfort and turn onto her side. 4. Obtain the next IV fluid bag from the medication room

1. Reconnect the drainage tubing

Unmet and partially met goals require the nurse to do which of the following? (Select all that apply.) 1. Redefine priorities 2. Continue intervention 3. Discontinue care plan 4. Gather assessment data on a different nursing diagnosis

1. Redefine priorities 2. Continue intervention When you determine that a goal has not been met or has been met only partially, intervention must continue; and the fact that the health problem still exists suggests that priorities may need to be redefined. You do not discontinue a plan unless a goal has been achieved. Evaluation never involves comparing a patient's data with that of another patient. A patient may develop new diagnoses at any time, but assessment of a new diagnosis does not address goals for an existing diagnosis.

The following nursing diagnoses all apply to one patient. As the nurse adds these diagnoses to the care plan, which diagnoses will not include defining characteristics? 1. Risk for aspiration 2. Acute confusion 3. Readiness for enhanced coping 4. Sedentary lifestyle

1. Risk for aspiration

Which of the following are examples of data validation? (Select all that apply.) 1. The nurse assesses the patient's heart rate and compares 2. The nurse asks the patient if he is having pain and then asks the patient to rate the severity. 3. The nurse observes a patient reading a teaching booklet and asks the patient if he has questions about its content. 4. The nurse obtains a blood pressure value that is abnormal and asks the charge nurse to repeat the measurement. 5. The nurse asks the patient to describe a symptom by saying, "Go on."

1. The nurse assesses the patient's heart rate and compares 4. The nurse obtains a blood pressure value that is abnormal and asks the charge nurse to repeat the measurement.

A nurse gathers the following assessment data. Which of the following cues form(s) a pattern suggesting a problem? (Select all that apply.) 1. The skin around the wound is tender to touch. 2. Fluid intake for 8 hours is 800 mL. 3. Patient has a heart rate of 78 and regular. 4. Patient has drainage from surgical wound. 5. Body temperature is 101° F (38.3° C). 6. Patient asks, "I'm worried that I won't return to work when I planned."

1. The skin around the wound is tender to touch 4. Patient has drainage from surgical wound. 5. Body temperature is 101° F (38.3° C).

When working with an older person, you would keep in mind that the older person is most likely to experience which of following changes with aging? 1. Thinning of the epidermis 2. Thickening of the epidermis 3. Oiliness of the skin 4. Increased elasticity of the skin

1. Thinning of the epidermis, The epidermis thins with aging, and there is decreased strength and elasticity of the skin, increased dryness and scaliness of the skin, and diminished pain perception due to decreased sensation of pressure and light touch.

The nurse cares for a client with a wound in the late regeneration phase of tissue repair. The wound may be protected by applying a: 1. Transparent film 2. Hydrogel dressing 3. Collogenase dressing 4. Wet to dry dressing

1. Transparent film; Wounds in the regeneration phase of healing need to be protected as new tissue grows. Answers 2, 3, and 4 are dressings used to remove nonviable tissue.

A nurse identifies several interventions to resolve the patient's nursing diagnosis of impaired skin integrity. Which of the following are written in error? (Select all that apply.) 1. Turn the patient regularly from side to back to side. 2. Provide perineal care, using Dove soap and water, every shift and after each episode of urinary incontinence. 3. Apply a pressurerelief device to bed. 4. Apply transparent dressing to sacral pressure ulcer.

1. Turn the patient regularly from side to back to side. 3. Apply a pressurerelief device to bed.

When receiving a report at the beginning of your shift, you learn that your assigned client has a surgical incision that is healing by primary intention. You know that your client's incision is: 1. Well approximated, with minimal or no drainage. 2. Going to take a little longer than usual to heal. 3. Going to have more scarring than most incisions. 4. Draining some serosanguineous drainage.

1. Well approximated, with minimal or no drainage; Primary intention means that the wound edges are well approximated, with minimal or no tissue loss as well as formation of minimal granulation tissue and scarring.

A 55-year-old female is about 20 pounds over her desired weight. She has been on a "low-calorie" diet with no improvement. Which statement reflects a healthy approach to the desired weight loss? "I need to: 1.Increase my exercise to at least 30 minutes every day." 2.Switch to a low-carbohydrate diet." 3.Keep a list of my forbidden foods on hand at all times." 4.Buy more organic and less processed foods."

1.Increase my exercise to at least 30 minutes every day."

A 72yearold patient has come to the health clinic with symptoms of a productive cough, fever, increased respiratory rate, and shortness of breath. His respiratory distress increases when he walks. He lives alone and did not come to the clinic until his neighbor insisted. He reports not getting his pneumonia vaccine this year. Blood tests show the patient's oxygen saturation to be lower than normal. The physician diagnoses the patient as having pneumonia. Match the priority level with the nursing diagnoses identified for this patient: Nursing Diagnoses 1. Impaired gas exchange _____ 2. Risk for activity intolerance _____ 3. Ineffective selfhealth management _____ Priority Level a. Long term b. Short term c. Intermediate

1b 2c 3a

A nurse on a cancer unit is reviewing and revising the written plan of care for a patient who has the nursing diagnosis of nausea. Place the following steps in their proper order: 1. The nurse revises approaches in the plan for controlling environmental factors that worsen nausea. 2. The nurse enters data in the assessment column showing new information about the patient's nausea. 3. The nurse adds the current date to show that the diagnosis of nausea is still relevant. 4. The nurse decides to use the patient's self-report of appetite and fluid intake as evaluation measures.

2 3 1 4

A nurse is conducting a patient-centered interview. Place the statements from the interview in the correct order. 1."You say you've lost weight. Tell me how much weight you have lost in the last month." 2."My name is Todd. I'll be the nurse taking care of you today. I'm going to ask you a series of questions to gather your health history." 3."I have no further questions. Thank you for your patience." 4."Tell me what brought you to the hospital." 5."So, to summarize, you've lost about 6 pounds in the last month, and your appetite has been poor—correct?"

2 4 1 5 3

Place the following activities of planning in the correct order of their use 1) Establish goals/outcomes. 2) Write the care plan. 3) Set priorities. 4) Choose interventions

2 4 1 3 Rationale: In planning, first the nurse sets priorities and then writes goals/outcomes, selects interventions, and then writes the nursing care plan.

The nurse found a 68-year-old female patient wandering in the hall. The patient says she is looking for the bathroom. Which interventions are appropriate to ensure the safety of the patient? (Select all that apply.) 1 Insert a urinary catheter. 2 Leave a night light on in the bathroom. 3 Ask the physician to order a restraint. 4 Keep the bed in low position with upper and lower side rails up. 5 Assign a staff member to stay with the patient. 6 Provide scheduled toileting during the night shift. 7 Keep the pathway from the bed to the bathroom clear.

2 Leave a night light on in the bathroom. 6 Provide scheduled toileting during the night shift. 7 Keep the pathway from the bed to the bathroom clear.

Which statement best reflects the nurse's assessment of the fifth vital sign? 1) "Do you have any complaints?" 2) "Are you experiencing any discomfort right now?" 3) "Is there anything I can do for you now?" 4) "Do you have any complaints of pain?"

2) "Are you experiencing any discomfort right now?"

A client who is having a mastectomy expresses sadness about losing her breast. Based on this information, the nurse would identify that the client is at risk for which nursing diagnosis? 1) Body Image Disturbance 2) Anticipatory Grieving 3) Fear 4) Ineffective Coping

2) Anticipatory Grieving

Which of the following represents application of the components of evaluating? 1) Goal achievement must be written as either completely met or unmet. 2) Data related to expected outcomes must be collected. 3) If the outcome was achieved, conclude that the plan was effective. 4) After determining that the outcome was not met, start over with a new nursing care plan.

2) Data related to expected outcomes must be collected. Rationale: Evaluating requires that client behavior be compared to expected outcomes. Goals may be partially met in addition to completely met or unmet (option 1). An outcome may be achieved but not be a direct result of the plan or interventions (option 3). A care plan should be continued, modified, or terminated based on achievement of outcomes (option 4).

In the diagnostic statement "Excess Fluid Volume related to decreased venous return as manifested by lower extremity edema (swelling)," the etiology of the problem is which of the following? 1) Excess fluid volume 2) Decreased venous return 3) Edema 4) Unknown

2) Decreased venous return Rationale: Because the venous return is impaired, fluid is static, resulting in swelling. Therefore, decreased venous return is the cause (etiology) of the problem. Excess Fluid Volume is the nursing diagnosis, and edema of the lower extremity is the sign/symptom or critical attribute. The cause is known.

The nurse is conducting the diagnosing phase (nursing diagnosis) of the nursing process for a client with a seizure disorder. Which of the following steps exists between data analysis and formulating the diagnostic statement? 1) Assess the client's needs. 2) Delineate the client's problems and strengths. 3) Determine which interventions are most likely to succeed. 4) Estimate the cost of several different approaches.

2) Delineate the client's problems and strengths. Rationale: In diagnosing, data from assessment (option 1) are analyzed and problems, risks, and strengths are identified before diagnostic statements can be established. Interventions (option 3) are more commonly part of the planning and implementing phases of the nursing process. Cost (option 4) is an important consideration but would be estimated in the planning phase.

Which of the following represent effective planning of the interview setting? Select all that apply. 1) Keep the lighting dimmed so as not to stress the client's eyes. 2) Ensure that no one can overhear the interview conversation. 3) Stand near the client's head while they are in the bed or chair. 4) Keep approximately 3 feet from the client during the interview. 5) Use a standard form to be sure all relevant data are covered in the interview.

2) Ensure that no one can overhear the interview conversation. 4) Keep approximately 3 feet from the client during the interview. 5) Use a standard form to be sure all relevant data are covered in the interview. Rationale: The nurse plans the interview so that privacy is observed. A comfortable distance between nurse and client to respect the client's personal space is about 3 feet. Using a standard form will help ensure the nurse doesn't omit gathering any vital information. Lighting should be at a normal level—neither bright nor dim (option 1). The nurse should be at the same height as the client, usually sitting, at approximately a 45-degree angle facing the client. The nurse standing over the client creates an uncomfortable atmosphere for an interview (option 3).

The nurse plans to remove the client's sutures. Which action demonstrates appropriate standards of care? Select all that apply. 1) Use clean technique. 2) Grasp the suture at the knot with a pair of forceps. 3) Place the curved tip of the suture scissors under the suture as close to the skin as possible. 4) Pull the suture material that is visible beneath the skin during removal. 5) Remove alternate sutures first.

2) Grasp the suture at the knot with a pair of forceps. 3) Place the curved tip of the suture scissors under the suture as close to the skin as possible. 5) Remove alternate sutures first.

The nurse assesses a postoperative client who has a rapid, weak pulse; urine output less than 30 mL/hr; and decreased blood pressure. The client's skin is cool and clammy. What complication should the nurse suspect? 1) Thrombophlebitis 2) Hypovolemic shock 3) Pneumonia 4) Wound dehiscence

2) Hypovolemic shock

The client has a high-priority nursing diagnosis of Risk for Impaired Skin Integrity related to the need for several weeks of imposed bed rest. The nurse evaluates the client after 1 week and finds the skin integrity is not impaired. When the care plan is reviewed, the nurse should perform which of the following? 1) Delete the diagnosis since the problem has not occurred. 2) Keep the diagnosis since the risk factors are still present. 3) Modify the nursing diagnosis to Impaired Mobility. 4) Demote the nursing diagnosis to a lower priority.

2) Keep the diagnosis since the risk factors are still present. Rationale: There is no reason to delete or modify the nursing diagnosis or demote its priority because the risk factors that prompted it are still present.

Which interventions, when implemented by the nurse, would apply the gate control theory of pain? Select all that apply. 1) Oral analgesics around the clock 2) Massage 3) Patient-controlled analgesia 4) Heat or cold application 5) Teaching

2) Massage 4) Heat or cold application 5) Teaching

The nurse assesses a postoperative client with an abdominal wound and finds the client drowsy when not aroused. The client's pain is ranked 2 on a scale of 0 to 10, vital signs are within preoperative range, extremities are warm with good pulses but very dry skin. The client declines oral fluids due to nausea, and reports no bowel movement in the past 2 days. Hip dressing is dry with drains intact. Which element is most likely to be considered of high priority for a change in the current care plan? 1) Pain 2) Nausea 3) Constipatio 4) Potential for wound infection

2) Nausea Rationale: More detailed assessment data and consultation with the client would be needed to absolutely confirm the priority. Postoperative nausea to the level of inhibiting oral intake has the greatest likelihood of leading to complications and requires nursing intervention now. The client's pain level is not extreme considering the recency of the surgery, and pain intervention can be assumed to be effective (option 1). Although the constipation is probably bordering on abnormal, a nursing intervention would most likely begin with oral treatment, which is not possible due to the nausea. More invasive interventions such as an enema or suppository would not be commonly administered the first day postoperative (option 3). Wound infection can occur, but there are no data to indicate that this requires a change in the current plan (option 4).

When a client has arrived at the nursing unit from surgery, the nurse is most likely to give priority to which of the following assessments? 1) Pain tolerance 2) Pain intensity 3) Location of pain 4) Pain history

2) Pain intensity Rationale: The client's pain intensity needs to be assessed first for effective pain management. In a postoperative client it is important to assess pain intensity frequently to manage the acute pain experience. Option 1: The most pain a person is willing to tolerate before taking action can be discussed with the client after the pain intensity has been assessed. Option 3, location of pain, is important, but it is not the priority. Option 4: This information is important but not for a client in acute pain. The priority would be to assess the pain intensity.

If the nurse planned to evaluate the length of time clients must wait for a nurse to respond to a client need reported over the intercom system on each shift, which process does this reflect? 1) Structure evaluation 2) Process evaluation 3) Outcome evaluation 4) Audit

2) Process evaluation Rationale: Because this assessment focuses on how care is provided, it is a process evaluation. A structure evaluation would focus on the setting (e.g., how well equipment functions), and outcome evaluations focus on changes in client status (e.g., whether reported satisfaction levels vary with type of person who answers the call light). An audit would be a chart or document review.

A patient with a history of a stroke that left her confused and unable to communicate returns from interventional radiology following placement of a gastrostomy tube. The health care provider's order reads as follows: "Vicodin 1 tab, per tube, q4 hours, prn." Which action by the nurse is most appropriate? 1) No action is required by the nurse because the order is appropriate. 2) Request to have the ordered changed to ATC for the first 48 hours. 3) Ask for a change of medication to meperidine (Demerol) 50 mg IVP, q3 hours, prn. 4) Begin the Vicodin when the patient shows nonverbal symptoms of pain.

2) Request to have the ordered changed to ATC for the first 48 hours.

The client's arterial blood gas results are pH 7.32; PaCO2 58; HCO3 32. The nurse knows that the client is experiencing which acid-base imbalance? 1) Metabolic acidosis 2) Respiratory acidosis 3) Metabolic alkalosis 4) Respiratory alkalosis

2) Respiratory acidosis

A man brings his elderly wife to the emergency department. He states that she has been vomiting and has had diarrhea for the past 2 days. She appears lethargic and is complaining of leg cramps. What should the nurse do first? 1) Start an IV. 2) Review the results of serum electrolytes. 3) Offer the woman foods that are high in sodium and potassium content. 4) Administer an antiemetic.

2) Review the results of serum electrolytes.

The nurse is providing health teaching for a patient using herbal compounds such as melatonin for sleep. Which points need to be included? (Select all that apply.) 1) Can cause urinary retention 2) Should not be used indefinitely 3) May cause diarrhea and anxiety 4) May interfere with prescribed medications 5) Can lead to further sleep problems over time 6) Are not regulated by the U.S. Food and Drug Administration (FDA)

2) Should not be used indefinitely 4) May interfere with prescribed medications 6) Are not regulated by the U.S. Food and Drug Administration (FDA)

Which of the following elements is best categorized as secondary subjective data? 1) The nurse measures a weight loss of 10 pounds since the last clinic visit. 2) Spouse states the client has lost all appetite. 3) The nurse palpates edema in lower extremities. 4) Client states severe pain when walking up stairs.

2) Spouse states the client has lost all appetite. Rationale: Primary data come from the client (option 4), whereas secondary data come from any other source (chart, family). Subjective data are covert (reported or an opinion), whereas objective data can be measured or validated (weight—option 1, edema—option 3). If the spouse had stated that the client had eaten only toast and tea, this would be secondary objective (measured) data.

A patient is being discharged home on an around-the-clock (ATC) opioid for chronic back pain. Because of this order, the nurse anticipates an order for which class of medication? 1) Stool softener 2) Stimulant laxative 3) H 2 receptor blocker 4) Proton pump inhibitor

2) Stimulant laxative

The primary care provider prescribed 5 mL of a medication to be given deep intramuscular for a 40-year-old female who is 5'7" tall and weighs 135 pounds. Which is the most appropriate method of administration? 1) A tuberculin syringe, #25-#27 gauge, 1/4- to 5/8-inch needle 2) Two 3-mL syringes, #20-#23 gauge, 1 1/2-inch needle 3) Two 2-mL syringes, #25 gauge, 5/8-inch needle 4) Two 2-mL syringes, #20-#23 gauge, 1-inch needle

2) Two 3-mL syringes, #20-#23 gauge, 1 1/2-inch needle Rationale: Five milliliters is too large an amount to inject into one site. The nurse needs to divide the amount into two 2.5-mL injections. A 3-mL syringe could be used. The length of the needle will depend on the muscle development of the client. The nurse needs to assess the client. The presumption, based on the information provided, is that this client's muscle mass is within normal limits. The needle length would need to be 1 1/2 inches because the medication is ordered to be given "deep IM." This also suggests that the medication should be given in the preferred site for IM injections—the ventrogluteal site—because it provides the greatest thickness of gluteal muscle. The gauge of the needle for an IM injection into the ventrogluteal muscle can range between #20 and #23. The nurse needs to assess the viscosity of the medication. Smaller gauges (e.g., #23) produce less tissue trauma; however, viscous solutions may require a larger gauge (e.g., #20-#21).

A client's family asks you to explain some keloid scars that the client developed. The best explanation of the keloid scars would be that keloid scars are: 1. Due to a relatively rare inherited tendency. 2. Caused by an abnormal amount of collagen being laid down in scar formation. 3. Most common in pale-skinned people of Northern European ancestry. 4. Caused by repeated and abrupt early disruption of eschar being formed.

2. Caused by an abnormal amount of collagen being laid down in scar formation; Keloid scars are due to an abnormal amount of collagen being laid down in scar formation in the maturation phase, and they are more apt to occur in a dark-skinned person.

A nurse checks a physician's order and notes that a new medication was ordered. The nurse is unfamiliar with the medication. A nurse colleague explains that the medication is an anticoagulant used for postoperative patients with risk for blood clots. The nurse's best action before giving the medication is to: 1. Have the nurse colleague check the dose with her before giving the medication. 2. Consult with a pharmacist to obtain knowledge about the purpose of the drug, the action, and the potential side effects. 3. Ask the nurse colleague to administer the medication to her patient. 4. Administer the medication as prescribed and on time.

2. Consult with a pharmacist to obtain knowledge about the purpose of the drug, the action, and the potential side effects. When a nurse performs a new or unfamiliar procedure, such as giving a new medication, it is important to assess personal competency and determine if new knowledge or assistance is needed. The nurse's best action is to check with the pharmacist about the medication. Having another nurse check the dosage is appropriate if the nurse is still uncertain about the medication. Once the nurse feels prepared, the medication is administered as prescribed. You never ask a colleague to give a medication to a patient to whom you are assigned.

The nurse reviews a patient's medical record and sees that tube feedings are to begin after a feeding tube is inserted. In recent past experiences the nurse has seen patients on the unit develop diarrhea from tube feedings. The nurse consults with the dietitian and physician to determine the initial rate that will be ordered for the feeding to lessen the chance of diarrhea. This is an example of what type of direct care measure? 1. Preventive 2. Controlling for an adverse reaction 3. Consulting 4. Counseling

2. Controlling for an adverse reaction Anticipating the need to start the feeding at a slower rate is an example of controlling for an adverse reaction, which in this case would be a harmful or unintended effect (diarrhea) of therapeutic intervention.

A home health nurse visits a client who twisted an ankle in the morning. The client has an ice bag on the ankle. Which one of the client's chronic conditions contraindicates the use of ice? 1. Gastritis 2. Diabetes 3. Glaucoma 4. Osteoporosis

2. Diabetes; Diabetes contradicts the use for ice. Clients with neurological or circulatory impairment are at risk for injury with ice use.

A patient is being discharged after abdominal surgery. The abdominal incision is healing well with no signs of redness or irritation. Following instruction, the patient has demonstrated effective care of the incision, including cleansing the wound and applying dressings correctly to the nurse. These behaviors are an example of: 1. Evaluative measure. 2. Expected outcome. 3. Reassessment. 4. Standard of care.

2. Expected outcome. An expected outcome is an end result that is measureable, desirable, observable, and translates into observable patient behaviors. It is a measure that tells you if the educational interventions led to successful goal achievement, the patient's self-care of the wound. An evaluative measure would be the process of observing the patient. Reassessment is a behavior performed by the nurse. The type of wound cleanser and dressings would be a standard of care.

An appropriate nursing diagnosis for a client with large areas of skin excoriation resulting from scratching an allergic rash is: 1. Risk for Impaired Skin Integrity 2. Impaired Skin Integrity 3. Impaired Tissue Integrity 4. Risk for Infection

2. Impaired Skin Integrity; The client has an actual impairment of the skin due to the rash and the scratching so is no longer "at risk". Because the damage is at the skin level, it is not impaired tissue integrity (option 3) since that would involve deeper tissues. Surface excoriation is also not prone to becoming infected.

A nurse assesses a 78yearold patient who weighs 240 pounds (108.9 kg) and is partially immobilized because of a stroke. The nurse turns the patient and finds that the skin over the sacrum is very red and the patient does not feel sensation in the area. The patient has had fecal incontinence on and off for the last 2 days. The nurse identifies the nursing diagnosis of risk for impaired skin integrity. Which of the following goals are appropriate for the patient? (Select all that apply.) 1. Patient will be turned every 2 hours within 24 hours. 2. Patient will have normal bowel function within 72 hours. 3. Patient's skin will remain intact through discharge. 4. Patient's skin condition will improve by discharge.

2. Patient will have normal bowel function within 72 hours. 3. Patient's skin will remain intact through discharge.

Which of the following outcome statements for the goal, "Patient will achieve a gain of 10 lbs (4.5 kg) in body weight in a month" are worded incorrectly? (Select all that apply.) 1. Patient will eat at least three fourths of each meal by 1 week. 2. Patient will verbalize relief of nausea and have no episodes of vomiting in 1 week. 3. Patient will eat foods with highcalorie content by 1 week. 4. Give patient liquid supplements 3 times a day.

2. Patient will verbalize relief of nausea and have no episodes of vomiting in 1 week. 4. Give patient liquid supplements 3 times a day.

A nurse caring for a patient with pneumonia sits the patient up in bed and suctions the patient's airway. After suctioning, the patient describes some discomfort in his abdomen. The nurse auscultates the patient's lung sounds and gives him a glass of water. Which of the following would be appropriate evaluative criteria used by the nurse? (Select all that apply.) 1. Patient drinks contents of water glass. 2. Patient's lungs are clear to auscultation in bases. 3. Patient reports abdominal pain on scale of 0 to 10. 4. Patient's rate and depth of breathing are normal with head of bed elevated

2. Patient's lungs are clear to auscultation in bases. 4. Patient's rate and depth of breathing are normal with head of bed elevated. The criteria of clear lung sounds and rate and depth of breathing are evaluative criteria for determining if the patient's airway is clear. Drinking the contents of the water glass is a completed intervention. The patient's report of pain is assessment data

7. In which of the following examples is a nurse applying critical thinking attitudes when preparing to insert an intravenous (IV) catheter? (Select all that apply.) 1. Following the procedural guideline for IV insertion 2. Seeking necessary knowledge about the steps of the procedure from a more experienced nurse 3. Showing confidence in performing the correct IV insertion technique 4. Being sure that the IV dressing covers the IV site completely

2. Seeking necessary knowledge about the steps of the procedure from a more experienced nurse Seeking necessary knowledge about the steps of the procedure shows humility. The nurse recognizes that she needs clarification from a senior colleague. Another example of a critical thinking attitude is confidence. In this case confidently inserting an IV line allows the nurse to convey expertise and a sense of calm, leading the patient to trust the nurse. Following policy and procedure is an example of following standards of care, not of a critical thinking attitude. Making sure that the dressing is covered is a step in following good standards of IV care but is not a critical thinking attitude.

Which of the following actions would place a client at the greatest risk for a shearing force injury to the skin? 1. Walking without shoes 2. Sitting in Fowler's position 3. Lying supine in bed 4. Using a heating pad

2. Sitting in Fowler's position; None of the other movements or situations creates the combination of friction and pressure with downward movement seen in bedridden clients positioned in Fowler's position.

You are caring for an assigned client and notice a superficial ulcer on the client's buttock that appears as a shallow crater involving the epidermis and the dermis. Which of the following stages would you say best describes this break in skin integrity? 1. Stage I 2. Stage II 3. Stage III 4. Stage IV

2. Stage II; Stage I pressure ulcer involves a nonblanchable erythema of intact skin, while a stage II involves a partial-thickness skin loss involving epidermis, dermis, or both, with the ulcer being superficial and presenting as an abrasion, blister, or shallow crater.

What is an indication of proper use of a triangle arm sling? 1. The elbow is kept flexed at 90 degrees or more. 2. The knot is placed on either side of the vertebrae of the neck. 3. The sling extends to just proximal of the hand. 4. The sling is removed q2h to assess for circulation and skin integrity.

2. The knot is placed on either side of the vertebrae of the neck; The knot of the triangle sling must be kept off the spinal processes because this would be uncomfortable and put unnecessary pressure on the vertebrae. The elbow should be flexed slightly less than 80 degrees (not > 90 as in option 1) so the hand is above the elbow to prevent dependent swelling. The sling must extend past the wrist in order to support the hand. Although the sling must be removed to check for circulation and skin integrity, every 2 hours (option 4) is unnecessarily frequent and impractical.

A patient comes to a medical clinic with the diagnosis of asthma. The nurse practitioner decides that the patient's obesity adds to the difficulty of breathing; the patient is 5 feet 7 inches tall and weighs 200 pounds (90.7 kg). Based on the nursing diagnosis of imbalanced nutrition: more than body requirements, the practitioner plans to place the patient on a therapeutic diet. Which of the following are evaluative measures for determining if the patient achieves the goal of a desired weight loss? (Select all that apply.) 1. The patient eats 2000 calories a day. 2. The patient is weighed during each clinic visit. 3. The patient discusses factors that increase the risk of an asthma attack. 4. The patient's food diary that tracks intake of daily meals is reviewed.

2. The patient is weighed during each clinic visit. 4. The patient's food diary that tracks intake of daily meals is reviewed. Weighing the patient during each clinic visit and reviewing a food diary indicate whether weight loss is occurring and if the patient is eating the proper foods designed to reduce his or her weight.

You are caring for a patient after surgery who underwent a liver resection. His prothrombin time (PT) or an activated partial thromboplastin time (APTT) is greater than normal. He has low blood pressure; tachycardia; thready pulse; and cool, clammy, pale skin, and he is restless. You assess his surgical wound, and the dressing is saturated with blood. Which immediate interventions should you perform? (Select all that apply.) A. Notify the surgeon. B. Maintain intravenous (IV) fluid infusion and prepare to give volume replacement. C. Monitor the patient's vital signs every 15 minutes or more frequently until his condition stabilizes. D. Wean oxygen therapy. E. Provide comfort through bathing.

A, B, C

A nurse assesses a patient who comes to the pulmonary clinic. "I see that it's been over 6 months since you've been in, but your appointment was for every 2 months. Tell me about that. Also I see from your last visit that the doctor recommended routine exercise. Can you tell me how successful you have been following his plan?" The nurse's assessment covers which of Gordon's functional health patterns? 1. Value-belief pattern 2. Cognitive-perceptual pattern 3. Coping-stress-tolerance pattern 4. Health perception-health management pattern

4. Health perception-health management pattern

A client has a diabetic stasis ulcer on the lower leg. The nurse uses a hydrocolloid dressing to cover it. The procedure for application includes: 1. Cleaning the skin and wound with betadine 2. Removing all traces of residues for the old dressing 3. Choosing a dressing no more than quarter-inch larger than the wound size 4. Holding in place for one minute to allow it to adhere

4. Holding in place for one minute to allow it to adhere; The skin is cleansed with normal saline or a mild cleanser. Residue of old dressings will dissolve. The dressing size is to be 3-4 cm (1.5 inches) larger than the size of the wound.

A nurse is reviewing a patient's list of nursing diagnoses in the medical record. The most recent nursing diagnosis is diarrhea related to intestinal colitis. This is an incorrectly stated diagnostic statement, best described as: 1. Identifying the clinical sign instead of an etiology. 2. Identifying a diagnosis based on prejudicial judgment. 3. Identifying the diagnostic study rather than a problem caused by the diagnostic study. 4. Identifying the medical diagnosis instead of the patient's response to the diagnosis.

4. Identifying the medical diagnosis instead of the patient's response to the diagnosis.

Why is a client with fever often predisposed to pressure ulcers? 1. Pain perception is diminished. 2. Medications given to relieve fever cause edema. 3. The client may be too weak to change position. 4. Increased metabolism causes increased oxygen needs that cannot be met.

4. Increased metabolism causes increased oxygen needs that cannot be met; Increased metabolism causes increased oxygen needs that cannot be met; therefore, a client with a fever is predisposed to pressure ulcers. Answers 1 and 2 are false statements. Answer 3 may be a cause of pressure ulcers and may occur in clients with fever, but it is not directly related.

Setting a time frame for outcomes of care serves which of the following purposes? 1. Indicates which outcome has priority 2. Indicates the time it takes to complete an intervention 3. Indicates how long a nurse is scheduled to care for a patient 4. Indicates when the patient is expected to respond in the desired manner

4. Indicates when the patient is expected to respond in the desired manner

The evaluation process includes interpretation of findings as one of its five elements. Which of the following is an example of interpretation? 1. Evaluating the patient's response to selected nursing interventions 2. Selecting an observable or measurable state or behavior that reflects goal achievement 3. Reviewing the patient's nursing diagnoses and establishing goals and outcome statements 4. Matching the results of evaluative measures with expected outcomes to determine patient's status

4. Matching the results of evaluative measures with expected outcomes to determine patient's status When interpreting findings, you compare the patient's behavioral responses and physiological signs and symptoms that you expect to see with those actually seen from your evaluation.

A patient has limited mobility as a result of a recent knee replacement. The nurse identifies that he has altered balance and assists him in ambulation. The patient uses a walker presently as part of his therapy. The nurse notes how far the patient is able to walk and then assists him back to his room. Which of the following is an evaluative measure? 1. Uses walker during ambulation 2. Presence of altered balance 3. Limited mobility in lower extremities 4. Observation of distance patient is able to walk

4. Observation of distance patient is able to walk An evaluative measure determines a patient's response to therapy, in this case how well the patient is able to ambulate (distance walked).

The patient receiving total parenteral nutrition (TPN) asks the nurse why his blood glucose is being checked since he does not have diabetes. What is the best response by the nurse? A. TPN can cause hyperglycemia, and it is important to keep your blood glucose level in an acceptable range. B. The high concentration of dextrose in the TPN can give you diabetes; thus you need to be monitored closely. C. Monitoring your blood glucose level helps to determine the dose of insulin that you need to absorb the TPN. D. Checking your blood glucose level regularly helps to determine if the TPN is effective as a nutrition intervention.

A

Frontal or ventral surface face up: A. Pronation B. Supination C. Flexion D. Extension E. Hyperextension

B

Indicated by a diminished or unequal carotid pulsation: A. Bruit B. Atherosclerosis C. Occlusion D. Syncope

B

The home care nurse is seeing the following patients. Which patient is at greatest risk for experiencing inadequate nutrition? A. A 55-year-old obese man recently diagnosed with diabetes mellitus B. A recently widowed 76-year-old woman recovering from a mild stroke C. A 22-year-old mother with a 3-year-old toddler who had tonsillectomy surgery D. A 46-year-old man recovering at home following coronary artery bypass surgery

B

The nurse teaches a patient who has had surgery to increase which nutrient to help with tissue repair? A. Fat B. Protein C. Vitamin D. Carbohydrate

B

While assessing the adult patient's lungs, the nurse identifies the following assessment findings. Which finding should be reported to the health care provider? A. Respiratory rate: 14 B. Pain reported when palpating posterior lower thorax C. Thorax rising and falling symmetrically for right and left lungs D. Vesicular breath sounds heard with auscultation of peripheral lung fields

B

What is the best method to sterilize a straight urinary catheter and suction tube in the home setting? A) Use an autoclave. B) Use boiling water. C) Use ethylene oxide gas. D) Use chemicals for disinfection.

B. Use boiling water.

A nurse educator is conducting a parenting class for new parents. Which of the following statements made by a participant indicates a need for further clarification and instruction? A. "I will begin swimming lessons as soon as my baby can close her mouth under water." B. "Once my baby can sit up, he should be safe in the bathtub." C. "I will test the temperature of the water before placing my baby in the bath." D. "Once my infant starts to push up, I will remove the mobile from over the bed."

B. "Once my baby can sit up, he should be safe in the bathtub."

A home health nurse is discussing the dangers of food poisoning with a client. Which of the following information should the nurse including in her counseling? (Select all that apply.) A. Most food poisoning is caused by a virus. B. Immunocompromised individuals are at risk for complications from food poisoning C. Clients who are especially at risk are instructed to eat or drink only pasteurized milk, yogurt, cheese, or other dairy products. D. Healthy individuals usually recover from the illness in a few weeks. E. Handling raw and fresh food separately to avoid cross contamination may prevent food poisoning.

B. Immunocompromised individuals are at risk for complications from food poisoning C. Clients who are especially at risk are instructed to eat or drink only pasteurized milk, yogurt, cheese, or other dairy products. E. Handling raw and fresh food separately to avoid cross contamination may prevent food poisoning.

After seeing a patient, the physician gives a nursing student a verbal order for a new medication. The nursing student first needs to: A. Follow ISMP guidelines for safe medication abbreviations. B. Explain to the physician that the order needs to be given to a registered nurse. C. Write down the order on the patient's order sheet and read it back to the physician. D. Ensure that the six rights of medication administration are followed when giving the medication.

B. Rationale: Nursing students cannot take orders.

A nursing student takes a patient's antibiotic to his room. The patient asks the nursing student what it is and why he should take it. Which information does the nursing student include when replying to the patient? A. Only the patient's physician can give this information. B. The student provides the name of the medication and a description of its desired effect. C. Information about medications is confidential and cannot be shared. D. He has to speak with his assigned nurse about this.

B. Rationale: Patients need to know information about their medications so they can take them correctly and safely.

If a patient who is receiving intravenous (IV) fluids develops tenderness, warmth, erythema, and pain at the site, the nurse suspects: A. Sepsis. B. Phlebitis. C. Infiltration. D. Fluid overload.

B. Rationale: Redness, warmth, and tenderness at the IV site are signs of phlebitis.

The nurse receives an order to start giving a loop diuretic to a patient to help lower his or her blood pressure. The nurse determines the appropriate route for administering the diuretic according to: A. Hospital policy. B. The prescriber's orders. C. The type of medication ordered. D. The patient's size and muscle mass.

B. Rationale:The order from the prescriber needs to indicate the route of administration.

The client is a chronic carrier of infection. To prevent the spread of the infection to other clients or health care providers, the nurse emphasizes interventions that do which of the following? Eliminate the reservoir. Block the portal of exit from the reservoir. Block the portal of entry into the host. Decrease the susceptibility of the host.

Block the portal of exit from the reservoir. Rationale: Blocking the movement of the organism from the reservoir will succeed in preventing the infection of any other persons. Since the carrier person is the reservoir and the condition is chronic, it is not possible to eliminate the reservoir (option 1). Blocking the entry into a host (option 3) or decreasing the susceptibility of the host (option 4) will be effective for only that one single individual and, thus, is not as effective as blocking exit from the reservoir. Cognitive Level: Understanding. Client Need: Safe, Effective Care Environment. Nursing Process: Planning.

Abnormal neurological condition in which language function is defective or absent; related to injury to speech center in cerebral cortex, causing receptive or expressive aphasia. A. Adventitious sounds B. Alopecia C. Aphasia D. Apical impulse or point of maximal impulse E. Arcus Senilis

C

While preparing to do a sterile dressing change, a nurse accidentally sneezes over the sterile field that is on the over-the-bed table. Which of the following principles of surgical asepsis, if any, has the nurse violated? A) When a sterile field comes in contact with a wet surface, the sterile field is contaminated by capillary action. B) Fluid flows in the direction of gravity. C) A sterile field becomes contaminated by prolonged exposure to air. D) None of the principles were violated.

C. A sterile field becomes contaminated by prolonged exposure to air.

A patient is isolated for pulmonary tuberculosis. The nurse notes that the patient seems to be angry, but he knows that this is a normal response to isolation. Which is the best intervention? A) Provide a dark, quiet room to calm the patient. B) Reduce the level of precautions to keep the patient from becoming angry. C) Explain the reasons for isolation procedures and provide meaningful stimulation. D) Limit family and other caregiver visits to reduce the risk of spreading the infection.

C. Explain the reasons for isolation procedures and provide meaningful stimulation.

A patient has an indwelling urinary catheter. Why does an indwelling urinary catheter present a risk for urinary tract infection? A) It keeps an incontinent patient's skin dry. B) It can get caught in the linens or equipment. C) It obstructs the normal flushing action of urine flow. D) It allows the patient to remain hydrated without having to urinate.

C. It obstructs the normal flushing action of urine flow.

The nurse has redressed a patient's wound and now plans to administer a medication to the patient. Which is the correct infection control procedure? A) Leave the gloves on to administer the medication. B) Remove gloves and administer the medication. C) Remove gloves and perform hand hygiene before administering the medication. D) Leave the medication on the bedside table to avoid having to remove gloves before leaving the patient's room.

C. Remove gloves and perform hand hygiene before administering the medication.

A patient is transitioning from the hospital to the home environment. A home care referral is obtained. What is a priority in relation to safe medication administration for the discharge nurse? A. Set up the follow-up appointments with the physician for the patient. B. Ensure that someone will provide housekeeping for the patient at home. C. Ensure that the home care agency is aware of medication and health teaching needs. D. Make sure that the patient's family knows how to safely bathe him or her and provide mouth care.

C. Rationale: A nursing responsibility is to collaborate with community resources when patients have home care needs or difficulty understanding their medications.

A nurse is administering medications to a 4-year-old patient. After he or she explains which medications are being given, the mother states, "I don't remember my child having that medication before." What is the nurse's next action? A. Give the medications B. Identify the patient using two patient identifiers C. Withhold the medications and verify the medication orders D. Provide medication education to the mother to help her better understand her child's medications

C. Rationale: Do not ignore patient or caregiver concerns; always verify orders whenever a medication is questioned before administering it.

A patient is receiving an intravenous (IV) push medication. If the drug infiltrates into the outer tissues, the nurse: A. Continues to let the IV run. B. Applies a warm compress to the infiltrated site. C. Stops the administration of the medication and follows agency policy. D. Should not worry about this because vesicant filtration is not a problem.

C. Rationale: When an IV medication infiltrates, stop giving the medication and follow agency policy.

A home health nurse is discussing the dangers of carbon monoxide poisoning with a client. Which of the following information should the nurse include in her counseling? A. Carbon monoxide has a distinct odor. B. Water heaters should be inspected every 5 years. C. The lungs are damaged from carbon monoxide inhalation. D. Carbon monoxide binds with hemoglobin in the body.

C. The lungs are damaged from carbon monoxide inhalation.

A mother and her 3-year-old live in a home built in 1932. Which NANDA nursing diagnosis is most applicable for this child? Risk for Suffocation Risk for Injury Risk for Poisoning Risk for Disuse Syndrome

Risk for Poisoning Rationale: A home that was built prior to 1978 has lead-based paint. The ingestion of lead-based paint chips places that child at risk for elevated serum lead levels and neurologic deficits. The most appropriate nursing diagnosis for this child is Risk for Poisoning. Option 1: The risk for suffocation is greater in infants and is not related to a home with lead-based paint. Options 2 and 4 are not related to lead-based paint. Cognitive Level: Applying. Client Need: Safe, Effective Care Environment. Nursing Process: Nursing Diagnosis.

A 75-year-old client, hospitalized with a cerebral vascular accident (stroke), becomes disoriented at times and tries to get out of bed, but is unable to ambulate without help. What is the most appropriate safety measure? Restrain the client in bed. Ask a family member to stay with the client. Check the client every 15 minutes. Use a bed exit safety monitoring device

Use a bed exit safety monitoring device. Rationale: Option 4 is an intervention that can allow the client to feel independent and also alert the nursing and nursing staff when the client needs assistance. It is the most realistic answer that promotes client safety. Option 1 can increase agitation and confusion and removes the client's independence. Option 2 would help but transfers the responsibility to the family member. Option 3 is inappropriate since the client could fall during the unobserved interval and it is not a realistic answer for the nurse. Cognitive Level: Analyzing. Client Need: Safe, Effective Care Environment. Nursing Process: Implementation

The nurse needs to collect a sputum specimen to identify the presence of tuberculosis (TB). Which nursing action(s) is/are indicated for this type of specimen? Select all that apply. Collect the specimen in the evening. Send the specimen immediately to the laboratory. Ask the client to spit into the sputum container. Offer mouth care before and after collection of the sputum specimen. Collect a specimen for 3 consecutive days.

Send the specimen immediately to the laboratory. Offer mouth care before and after collection of the sputum specimen. Collect a specimen for 3 consecutive days.

A patient with a cardiac history is taking the diuretic furosemide (Lasix) and is seen in the emergency department for muscle weakness. Which laboratory value do you assess first?

Serum Potassium

A client is to have a thoracentesis in order to aspirate pleural fluid for biopsy. In order to prepare the client for the procedure, the nurse best positions the client in which manner? Lying in a lateral position with the affected lung down and back, curved into a fetal position. The head is supported with a pillow. The arms are positioned comfortably away from the chest wall. Lying in a 10-degree reverse Trendelenburg position with the arms over the head. Small pillows allowed under the head and arms. Sitting in a Fowler's position with the arms abducted and supported by pillows placed on each side of the body. The head is lying flat against the mattress. Sitting on the side of the bed, leaning over a bedside table with a pillow on it, arms overhead supported by the pillow

Sitting on the side of the bed, leaning over a bedside table with a pillow on it, arms overhead supported by the pillow

The nurse is having difficulty obtaining a capillary blood sample from a client's finger to measure blood glucose using a blood glucose monitor. Which procedure will increase the blood flow to the area to ensure an adequate specimen? Raise the hand on a pillow to increase venous flow. Pierce the skin with the lancet in the middle of the finger pad. Wrap the finger in a warm cloth for 30--60 seconds. Pierce the skin at a 45-degree angle.

Wrap the finger in a warm cloth for 30--60 seconds.

It would be appropriate to delegate the taking of vital signs of which client to unlicensed assistive personnel? A client being prepared for elective facial surgery with a history of stable hypertension. A client receiving a blood transfusion with a history of transfusion reactions. A client recently started on a new antiarrhythmic agent. A client who is admitted frequently with asthma attacks.

Your Answer: A client being prepared for elective facial surgery with a history of stable hypertension. Rationale: Vital signs measurement may be delegated to UAP if the client is in stable condition, the findings are expected to be predictable, and the technique requires no modification. Only the preoperative client meets these requirements. In addition, UAP are not delegated to take apical pulse measurements for the client with an irregular pulse as would be the case with the client newly started on antiarrhythmic medication (option 3). Cognitive Level: Applying. Client Need: Health Promotion and Maintenance. Nursing Process: Planning.

The nurse reports that the client has dyspnea when ambulating. The nurse is most likely to have assessed which of the following? Shallow respirations Wheezing Shortness of breath Coughing up blood

Your Answer: Shortness of breath Rationale: Dyspnea, difficult or labored breathing, is commonly related to inadequate oxygenation. Therefore, the client is likely to experience shortness of breath, that is, a sense that none of the breaths provide enough oxygen and an immediate second breath is needed. Option 1: Shallow respirations are seen in tachypnea (rapid breathing). Option 2: Wheezing is a high-pitched breathing sound that may or may not occur with dyspnea. Option 4: The medical term for coughing up blood is hemoptysis and is unrelated to dyspnea. Cognitive Level: Applying. Client Need: Health Promotion and Maintenance. Nursing Process: Evaluation.

When the nurse enters the room to measure vital signs in preparing the client for a diagnostic test, the client is on the phone. What technique should the nurse use to determine the respiratory rate? Count the respirations during conversational pauses. Ask the client to end the phone call now and resume it at a later time. Wait at the client's bedside until the phone call is completed and then count respirations. Since there is no evidence of distress or urgency, defer the measurement.

Your Answer: Since there is no evidence of distress or urgency, defer the measurement. Rationale: Since the client's needs are always considered first, the measurement should be delayed unless the client is in distress or there are other urgent reasons. Option 1: Respirations should be measured for 30 seconds to 1 minute and are affected by talking. Option 2: There needs to be an important reason for interrupting the client. Option 3: It is inappropriate to wait and listen to the client's conversation. Cognitive Level: Understanding. Client Need: Health Promotion and Maintenance. Nursing Process: Planning.

An 85-year-old client has had a stroke resulting in right-sided facial drooping, difficulty swallowing, and the inability to move self or maintain position unaided. The nurse determines that which sites are appropriate for taking the temperature? (Select all that apply.) Oral Rectal Axillary Tympanic Temporal artery

Your Answers: Axillary Tympanic Temporal artery Rationale: For this client, the nurse could take an axillary, tympanic, or temporal artery temperature. Due to the facial drooping and difficulty swallowing, the oral route is not recommended (option 1). Although the rectal route could be used, it would require unnecessary moving and positioning of a client who cannot assist, and it would not provide a significant advantage over the other routes (option 2). Cognitive Level: Applying. Client Need: Health Promotion and Maintenance. Nursing Process: Assessment.

An older client with renal insufficiency is to receive a cardiac medication. Which is the nurse most likely to administer? 1) A decreased dosage 2) The standard dosage 3) An increased dosage 4) Divided dosages

1) A decreased dosage Rationale: Due to renal insufficiency, the dose of the medication would need to be decreased in order to avoid accumulation of the medication and the risk of toxicity.

When using ice massage for pain relief, which of the following are correct? (Select all that apply.) 1) Apply ice using firm pressure over skin. 2) Apply ice until numbness occurs and remove the ice for 5 to 10 minutes. 3) Apply ice until numbness occurs and discontinue application. 4) Apply ice for no longer than 10 minutes.

1) Apply ice using firm pressure over skin. 2) Apply ice until numbness occurs and remove the ice for 5 to 10 minutes.

What is the removal of devitalized tissue from a wound called? a) Debridement b) Pressure reduction c) Negative pressure wound therapy d) Sanitization

a) Debridement Debridement is the removal of nonliving tissue, cleaning the wound to move toward healing.

A 56-year-old patient with diabetes admitted for community acquired pneumonia has a temperature of 38.2°C (100.8°F) via the temporal artery. Which additional assessment data are needed in planning interventions for the patient's infection? (Select all that apply.) a) Heart rate b) Presence of diaphoresis c) Smoking history d) Respiratory rate e) Recent bowel movement f) Blood pressure in right arm g) Patient's normal temperature h) Blood pressure in distal extremity

a) Heart rate d) Respiratory rate g) Patient's normal temperature You need to determine the patient's usual temperature to evaluate the degree of temperature elevation. Heart rate and respiratory rate increase with temperature. The presence of diaphoresis may contribute to fluid volume deficit from hyperthermia

Which of the following patients is at most risk for tachypnea? (Select all that apply.) a) Patient just admitted with four rib fractures b) Woman who is 9 months' pregnant c) Adult who has consumed alcoholic beverages d) Adolescent awaking from sleep

a) Patient just admitted with four rib fractures b) Woman who is 9 months' pregnant Rib fractures would cause splinting and pain to increase respiratory rate. Pregnancy impedes diaphragmatic excursion, causing shallow, frequent breaths

Which type of pressure ulcer is noted to have intact skin and may include changes in one or more of the following: skin temperature (warmth or coolness), tissue consistency (firm or soft), and/or pain? a) Stage I b) Stage II c) Stage III d) Stage IV

a) Stage A stage I pressure ulcer does not have a break in the skin but has a redness that does not blanch. Depending on the skin color, there may be a discoloration; the area may feel warm because of the vasodilation or cool if blood is constricted in the area; and the tissue may feel firm if there is edema in the area or soft if the blood flow is compromised. The patient may report pain in the area.

Which one of the following instructions is crucial for the nurse to give to both family members and the patient who is about to be started on a patient-controlled analgesia (PCA) of morphine? 1) Only the patient should push the button. 2) Do not use the PCA until the pain is severe. 3) The PCA prevents overdoses from occurring. 4) Notify the nurse when the button is pushed.

1) Only the patient should push the button.

Which of the following is likely to occur if the goal statement is poorly written? 1) There is no standard against which to compare outcomes. 2) The nursing diagnoses cannot be prioritized. 3) Only dependent nursing interventions can be used. 4) It is difficult to determine which nursing interventions can be delegated.

1) There is no standard against which to compare outcomes. Rationale: Goal statements provide the standard against which outcomes are measured. Nursing diagnoses are prioritized before goals are written (option 2). Both independent and dependent interventions may be appropriate for any goal (option 3). Clarity of the goal does not influence delegation of the intervention (option 4).

While caring for a patient with cancer pain, the nurse knows that the World Health Organization (WHO) analgesic ladder recommends: 1) Transitioning use of adjuvants with nonsteroidal antiinflammatory drugs (NSAIDs) to opioids. 2) Using acetaminophen for refractory pain. 3) Limiting the use of opioids because of the likelihood of side effects. 4) Avoiding total sedation, regardless of how severe the pain is.

1) Transitioning use of adjuvants with nonsteroidal antiinflammatory drugs (NSAIDs) to opioids.

The nurse determines that a field remains sterile if which of the following conditions exist? Tips of wet forceps are held upward when held in ungloved hands. The field was set up 1 hour before the procedure. Sterile items are 2 inches from the edge of the field. The nurse reaches over the field rather than around the edges.

Sterile items are 2 inches from the edge of the field. Rationale: All items within 1 inch of the edge of the sterile field are considered contaminated because the edge of the field is in contact with unsterile areas. When hands are ungloved, forceps tips are to be held downward to prevent fluid from becoming contaminated by the hands and then returned to the sterile field (option 1). Fields should be established immediately before use to prevent accidental contamination when not observed closely (option 2). Reaching over a sterile field increases the chances of dropping an unsterile item onto or touching the sterile field (option 4). Cognitive Level: Applying. Client Need: Safe, Effective Care Environment. Nursing Process: Evaluation.

While receiving a blood transfusion, your patient develops chills, tachycardia, and flushing. What is your priority action?

Stop the Transfusion

Which nursing intervention is the highest in priority for a client at risk for falls in a hospital setting? Keep all of the side rails up. Review prescribed medications. Complete the "get up and go" test. Place the bed in the lowest position.

Place the bed in the lowest position. Rationale: Placing the bed in the lowest position results in a client falling the shortest distance. The client is least likely to fall when getting out of bed is at an appropriate height. Option 1 can cause a fall with injury because the client may fall from a higher distance when trying to get over the rail. Option 2 is important to do as certain medications can increase the risk of falling. However this is not the best answer because it is N/A to all clients. Option 3 would help the nurse to assess a client's risk for falling but would not prevent injury. Cognitive Level: Analyzing. Client Need: Safe, Effective Care Environment. Nursing Process: Implementation.

A 78-year-old male client needs to complete a 24-hour urine specimen. In planning his care, the nurse realizes that which measure is most important? Instruct the client to empty his bladder and save this voiding to start the collection. Instruct the client to use sterile individual containers to collect the urine. Post a sign stating "Save All Urine" in the bathroom. Keep the urine specimen in the refrigerator.

Post a sign stating "Save All Urine" in the bathroom. Option 3 is the most important nursing measure. This will inform the staff that the client is on a 24-hour urine collection. Option 1 is not appropriate since the first voided specimen is to be discarded. Option 2 is not an appropriate nursing measure since the specimen container is clean not sterile, and one container is needed—not individual containers. Option 4 is inappropriate because some 24-hour urine collections do not require refrigeration.

A nurse sees smoke emerging from the suction equipment being used. Which is the greatest priority in the event of a fire? Report the fire. Extinguish the fire. Protect the clients. Contain the fire.

Protect the clients. Rationale: In the event of a fire, the nurse's priority responsibility is to rescue or protect the clients under his or her care. The next priorities are to report or alert the fire department, contain or confine, and extinguish the fire. Cognitive Level: Understanding. Client Need: Safe, Effective Care Environment. Nursing Process: Implementation.

Your patient who has diabetic ketoacidosis is breathing rapidly and deeply. Intravenous (IV) fluids and other treatments have just been started. What should you do about this patient's breathing?

Provide Frequent Oral Care to Keep Her Mucous Membranes Moist

The client is supposed to have a fecal occult blood test done on a stool sample. The nurse is going to use the Hemoccult test. Which of the following indicates that the nurse is using the correct procedure? Select all that apply. Mixes the reagent with the stool sample before applying to the card. Collects a sample from two different areas of the stool specimen. Assesses for a blue color change. Asks a colleague to verify the pink color results. Asks the client if he has taken vitamin C in the past few days.

Collects a sample from two different areas of the stool specimen. Assesses for a blue color change. Asks the client if he has taken vitamin C in the past few days. Rationale: The nurse should obtain the stool specimen from two different areas of the stool. The nurse should observe for a blue color change, which is indicative of a positive result. The nurse should assess for the ingestion of vitamin C by the client because it is contraindicated for 3 days prior to taking the specimen. Option 1 is incorrect since the reagent is placed on the specimen after it is applied to the testing card. Option 4 is incorrect because a pink color would be considered negative and does not require verification.

The client's temperature at 8:00 AM using an oral electronic thermometer is 36.1°C (97.2°F). If the respiration, pulse, and blood pressure were within normal range, what would the nurse do next? Wait 15 minutes and retake it. Check what the client's temperature was the last time it was taken. Retake it using a different thermometer. Chart the temperature; it is normal.

Correct Answer: Check what the client's temperature was the last time it was taken. Rationale: Although the temperature is slightly lower than expected for the morning, it would be best to determine the client's previous temperature range next. This may be a normal range for this client. Depending on that finding, the nurse might want to retake it in a few minutes—no need to wait 15 minutes (option 3) or with another thermometer to see if the initial thermometer was functioning properly. Chart after determining that the temperature has been measured properly (option 4). Cognitive Level: Applying. Client Need: Health Maintenance and Promotion. Nursing Process: Assessment.

Based on knowledge of peptic ulcer disease (PUD), the nurse anticipates the presence of which bacteria when reviewing the laboratory data for a patient suspected of having PUD? A. Micrococcus B. Staphylococcus C. Corynebacterium D. Helicobacter pylori

D

Blowing or whooshing sounds created by changes in blood flow through the heart or abnormalities in the valve closure. A. Kyphosis B. Lordosis C. Malignancy D. Murmurs

D

The second heart sound (S2) occurs when: A. Systole begins B. There is rapid ventricular filling C. The mitral and tricuspid valves close D. The aortic and pulmonic valves close

D

The nurse wears a gown when: A) The patient's hygiene is poor. B) The nurse is assisting with medication administration. C) The patient has acquired immunodeficiency syndrome (AIDS) or hepatitis. D) Blood or body fluids may get on the nurse's clothing from a task that he or she plans to perform.

D. Blood or body fluids may get on the nurse's clothing from a task that he or she plans to perform.

Which is the most likely means of transmitting infection between patients? A) Exposure to another patient's cough B) Sharing equipment among patients C) Disposing of soiled linen in a shared linen bag D) Contact with a health care worker's hands

D. Contact with a health care worker's hands

A family member is providing care to a loved one who has an infected leg wound. What would you instruct the family member to do after providing care and handling contaminated equipment or organic material? A) Wear gloves before eating or handling food. B) Place any soiled materials into a bag and double bag it. C) Have the family member check with the doctor about need for immunization. D) Perform hand hygiene after care and/or handling contaminated equipment or material.

D. Perform hand hygiene after care and/or handling contaminated equipment or material.

Who should you verify medications calculations with and why? A. The Nurse Assistant, to ensure accuracy. B. The pharmacy, because only the pharmacist knows accurate calculations. C. The patient, because they are a math teacher. D. Another Nurse, to ensure accuracy.

D. Another nurse, to ensure accuracy.

A child in the hospital starts to have a grand mal seizure while playing in the playroom. What is your most important nursing intervention during this situation? A. Begin cardiopulmonary respiration. B. Restrain the child to prevent injury. C. Place a tongue blade over the tongue to prevent aspiration. D. Clear the area around the child to protect the child from injury.

D. Clear the area around the child to protect the child from injury.

The patient has an order for 2 tablespoons of Milk of Magnesia. How much medication does the nurse give him or her? A. 2 mL B. 5 mL C. 16 mL D. 30 mL

D. Rationale: 1 tablespoon = 15 mL; 2 tablespoons = 30 mL.

A nurse is administering eardrops to an 8-year-old patient with an ear infection. How does the nurse pull the patient's ear when administering the medication? A. Outward B. Back C. Upward and back D. Upward and outward

D. Rationale: Eardrops are administered with the ear positioned upward and outward for patients greater than 3 years of age.

A nurse accidently gives a patient a medication at the wrong time. The nurse's first priority is to: A. Complete an occurrence report. B. Notify the health care provider. C. Inform the charge nurse of the error. D. Assess the patient for adverse effects.

D. Rationale: Patient safety and assessing the patient are priorities when a medication error occurs.

Which of the following defining characteristics is consistent with fluid volume deficit?

Dry Mucous Membranes, Thready Pulse, Tachycardia

A client has a wound infection. What local human response should the nurse expect to identify?

Edema; Chemical mediators increase the permeability of small blood vessels, thereby causing fluid to move into the interstitial compartment, resulting in local edema.

You teach patients to replace sweat, vomiting, or diarrhea fluid losses with which type of fluid?

Fluid that has Sodium (salt) in it

In caring for a client on contact precautions for a draining infected foot ulcer, which action should the nurse perform? Wear a mask during dressing changes. Provide disposable meal trays and silverware. Follow standard precautions in all interactions with the client. Use surgical aseptic technique for all direct contact with the client.

Follow standard precautions in all interactions with the client. Rationale: Standard precautions include all aspects of contact precautions with the exception of placing the client in a private room. A mask is indicated when working over a sterile wound rather than an infected one (option 1). Disposable food trays are not necessary for clients with infected wounds unlikely to contaminate the client's hands (option 2). Sterile technique (surgical asepsis) is not indicated for all contact with the client (option 4). The nurse would utilize clean technique when dressing the wound to prevent introduction of additional microbes. Cognitive Level: Applying. Client Need: Safe, Effective Care Environment. Nursing Process: Implementation.

The nurse practitioner requests a laboratory blood test to determine how well a client has controlled her diabetes during the past 3 months. Which blood test will provide this information? Fasting blood glucose Capillary blood specimen Glycosylated hemoglobin GGT (gamma-glutamyl transferase)

Glycosylated hemoglobin A glycosylated hemoglobin will indicate the glucose levels for a period of time, which is indicated by the nurse practitioner. Options 1 and 2 will provide information about the current blood glucose not the past history. Option 4 is used to assess for liver disease.

When planning to teach health care topics to a group of male adolescents, which topic should the nurse consider a priority? Sports contribute to an adolescent's self-esteem. Sunbathing and tanning beds can be dangerous. Guns are the most frequently used weapon for adolescent suicide. A driver's education course is mandatory for safety.

Guns are the most frequently used weapon for adolescent suicide. Rationale: Suicide and homicide are two leading causes of death among teenagers. Adolescent males commit suicide at a higher rate than adolescent females. Options 1 and 2 are true; however, neither would be as high a priority as preventing suicide. Option 4 is not true. A driver's education course does not ensure safe practice. Cognitive Level: Analysis. Client Need: Safe, Effective Care Environment. Nursing Process: Planning.

While preparing the client for a colonoscopy, the nurse's responsibilities include: Explaining the risks and benefits of the exam Instructing the client about the bowel preparation prior to the test Instructing the client about medication that will be used to sedate the client Explaining the results of the exam

Instructing the client about the bowel preparation prior to the test

A client reports an iodine allergy. This information is most significant if the client is scheduled for which exam? Lung Scan Computed Tomography Magnetic Resonance Imaging Intravenous Pyelogram

Intravenous Pyelogram

While applying sterile gloves (open method), the cuff of the first glove rolls under itself about 1/4 inch. What is the best action for the nurse to take? Remove the glove and start over with a new pair. Wait until the second glove is in place and then unroll the cuff with the other sterile hand. Ask a colleague to assist by unrolling the cuff. Leave the cuff rolled under. End of Question 5

Leave the cuff rolled under. Rationale: It should not be necessary to unroll this small edge of the cuff. The most important consideration is the sterility of the fingers and hand that will be used to perform the sterile procedure. The rolled-under portion is now contaminated and should not be unrolled by the nurse or colleague since it would then touch the remaining sterile portion of the glove (option 3). Cognitive Level: Applying. Client Need: Safe, Effective Care Environment. Nursing Process: Implementation.

The following blood pressures, taken 6 months apart, were recorded from patients screened by the nurse at the assisted living facility. Which patient should be referred to the healthcare provider for hypertension evaluation? a) 120/80, 118/78, 124/82 b) 128/84, 124/86, 128/88 c) 148/82, 148/78, 134/86 d) 154/78, 118/76, 126/84

c) 148/82, 148/78, 134/86 The definition of hypertension requires two elevated blood pressure measurements in a row. All of the other choices describe prehypertension

Which of the following patients is most at risk for tachycardia? a) A healthy professional tennis player b) A patient admitted with hypothermia c) A patient with a fever of 39.4°C (103°F) d) A 90-year-old male taking beta blockers

c) A patient with a fever of 39.4°C (103°F) Patients with a fever have a high heart rate. A healthy athlete has a low heart rate because of conditioning. Hypothermia slows the heart. Beta-blockers reduce heart rate

A patient presents in the clinic with dizziness and fatigue. The nursing assistant reports a very slow radial pulse of 44. What is your priority intervention? a) Request that the nursing assistant repeat the pulse check b) Call for a stat electrocardiogram (ECG) c) Assess the patient's apical pulse and evidence of a pulse deficit d) Prepare to administer cardiac-stimulating medications

c) Assess the patient's apical pulse and evidence of a pulse deficit Your priority is to assess the patient first. The nurse cannot delegate vital signs to an unstable patient. Therefore first you determine if the patient has a pulse deficit. Calling for a stat electrocardiogram and preparing to administer cardiac-stimulating medications require notification of the health care provider and occur after you assess the patient.

A patient has been transferred to your unit from the respiratory intensive care unit, where he has been for the past 2 weeks recovering from pneumonia. He is receiving oxygen via 4 L nasal cannula. His respiratory rate is 26 breaths/min, and his oxygen saturation is 92%. In planning his care, which information is most helpful in determining your priority nursing interventions? a) Activity order b) Medication list c) Baseline vital signs d) Patient's perception of dyspnea

c) Baseline vital signs Knowledge of baseline vital signs is needed to prioritize care

Which description best fits that of serous drainage from a wound? a) Fresh bleeding b) Thick and yellow c) Clear, watery plasma d) Beige to brown and foul smelling

c) Clear, watery plasma Serous fluid generally is serum and presents as light red, almost clear fluid.

Which of the numbered areas is considered sterile on a person in the operating room? You may assume that all articles were sterile when applied.

area 1 Rationale: Sterile objects are considered unsterile if placed lower than the waist. Only area 1 in this situation would be considered sterile. Above the neck, higher than 2 inches above the elbow, below the waist/table, and the back are all considered unsterile. Cognitive Level: Applying. Client Need: Physiological Integrity. Nursing Process: Planning.

What does the Braden Scale evaluate? a) Skin integrity at bony prominences, including any wounds b) Risk factors that place the patient at risk for skin breakdown c) The amount of repositioning that the patient can tolerate d) The factors that place the patient at risk for poor healing

b) Risk factors that place the patient at risk for skin breakdown The Braden Scale measures factors in six subscales that can predict the risk of pressure ulcer development. It does not assess skin or wounds.

After surgery the patient with a closed abdominal wound reports a sudden "pop" after coughing. When the nurse examines the surgical wound site, the sutures are open, and pieces of small bowel are noted at the bottom of the now-opened wound. Which corrective intervention should the nurse do first? a) Allow the area to be exposed to air until all drainage has stopped b) Place several cold packs over the area, protecting the skin around the wound c) Cover the area with sterile, saline-soaked towels and immediately notify the surgical team; this is likely to indicate a wound evisceration d) Cover the area with sterile gauze, place a tight binder over it, and ask the patient to remain in bed for 30 minutes because this is a minor opening in the surgical wound and should reseal quickly

c) Cover the area with sterile, saline-soaked towels and immediately notify the surgical team; this is likely to indicate a wound evisceration If a patient has an opening in the surgical incision and a portion of the small bowel is noted, the small bowel must be protected until an emergency surgical repair can be done. The small bowel and abdominal cavity should be maintained in a sterile environment; thus sterile towels that are moistened with sterile saline should be used over the exposed bowel for protection and to keep the bowel moist.

A patient has been hospitalized for the past 48 hours with a fever of unknown origin. His medical record indicates tympanic temperatures of 38.7°C (101.6°F) (0400), 36.6°C (97.9°F) (0800), 36.9°C (98.4°F) (1200), 37.6°C (99.6°F) (1600), and 38.3°C (100.9°F) (2000). How would you describe this pattern of temperature measurements? a) Usual range of circadian rhythm measurements b) Sustained fever pattern c) Intermittent fever pattern d) Resolving fever pattern

c) Intermittent fever pattern The pattern returns to acceptable levels at least once in 24 hours interspersed with fever spikes

As you are obtaining the oxygen saturation on a 19-year-old college student with severe asthma, you note that she has black nail polish on her nails. You remove the polish from one nail, and she asks you why her nail polish had to be removed. Your best reply is: a) Nail polish attracts microorganisms and contaminates the finger sensor. b) Nail polish increases oxygen saturation. c) Nail polish interferes with sensor function. d) Nail polish creates excessive heat in sensor probe.

c) Nail polish interferes with sensor function. Nail polish reduces light transmission and can alter oxygen saturation measurement.

Which of the following is an indication for a binder to be placed around a surgical patient with a new abdominal wound? a) Collection of wound drainage b) Reduction of abdominal swelling c) Reduction of stress on the abdominal incision d) Stimulation of peristalsis (return of bowel function) from direct pressure

c) Reduction of stress on the abdominal incision A binder placed over the abdomen can provide protection to the abdominal incision by offering support and decreasing stress from coughing and movement.

When is an application of a warm compress indicated? (Select all that apply.) a) To relieve edema b) For a patient who is shivering c) To improve blood flow to an injured part d) To protect bony prominences from pressure ulcers

c) To improve blood flow to an injured part Warm compresses can improve circulation by dilating blood vessels, and they reduce edema. The moisture of the compress conducts heat

On assessing your patient's sacral pressure ulcer, you note that the tissue over the sacrum is dark, hard, and adherent to the wound edge. What is the correct stage for this patient's pressure ulcer? a) Stage II b) Stage IV c) Unstageable d) Suspected deep tissue damage

c) Unstageable To determine the stage of a pressure ulcer you examine the depth of the tissue involvement. Since the pressure ulcer assessed was covered with necrotic tissue, the depth could not be determined. Thus this pressure ulcer cannot be staged.

Which skin care measures are used to manage a patient who is experiencing fecal and urinary incontinence? a) Keeping the buttocks exposed to air at all times b) Using a large absorbent diaper, changing when saturated c) Using an incontinence cleaner, followed by application of a moisture-barrier ointment d) Frequent cleaning, applying an ointment, and covering the areas with a thick absorbent towel

c) Using an incontinence cleaner, followed by application of a moisture-barrier ointment Skin that is in contact with stool and urine can become moist and soft, allowing it to become damaged. The stool contains bacteria and in some cases enzymes that can harm the skin if in contact for a prolonged period of time. The use of an incontinence cleaner provides a gentle removal of stool and urine, and the use of the moisture-barrier ointment provides a protective layer between the skin and the next incontinence episode.

The nurse would assess for signs of hypomagnesemia in which of the following clients? Select all that apply. 1) A client with renal failure. 2) A client with pancreatitis. 3) A client taking magnesium-containing antacids. 4) A client with excessive nasogastric drainage. 5) A client with chronic alcoholism.

2) A client with pancreatitis. 4) A client with excessive nasogastric drainage. 5) A client with chronic alcoholism.

Which of the following signs or symptoms in an opioid-naïve patient is of greatest concern to the nurse when assessing the patient 1 hour after administering an opioid? 1) Oxygen saturation of 95% 2) Difficulty arousing the patient 3) Respiratory rate of 10 breaths/min 4) Pain intensity rating of 5 on a scale of 0 to 10

2) Difficulty arousing the patient

Your patient had 200 mL of ice chips and 900 mL intravenous (IV) fluid during your shift. Which total intake should you record?

1000 mL

The health care provider's order is 1000 mL 0.9% NaCl IV over 6 hours. Which rate do you program into the infusion pump?

167 mL/hr

The family of a patient who is confused and ambulatory insists that all four side rails be up when the patient is alone. What is the best action to take in this situation? (Select all that apply.) 1 Contact the nursing supervisor. 2 Restrict the family's visiting privileges. 3 Ask the family to stay with the patient if possible. 4 Inform the family of the risks associated with side-rail use. 5 Thank the family for being conscientious and put the four rails up. 6 Discuss alternatives with the family that are appropriate for this patient.

3 Ask the family to stay with the patient if possible. 4 Inform the family of the risks 6 Discuss alternatives with the family that are appropriate for this patient.

The nursing diagnosis readiness for enhanced communication is an example of a(n): 1. Risk nursing diagnosis. 2. Actual nursing diagnosis. 3. Health promotion nursing diagnosis 4. Wellness nursing diagnosis.

3. Health promotion nursing diagnosis

A patient is recovering from surgery for removal of an ovarian tumor. It is 1 day after her surgery. Because she has an abdominal incision and dressing and a history of diabetes, the nurse has selected a nursing diagnosis of risk for infection. Which of the following is an appropriate goal statement for the diagnosis? 1. Patient will remain afebrile to discharge. 2. Patient's wound will remain free of infection by discharge. 3. Patient will receive ordered antibiotic on time over next 3 days. 4. Patient's abdominal incision will be covered with a sterile dressing for 2 days.

2. Patient's wound will remain free of infection by discharge When selecting an at-risk diagnosis, the goal is to avoid or prevent the condition at risk, in this case infection. The statement "Patient will remain afebrile to discharge" is a potential outcome measure for the goal. The patient receiving an ordered antibiotic and having the abdominal incision covered are both interventions.

A client's learning outcome is "client will verbalize medication names, purpose, and appropriate precautions". Which documented statement reflects evidence of learning? 1. Taught name, purpose, and precautions for the new cardiac medication; seemed to understand. 2. Written information about the medication provided and reviewed; corrected responses were given to follow up questions 3. Written information read to client; stated he would read it when he got home 4. Asked questions about the medication; satisfied with the information

2. Written information about the medication provided and reviewed; corrected responses were given to follow up questions

You assess four patients. Which patient is at greatest risk for the development of hypocalcemia?

28-year-old Who has Acute Pancreatitis

You observe a nursing student taking a blood pressure (BP) on a patient. The patient's BP range over the past 24 hours is 132/64 to 126/72 mm Hg. The student used a BP cuff that was too narrow for the patient. Which of the following BP readings made by the student is most likely caused by the incorrect choice of BP cuff? a) 96/40 mm Hg b) 110/66 mm Hg c) 130/70 mm Hg d) 156/82 mm Hg

d) 156/82 mm Hg When you use a blood pressure cuff that is too narrow or short, your patient will most likely have a BP reading that is higher than it really is; you will get a false-high reading

Which of the following describes a hydrocolloid dressing? a) A seaweed derivative that is highly absorptive b) Premoistened gauze placed over a granulating wound c) A debriding enzyme that is used to remove necrotic tissue d) A dressing that forms a gel that interacts with the wound surface

d) A dressing that forms a gel that interacts with the wound surface A hydrocolloid dressing is made of materials that are adhesive and can form a gel over the open area of the wound. Since moisture enhances wound healing, the gel that forms places the wound in the proper environment for healing.

When repositioning an immobile patient, the nurse notices redness over a bony prominence. What is indicated when a reddened area blanches on fingertip touch? a) A local skin infection requiring antibiotics b) Sensitive skin that requires special bed linen c) A stage III pressure ulcer needing the appropriate dressing d) Blanching hyperemia, indicating the attempt by the body to overcome the ischemic episode.

d) Blanching hyperemia, indicating the attempt by the body to overcome the ischemic episode. When repositioning an immobile patient, it is important to assess all bony prominences for the presence of redness, which can be the first sign of impaired skin integrity. Pressing over the area compresses the blood vessels in the area; and, if the integrity of the vessels is good, the area turns lighter in color and then returns to the red color. However, if the area does not blanch when pressure is applied, tissue damage is likely.

A 52-year-old woman is admitted with dyspnea and discomfort in her left chest with deep breaths. She has smoked for 35 years and recently lost over 10 pounds. Her vital signs on admission are: HR 112, BP 138/82, RR 22, tympanic temperature 36.8°C (98.2°F), and oxygen saturation 94%. She is receiving oxygen at 2 L via a nasal cannula. Which vital sign reflects a positive outcome of the oxygen therapy? a) Temperature: 37°C (98.6°F) b) Radial pulse: 112 c) Respiratory rate: 24 d) Oxygen saturation: 96% e) Blood pressure: 134/78

d) Oxygen saturation: 96% Oxygen therapy increases oxygen saturation. Temperature is not affected by the oxygen. There is no change in heart rate. Administering oxygen should decrease the respiratory rate. The decline in blood pressure is unlikely to be caused by oxygen.

When obtaining a wound culture to determine the presence of a wound infection, from where should the specimen be taken? a) Necrotic tissue b) Wound drainage c) Drainage on the dressing d) Wound after it has first been cleaned with normal saline

d) Wound after it has first been cleaned with normal saline Drainage that has been present on the wound surface can contain bacteria from the skin, and the culture may not contain the true causative organisms of a wound infection. By cleaning the area before obtaining the culture, the skin flora is removed.

The nurse is assessing the character of a patient's migraine headache and asks, "Do you feel nauseated when you have a headache?" The patient's response is "yes." In this case the finding of nausea is which of the following? 1. An objective finding 2. A clinical inference 3. A validation 4. A concomitant symptom

4. A concomitant symptom

The nurse makes the following statement during a change of shift report to another nurse. "I assessed Mr. Diaz, my 61-year-old patient from Chile. He fell at home and hurt his back 3 days ago. He has some difficulty turning in bed, and he says that he has pain that radiates down his leg. He rates his pain at a 6, but I don't think it's that severe. You know that back patients often have chronic pain. He seems fine when talking with his family. Have you cared for him before?" What does the nurse's conclusion suggest? 1. The nurse is making an accurate clinical inference. 2. The nurse has gathered cues to identify a potential problem area. 3. The nurse has allowed stereotyping to influence her assessment. 4. The nurse wants to validate her information with the other nurse.

3. The nurse has allowed stereotyping to influence her assessment.

Older adults are cautioned about the long-term use of sedatives and hypnotics because these medications can: 1) Cause headaches and nausea. 2) Be expensive and difficult to obtain. 3) Cause severe depression and anxiety. 4) Lead to sleep disruption.

4) Lead to sleep disruption.

During an admission nursing assessment, a client with diabetes describes his leg pain as a "dull, burning sensation." The nurse recognizes this description to be characteristic of which type of pain? 1) Physiological 2) Somatic 3) Visceral 4) Neuropathic

4) Neuropathic Rationale: Options 2 and 3 are subcategories of physiological pain (option 1). A clue to the answer is that the client has diabetes, which often leads to diabetic peripheral neuropathy

A semiconscious client in the postanesthesia care unit (PACU) is experiencing dyspnea (difficulty breathing). Which action should the nurse perform first? 1) Place a pillow under the client's head. 2) Remove the oropharyngeal airway. 3) Administer oxygen by mask. 4) Reposition the client to keep the tongue forward.

4) Reposition the client to keep the tongue forward.

A patient has the nursing diagnosis of nausea. The nurse develops a care plan with the following interventions. Which are examples of collaborative interventions? 1. Provide frequent mouth care. 2. Maintain intravenous (IV) infusion at 100 mL/hr. 3. Administer prochlorperazine (Compazine) via rectal suppository. 4. Consult with dietitian on initial foods to offer patient. 5. Control aversive odors or unpleasant visual stimulation that triggers nausea.

4. Consult with dietitian on initial foods to offer patient.

A patient has been taught how to cough and deep breathe. Which evaluation method is most appropriate? a. Return demonstration b. Computer instruction c. Verbalization of steps d. Cloze test

ANS: A To demonstrate mastery of the skill, have the patient perform a return demonstration under the same conditions that will be experienced at home or in the place where the skill is to be performed. Computer instruction is use of a programmed instruction format in which computers store response patterns for learners and select further lessons on the basis of these patterns (programs can be individualized). Computer instruction is a teaching tool, rather than an evaluation tool. Verbalization of steps can be an evaluation tool, but it is not as effective as a return demonstration when evaluating a psychomotor skill. The Cloze test, a test of reading comprehension, asks patients to fill in the blanks in a written paragraph

Which statements by the nurse indicate a good understanding of patient education/teaching? (Select all that apply.) a. "Patient education is a standard for professional nursing practice." b. "Patient teaching falls within the scope of nursing practice." c. "Patient education is an essential component of safe, patient-centered care." d. "Patient education is not effective with children." e. "Patient teaching can increase health care costs." f. "Patient teaching should be documented in the chart."

ANS: A, B, C, F Patient education has long been a standard for professional nursing practice. All state Nurse Practice Acts acknowledge that patient teaching falls within the scope of nursing practice. Patient education is an essential component of providing safe, patient-centered care. It is important to document evidence of successful patient education in patients' medical records. Patient education is effective for children. Different techniques must be used with children. Creating a well-designed, comprehensive teaching plan that fits a patient's unique learning needs reduces health care costs, improves the quality of care, and ultimately changes behaviors to improve patient outcomes.

2. Which situation will cause the nurse to postpone a teaching session? (Select all that apply.) a. The patient is mildly anxious. b. The patient is fatigued. c. The patient is asking questions. d. The patient is hurting. e. The patient is febrile (high fever). f. The patient is in the acceptance phase.

ANS: B, D, E Any condition (e.g., pain, fatigue) that depletes a person's energy also impairs his or her ability to learn, so the session should be postponed until the pain is relieved and the patient is rested. Postpone teaching when an illness becomes aggravated by complications such as a high fever or respiratory difficulty. A mild level of anxiety motivates learning. When patients are ready to learn, they frequently ask questions. When the patient enters the stage of acceptance, the stage compatible with learning, introduce a teaching plan.

A nurse has taught a patient about healthy eating habits. Which learning objective/outcome is most appropriate for the affective domain? a. The patient will state three facts about healthy eating. b. The patient will identify two foods for a healthy snack. c. The patient will verbalize the value of eating healthy. d. The patient will cook a meal with low-fat oil.

ANS: C Affective learning deals with expression of feelings and acceptance of attitudes, opinions, or values. Having the patient value healthy eating habits falls within the affective domain. Stating three facts or identifying two foods for a healthy snack falls within the cognitive domain. Cooking falls within the psychomotor domain

2. A nurse is teaching a group of healthy adults about the benefits of flu immunizations. Which purpose of patient education is the nurse fulfilling? a. Restoration of health b. Coping with impaired functions c. Promotion of health and illness prevention d. Health analogies

ANS: C As a nurse, you are a visible, competent resource for patients who want to improve their physical and psychological well-being. In the school, home, clinic, or workplace, you promote health and prevent illness by providing information and skills that enable patients to assume healthier behaviors. Injured and ill patients need information and skills to help them regain or maintain their level of health; this is referred to as restoration of health. Not all patients fully recover from illness or injury. Many have to learn to cope with permanent health alterations; this is known as coping with impaired functions. Analogies supplement verbal instruction with familiar images that make complex information more real and understandable. For example, when explaining arterial blood pressure, use an analogy of the flow of water through a hose.

A nurse is going to teach a patient about hypertension. Which action should the nurse implement first? a. Set mutual goals for knowledge of hypertension. b. Teach what the patient wants to know about hypertension. c. Assess what the patient already knows about hypertension. d. Evaluate the outcomes of patient education for hypertension.

ANS: C Assessment is the first step of any teaching session, then diagnosing, planning, implementation, and evaluation. An effective assessment provides the basis for individualized patient teaching. Assessing what the adult patient currently knows improves the outcomes of patient education.

A nurse is teaching an older adult patient about strokes. Which teaching technique is most appropriate for the nurse to use? a. Use a pamphlet about strokes with large font in blues and greens. b. Speak in a high tone of voice to describe strokes. c. Begin and end each teaching session with the most important information about strokes. d. Provide specific information about strokes in frequent, large amounts.

ANS: C Begin and end each teaching session with the most important information when teaching older adult patients. Also, if using written material, assess the patient's ability to read and use information that is printed in large type and in a color that contrasts highly with the background (e.g., black 14-point print on buff-colored paper). Avoid blues and greens because they are more difficult to see. Speak in a low tone of voice (lower tones are easier to hear than higher tones). Provide specific information in frequent, small (not large) amounts for older adult patients.

A nurse is teaching a culturally diverse patient about nutritional needs. What must the nurse do first before starting the teaching session? a. Obtain pictures of food. b. Get an interpreter. c. Establish a rapport. d. Refer to a dietitian.

ANS: C Establishing a rapport is important for all patients, especially culturally diverse patients, before starting teaching sessions. Obtaining pictures of food, getting an interpreter, and referring to a dietitian all occur after rapport is established.

17. Which nursing action is most appropriate for assessing a patient's learning needs? a. Assess the patient's total health care needs. b. Assess the patient's health literacy. c. Assess all sources of patient data. d. Assess the goals of patient care.

ANS: B Because health literacy influences how you deliver teaching strategies, it is critical for you to assess a patient's health literacy before providing instruction. The nursing process requires assessment of all sources of data to determine a patient's total health care needs. Evaluation of the teaching process involves determining outcomes of the teaching/learning process and the achievement of learning objectives, not patient care. Assessing the goal of meeting patient care is the evaluation component of the nursing process.

A nurse is teaching a patient who has low health literacy about chronic obstructive pulmonary disease (COPD) while giving COPD medications. Which technique is most appropriate for the nurse to use? a. Use complex analogies to describe COPD. b. Include the most important information on COPD at the beginning of the session. c. Ask for feedback to assess understanding of COPD at the end of the session. d. Offer pamphlets about COPD written at the eighth grade level with large type

ANS: B Include the most important information at the beginning of the session for illiterate patients or patients with a learning disability. Also, use visual cues and simple, not complex, analogies when appropriate. Another technique is to frequently ask patients for feedback to determine whether they comprehend the information. Additionally, provide teaching materials that reflect the reading level of the patient, with attention given to short words and sentences, large type, and simple format (generally, information written on a fifth grade reading level is recommended for adult learners).

5. Which action best indicates that learning has occurred? a. A nurse presents information about diabetes. b. A patient demonstrates how to inject insulin. c. A family member listens to a lecture on diabetes. d. A primary care provider hands a diabetes pamphlet to the patient.

ANS: B Learning is the purposeful acquisition of new knowledge, attitudes, behaviors, and skills. Complex patterns are required if the patient is to learn new skills, change existing attitudes, transfer learning to new situations, or solve problems. A new mother exhibits learning when she demonstrates how to bathe her newborn. A nurse presenting information and a primary care provider handing a pamphlet to a patient are examples of teaching. A family member listening to a lecture does not indicate that learning occurred; a change in knowledge, attitudes, behaviors, and/or skills must be evident.

After a teaching session on taking blood pressures, the nurse tells the patient, "You took that blood pressure like an experienced nurse." What type of reinforcement did the nurse use? a. Material b. Activity c. Social d. Entrusting

ANS: C Three types of reinforcers are social, material, and activity. When a nurse works with a patient, most reinforcers are social and are used to acknowledge a learned behavior (e.g., smiles, compliments, words of encouragement). Examples of material reinforcers include food, toys, and music. Activity reinforcers rely on the principle that a person is motivated to engage in an activity if he or she has the opportunity to engage in a more desirable activity after completion of the task. The entrusting approach is a teaching approach that provides the patient the opportunity to manage self-care. It is not a type of reinforcement.

12. A toddler is going to have surgery on the right ear. Which teaching method is most appropriate for this developmental stage? a. Encourage independent learning. b. Use discussion throughout the teaching session. c. Apply a bandage to a doll's ear. d. Develop a problem-solving scenario.

ANS: C Use play to teach a procedure or activity (e.g., handling examination equipment, applying a bandage to a doll) to toddlers. Encouraging independent learning is for the middle-aged adult. Use of discussion is for older children, adolescents, and adults, not for toddlers. Use problem solving to help adolescents make choices. Problem solving is too advanced for a toddler.

A nurse is assessing the ability to learn of a patient who has recently experienced a stroke. Which question/statement will best assess the patient's ability to learn? a. "What do you want to know about strokes?" b. "On a scale from 1 to 10, tell me where you rank your desire to learn." c. "Do you feel strong enough to perform the tasks I will teach you?" d. "Please read this handout and tell me what it means."

ANS: D A patient's reading level affects ability to learn. Reading level is often difficult to assess because patients who are functionally illiterate are often able to conceal it by using excuses such as not having the time or not being able to see. One way to assess a patient's reading level and level of understanding is to ask the patient to read instructions from an educational handout and then explain their meaning. Asking patients what they want to know identifies previous learning and learning needs and preferences; it does not assess ability to learn. Motivation is related to readiness to learn, not ability to learn. Just asking a patient if they feel strong is not as effective as actually assessing the patient's streng

16. A nurse is teaching the staff about nursing and teaching processes. Which information should the nurse include regarding the teaching process? During the teaching process, what should the nurse do? a. Assess all sources of data. b. Identify that it is the same as the nursing process. c. Perform nursing care therapies. d. Focus on a patient's learning needs.

ANS: D The teaching process focuses on the patient's learning needs and willingness and capability to learn. Nursing and teaching processes are not the same. All the rest are components of the nursing process: Assess all sources of data and perform nursing care therapies

During an assessment, the nurse learns that the client has a history of liver disease. Which diagnostic tests might be indicated for this client? Select all that apply. Alanine aminotransferase (ALT) Myoglobin Cholesterol Ammonia Brain natriuretic peptide or B-Type natriuretic peptide (BNP)

Alanine aminotransferase (ALT) Ammonia ALT is an enzyme that contributes to protein and carbohydrate metabolism. An increase in the enzyme indicates damage to the liver. The liver contributes to the metabolism of protein, which results in the production of ammonia. If the liver is damaged, the ammonia level is increased. Options 2, 3, and 5 (myoglobin, cholesterol, and BNP) are relevant for heart disease.

7. A nurse teaches a patient with heart failure healthy food choices. The patient states that eating yogurt is better than eating cake. In this situation, which element represents feedback? a. The nurse b. The patient c. The nurse teaching about healthy food choices d. The patient stating that eating yogurt is better than eating cake

ANS: D Feedback should show the success of the learner in achieving objectives (i.e., the learner verbalizes information or provides a return demonstration of skills learned). The nurse is the sender. The patient is the receiver. The teaching is the message.

When assisting with a bone marrow biopsy, the nurse should take which action? Assist the client to a right side-lying position after the procedure. Observe for signs of dyspnea, pallor, and coughing. Assess for bleeding and hematoma formation for several days after the procedure. Stand in front of the client and support the back of the neck and knees.

Assess for bleeding and hematoma formation for several days after the procedure. Rationale: Bone marrow aspiration includes deep penetration into soft tissue and large bones such as the sternum and iliac crest. This penetration can result in bleeding. The client should be observed for bleeding in the days following the procedure. Option 1 is a nursing action during a liver biopsy. Option 2 is a nursing action for a thoracentesis, and Option 4 is a nursing action for a lumbar puncture.

A primary care provider writes a prescription for 0.15 milligram of digoxin intravenously every day. The medication is available in a concentration of 400 micrograms per mL. How many mL will the nurse administer?

Answer: 0.38 mL Rationale: After converting to like numbers, the formula would be set up as follows: 400 micrograms = 1 mL 150 micrograms = X mL Cross multiply (400 X = 150 Divide by 400 X = 0.375

15. Which situation indicates to the nurse that the patient is ready to learn? a. A patient has sufficient upper body strength to move from a bed to a wheelchair. b. A patient has the ability to grasp and apply the elastic bandage. c. A patient with a below-the-knee amputation is motivated about how to walk with assistive devices. d. A patient has normal eyesight to identify the markings on a syringe and coordination to handle a syringe.

ANS: C Motivation or readiness to learn sometimes results from social task mastery, or physical motives may be involved. Often patient motives are physical. Some patients are motivated to return to a level of physical normalcy. For example, a patient with a below-the-knee amputation is motivated to learn how to walk with assistive devices. Do not confuse readiness to learn with ability to learn. All the other answers are examples of ability to learn because this often depends on the patient's level of physical development and overall physical health. To learn psychomotor skills, a patient needs to possess a certain level of strength, coordination, and sensory acuity. For example, it is useless to teach a patient to transfer from a bed to a wheelchair if he or she has insufficient upper body strength. An older patient with poor eyesight or an inability to grasp objects tightly cannot learn to apply an elastic bandage or handle a syringe.

10. A nurse wants the patient to begin to accept the chronic nature of diabetes. Which teaching technique should the nurse use to enhance learning? a. Lecture b. Demonstration c. Role play d. Question and answer session

ANS: C Affective learning deals with expression of feelings and acceptance of attitudes, opinions, or values. Role play and discussion (one-on-one and group) are effective teaching methods for the affective domain. Lecture and question and answer sessions are effective teaching methods for the cognitive domain. Demonstration is an effective teaching method for the psychomotor domain.

What physiological conditions are contraindicated for using heat as a therapy? (Select all that apply.) 1. The first 24 hours of injury 2. Active hemorrhage 3. Noninflammatory edema 4. Localized malignant tumor

All of the above; Heat causes vasodilatation and increases blood flow to the affected area bringing oxygen, nutrients, antibodies, and leukocytes. A possible disadvantage of heat is that it increases capillary permeability, which allows extracellular fluid and substances to pass through the capillary walls and may result in edema or an increase in preexisting edema. Contraindications include: the first 24 hours of injury, active hemorrhage, noninflammatory edema, localized malignant tumor, and skin disorder that causes redness or blisters.

An elderly man is admitted to the medical unit with a diagnosis of dehydration. Which sign or symptom is most representative of a sodium imbalance? 1) Hyperreflexia 2) Mental confusion 3) Irregular pulse 4) Muscle weakness

2) Mental confusion

How does the nurse support a culture of safety? (Select all that apply.) 1 Completing incident reports when appropriate 2 Completing incident reports for a near miss 3 Communicating product concerns to an immediate supervisor 4 Identifying the person responsible for an incident

1 Completing incident reports when appropriate 2 Completing incident reports for a near miss 3 Communicating product concerns to an immediate supervisor

To ensure the safe use of oxygen in the home by a patient, which of the following teaching points does the nurse include? (Select all that apply.) 1 Smoking is prohibited around oxygen. 2 Demonstrate how to adjust the oxygen flow rate based on patient symptoms. 3 Do not use electrical equipment around oxygen. 4 Special precautions may be required when traveling with oxygen

1 Smoking is prohibited around oxygen. 3 Do not use electrical equipment around oxygen. 4 Special precautions may be required when traveling with oxygen

Which intervention is appropriate to include on a care plan for improving sleep in the older adult? 1) Decrease fluids 2 to 4 hours before sleep 2) Exercise in the evening to increase fatigue 3) Allow the patient to sleep as late as possible 4) Take a nap during the day to make up for lost sleep

1) Decrease fluids 2 to 4 hours before sleep

The nurse is taking a sleep history from a patient. Which statement made by the patient needs further follow-up? 1) I always feel tired when I wake up in the morning. 2) I go to bed at the same time each night. 3) It takes me about 15 minutes to fall asleep. 4) Sometimes I have to get up during the night to urinate.

1) I always feel tired when I wake up in the morning.

Which of the following would indicate a significant cue when comparing data to standards? Select all that apply. 1) The client has moved partway toward a set goal (e.g., weight loss). 2) The client's vision is within normal range only when wearing glasses. 3) A child is able to control bladder and bowels at age 18 months. 4) A woman widowed recently states she is "unable to cry." 5) A 16-year-old high school student reports spending 6 hours doing homework five nights per week.

1) The client has moved partway toward a set goal (e.g., weight loss). 4) A woman widowed recently states she is "unable to cry." 5) A 16-year-old high school student reports spending 6 hours doing homework five nights per week. Rationale: A client's movement toward a goal (option 1) or whose behavior is inconsistent with population norms (options 4 and 5) represents a cue that further analysis toward creating a nursing diagnosis is required. Corrected vision (option 2) and bladder and bowel control at age 18 months (option 3) are consistent with population norms.

A postoperative patient is currently asleep. Therefore the nurse knows that: 1) The sedative administered may have helped him sleep, but assessment of pain is still needed. 2) The intravenous (IV) pain medication is effectively relieving his pain. 3) Pain assessment is not necessary. 4) The patient can be switched

1) The sedative administered may have helped him sleep, but assessment of pain is still needed.

Two months ago a client weighed 195 pounds. The current weight is 182 pounds. Calculate the client's precent weight loss and determine its significance. 1. ?% weight loss 2. Not significant 3. Significant weight loss 4. Severe weight loss

1. 6.7% weight loss 2. Not significant

What technique(s) best encourage(s) a patient to tell his or her full story? (Select all that apply.) 1. Active listening 2. Back channeling 3. Validating 4. Use of open-ended questions 5. Use of closed-ended questions

1. Active listening 2. Back channeling 4. Use of open-ended questions

Review the following nursing diagnoses and identify the diagnoses that are stated correctly. (Select all that apply.) 1. Anxiety related to fear of dying 2. Fatigue related to chronic emphysema 3. Need for mouth care related to inflamed mucosa 4. Risk for infection

1. Anxiety related to fear of dying 4. Risk for infection

What is the proper technique with gravity tube feeding? 1.Feeding bag is hung 1 foot higher than the tube's insertion point into the client 2.Nurse administers the next feeding only if there is less than 25mL of residual volume from the previous feeding 3.Place client in the left lateral position 4.Feeding is administered directly from the refrigerator

1.Feeding bag is hung 1 foot higher than the tube's insertion point into the client

An 82yearold patient who resides in a nursing home has the following three nursing diagnoses: risk for fall, impaired physical mobility related to pain, and wandering related to cognitive impairment. The nursing staff identified several goals of care. Match the goals with the appropriate outcome statements. Goals 1. Patient will ambulate independently in 3 days. _____ 2. Patient will be injury free for 1 month. _____ 3. Patient will be less agitated. _____ 4. Patient will achieve pain relief. _____ Outcomes a. Patient will express fewer nonverbal signs of discomfort. b. Patient will follow a set care routine. c. Patient will walk correctly using a walker. d. Patient will exit a low bed without falling.

1c 2d 3b 4a

A nurse is assigned to a new patient admitted to the nursing unit following admission through the emergency department. The nurse collects a nursing history and interviews the patient. Place the following steps for making a nursing diagnosis in the correct order. _____ 1. Considers context of patient's health problem and selects a related factor _____ 2. Reviews assessment data, noting objective and subjective clinical criteria _____ 3. Clusters clinical criteria that form a pattern _____ 4. Chooses diagnostic label

2 3 4 1

The nurse recognizes which of the following as a benefit of using a standardized care plan? 1) No individualization is needed. 2) The nurse chooses from a list of interventions. 3) They are much shorter than nurse-authored care plans. 4) They have been approved by accrediting agencies

2) The nurse chooses from a list of interventions. Rationale: Standardized care plans provide a list of interventions from which the nurse can choose. The plan must still be individualized (option 1). Standardized plans could be longer or shorter than nurse-authored ones (option 3), but have not been approved by any outside accreditor (option 4).

The patient rates his pain as a 6 on a scale of 0 to 10, with 0 being no pain and 10 being the worst pain. The patient's wife says that he can't be in that much pain since he has been sleeping for 30 minutes. Which is the most accurate resource for assessing the pain? 1) The patient's wife is the best resource for determining the level of pain since she has been with him continually for the entire day. 2) The patient's report of pain is the best method for assessing the pain. 3) The patient's health care provider has the best knowledge of the level of pain that the patient that should be experiencing. 4) The nurse is the most experienced at assessing pain.

2) The patient's report of pain is the best method for assessing the pain.

A new medical resident writes an order for OxyContin SR 10 mg PO q12 hours prn. Which part of the order does the nurse question? 1) The drug 2) The time interval 3) The dose 4) The route

2) The time interval

A nurse is scheduling a teaching situation. Which client is most ready to learn? 1. A 45 year old man whose doctor just informed him that he has cancer 2. A 3 year old child whose parents are reading a story book about going to the hospital 3. A 60 year old female who received medication 5 minutes ago fr relief of abdominal pain 4. A 70 year old man, recovering from a stroke, who has returned from physical therapy

2. A 3 year old child whose parents are reading a story book about going to the hospital- The client is most ready to learn is experiencing or has recently experienced the least amount of stress or is the least preoccupied with other concerns.

A client is scheduled to have a diagnostic procedure. Which questions by the nurse will most likely produce a "teachable moment"? Select all that apply 1. Have you eve had this procedure before? 2. What are your concerns about this procedure? 3. What would you like to know about the procedure? 4. Are you prepared for this procedure? 5. What have you heard or read about the procedure?

2. What are your concerns about this procedure? 3. What would you like to know about the procedure? 5. What have you heard or read about the procedure? These are open ended questions that will give opportunity to provide information that will help the nurse assess level of knowledge and subsequently provide/discuss needed information with the client.

The client is most likely to require the greatest amount of analgesia for pain during which period? 1) Immediately after surgery 2) 4 hours after surgery 3) 12 to 36 hours after surgery 4) 48 to 60 hours after surgery

3) 12 to 36 hours after surgery

Lateral spinal curvature. A. Scoliosis B. Stenosis C. Striae D. Lordosis

A

A patient with chronic low back pain who took an opioid around-the-clock (ATC) for the past year decided to abruptly stop the medication for fear of addiction. He is now experiencing shaking chills, abdominal cramps, and joint pain. The nurse recognizes that this patient is experiencing symptoms of: 1) Addiction. 2) Tolerance. 3) Pseudoaddiction. 4) Physical dependence.

4) Physical dependence.

A client is admitted with gastrointestinal bleeding. One of the earliest and most important blood tests completed will be: Electrolyte Panel Arterial Blood Gases Liver Panel Complete Blood Count

Complete Blood Count

The nurse administers an IV solution of D5 1/2NS to a postoperative client. This is classified as what type of intravenous solution?

Hypertonic

A nurse is concerned about a client's ability to withstand exposure to pathogens. What blood component should the nurse monitor?

Neutrophils; Neutrophils are the most numerous leukocytes (white blood cells) and are a primary defense against infection because they ingest and destroy microorganisms (phagocytosis).

The nurse identified that the patient has pain on a scale of 7, he winces during movement, and he expresses discomfort over the incisional area. He guards the area by resisting movement. The incision appears to be healing, but there is natural swelling. Write a three-part nursing diagnostic statement using the PES format.

Answer: Acute pain r/t incisional trauma e/b pain reported at 7, with guarding, and restricted turning and positioning

A nurse manager is reviewing care of a client who has had a seizure with nurses on the unit. Which of the following statements by a nurse requires further instruction? A. "I will place the client on his side." B. "I will go to the nurses' station for assistance." C. "I will administer medications as prescribed." D. "I will be prepared to insert an airway."

B. "I will go to the nurses' station for assistance."

A nurse is providing discharge instructions to a client who has a prescription for the use of oxygen in his home. Which of the following should the nurse teach the client about using oxygen safely in his home? (Select all that apply.) A. Family members who smoke must be at least 10 ft from the client when oxygen is in use. B. Nail polish should not be used near a client who is receiving oxygen. C. A "No Smoking" sign should be placed on the front door. D. Cotton bedding and clothing should be replaced with items made from wool. E. A fire extinguisher should be readily available in the home.

B. Nail polish should not be used near a client who is receiving oxygen. C. A "No Smoking" sign should be placed on the front door. E. A fire extinguisher should be readily available in the home.

In a patient with pneumonia, the nurse hears high-pitched, continuous musical sounds over the bronchi on expiration. These sounds are called: A. Rhonchi B. Crackles C. Wheezes D. Friction rubs

C

For a client with a previous blood pressure of 138/74 and pulse of 64, approximately how long should the nurse take to release the blood pressure cuff in order to obtain an accurate reading? 10-20 seconds 30-45 seconds 1-1.5 minutes 3-3.5 minutes

Correct Answer: 30-45 seconds Rationale: If the cuff is inflated to about 30 mmHg over previous systolic pressure, that would be 168. To ensure that the diastolic has been determined, the cuff should be released slowly until the mid-60s mmHg (and then completely) for someone with a previous reading of 74. The cuff should be deflated at a rate of 2 to 3 mm per second. Thus, a range of 90 mmHg will require 30 to 45 seconds. Cognitive Level: Analyzing. Client Need: Health Promotion and Maintenance. Nursing Process: Implementation.

Which statement made by a patient of a 2-month-old infant requires further education? A. I'll continue to use formula for the baby until he is a least a year old. B. I'll make sure that I purchase iron-fortified formula. C. I'll start feeding the baby cereal at 4 months. D. I'm going to alternate formula with whole milk starting next month.

D

A patient is to receive cephalexin (Kefl ex) 500 mg PO. The pharmacy has sent 250-mg tablets. How many tablets does the nurse administer? A. ½ tablet B. 1 tablet C. 1 ½ tablets D. 2 tablets

D. Rationale: Using dimensional analysis: Tablets = 1 tablet/250 mg x 500 mg = 500/250 = 2 tablets.

Assessment findings consistent with intravenous (IV) fluid infiltration include:

Edema and Pain; Pallor and Coolness

Which of the following are primary risk factors for pressure ulcers? Select all that apply. 1. Low-protein diet 2. Insomnia 3. Lengthy surgical procedures 4. Fever 5. Sleeping on a waterbed

1, 3, & 4; Risk factors for pressure ulcers include a low-protein diet, lengthy surgical procedures, and fever. Protein is needed for adequate skin health and healing. During surgery, the client is on a hard surface and may not be well protected from pressure on bony prominences. Fever increases skin moisture, which can lead to skin breakdown, plus the stress on the body from the cause of the fever could impair circulation and skin integrity. Insomnia (option 2) would generally involve restless sleeping, which transfers pressure to different parts of the body and would reduce chances of skin breakdown. A waterbed (option 5) distributes pressure more evenly than a regular mattress and, thus, actually reduces the chance of skin breakdown.

A nurse working on a medicine nursing unit is assigned to a 78-year-old patient who just entered the hospital with symptoms of H1N1 flu. The nurse finds the patient to be short of breath with an increased respiratory rate of 30 breaths/min. He lost his wife just a month ago. The nurse's knowledge about this patient results in which of the following assessment approaches at this time? (Select all that apply.) 1. A problem-focused approach 2. A structured comprehensive approach 3. Using multiple visits to gather a complete database 4. Focusing on the functional health pattern of role relationship

1. A problem-focused approach 3. Using multiple visits to gather a complete database

A patient who visits the allergy clinic tells the nurse practitioner that he is not getting relief from shortness of breath when he uses his inhaler. The nurse decides to ask the patient to explain how he uses the inhaler, when he should take a dose of medication, and what he does when he gets no relief. On the basis of Gordon's functional health patterns, which pattern does the nurse assess? 1. Health perception-health management pattern 2. Value-belief pattern 3. Cognitive-perceptual pattern 4. Coping-stress tolerance pattern

1. Health perception-health management pattern

A patient has been in the hospital for 2 days because of newly diagnosed diabetes. His medical condition is unstable, and the medical staff is having difficulty controlling his blood sugar. The physician expects that the patient will remain hospitalized at least 3 more days. The nurse identifies one nursing diagnosis as deficient knowledge regarding insulin administration related to inexperience with disease management. What does the nurse need to determine before setting the goal of "patient will selfadminister insulin?" (Select all that apply.) 1. Goal within reach of the patient 2. The nurse's own competency in teaching about insulin 3. The patient's cognitive function 4. Availability of family members to assist

1. Goal within reach of the patient 3. The patient's cognitive function

Thirty minutes after application is initiated, the client requests that the nurse leave the heating pad in place. The nurse explains to the client that: 1. Heat application for longer than thirty minutes can actually cause the opposite effect (constriction) of the one desired (dilation) 2. It will be acceptable to leave the pad in place for another thirty minutes

1. Heat application for longer than thirty minutes can actually cause the opposite effect (constriction) of the one desired (dilation); The heating pads need to be removed. After 30 minutes of heat application, the blood vessels in the area will begin to exhibit the rebound effect resulting in vasoconstriction. Lowering the temperature, but still delivering heat -dry or moist- will not prevent the rebound effect. The visual appearance of the site on inspection (option 3) does not indicate if rebound is occurring.

Which client information collected by the nurse reflects a systemic response to a wound infection? 1. Hyperthermia 2. Exudate 3. Edema 4. Pain

1. Hyperthermia; Hyperthermia is a common systemic response to infection. With hyperthermia, microorganisms or endotoxins stimulate phagocytotic cells that release pyrogens, which stimulate the hypothalamic thermoregulatory center, resulting in fever. Exudate, edema, and pain are all signs of infection but are considered local responses to infection or injury.

15. A nurse is starting on the evening shift and is assigned to care for a patient with a diagnosis of impaired skin integrity related to pressure and moisture on the skin. The patient is 72 years old and had a stroke. The patient weighs 250 pounds and is difficult to turn. As the nurse makes decisions about how to implement skin care for the patient, which of the following actions does the nurse implement? (Select all that apply.) 1. Review the set of all possible nursing interventions for the patient's problem 2. Review all possible consequences associated with each possible nursing action 3. Consider own level of competency 4. Determine the probability of all possible consequences

1. Review the set of all possible nursing interventions for the patient's problem 2. Review all possible consequences associated with each possible nursing action 4. Determine the probability of all possible consequences When making decisions about implementation, reviewing all possible interventions and consequences and determining the probability of consequences are necessary steps. The nurse is responsible for having the necessary knowledge and clinical competency to perform an intervention, but this is not part of the decision making involved

Which steps does the nurse follow when he or she is asked to perform an unfamiliar procedure? (Select all that apply.) 1. Seeks necessary knowledge 2. Reassesses the patient's condition 3. Collects all necessary equipment 4. Delegates the procedure to a more experienced staff member 5. Considers all possible consequences of the procedure

1. Seeks necessary knowledge 3. Collects all necessary equipment 5. Considers all possible consequences of the procedure You require additional knowledge and skills in situations in which you are less experienced. When you are asked to administer a new procedure with which you are unfamiliar, follow the three choices: seek necessary knowledge, collect necessary equipment, and consider all possible consequences of the procedure. Collecting necessary equipment and considering potential consequences is needed for any procedure

A client has a learning outcome of "Select foods that are low in fat content." Which statement reflects that the client has met this outcome? 1. I understand the importance of maintaining a low-fat diet 2. I feel better about myself now 3. See how I revised my favorite recipe to be lower in fat 4. Since changing my diet, my husband is also losing weight

3. See how I revised my favorite recipe to be lower in fat

The nurse is teaching a patient how to perform a testicular self-examination. Which statement by the nurse is correct? A. "The testes are normally round and feel smooth and rubbery." B. "The best time to do a testicular self-examination is before your bath or shower." C. "Perform a testicular self-examination weekly to detect signs of testicular cancer." D. "Since you are over 40 years old, you are in the highest risk group for testicular cancer."

A

The nurse teaches a patient about cranial nerves to help explain why the patient's right side of the mouth droops instead of moving up into a smile. What nerve does the nurse explain to the patient? A. VII — Facial B. V — Trigeminal C. XII — Hypoglossal D. XI— Spinal accessory

A

A 58-year-old patient with nerve deafness has come to his doctor's office for a routine examination. The patient wears two hearing aids. The advanced practice nurse who is conducting the assessment uses which of the following approaches while conducting the interview with this patient? (Select all that apply.) 1. Maintain a neutral facial expression 2. Lean forward when interacting with the patient 3. Acknowledge the patient's answers through head nodding 4. Limit direct eye contact

2. Lean forward when interacting with the patient 3. Acknowledge the patient's answers through head nodding

Which of the following behaviors would indicate that the nurse was utilizing the assessment phase of the nursing process to provide nursing care? 1) Proposes hypotheses. 2) Generates desired outcomes. 3) Reviews results of laboratory tests. 4) Documents care.

3) Reviews results of laboratory tests. Rationale: During assessment, data are collected, organized, validated, and documented. Hypotheses are generated during diagnosing; outcomes are set during planning; and documentation occurs throughout the nursing process.

The nurse notices that a patient has received oxycodone/acetaminophen (Percocet) (5/325) two tablets PO every 3 hours for the past 3 days. What concerns the nurse the most? 1) The patient's level of pain 2) The potential for addiction 3) The amount of daily acetaminophen 4) The risk for gastrointestinal bleeding

3) The amount of daily acetaminophen

The nurse incorporates which priority nursing intervention into a plan of care to promote sleep for a hospitalized patient? 1) Have patient follow hospital routines 2) Avoid awakening patient for nonessential tasks 3) Give prescribed sleeping medications at dinner 4) Turn television on low to late-night programming.

2) Avoid awakening patient for nonessential tasks

An elderly nursing home resident has refused to eat or drink for several days and is admitted to the hospital. The nurse should expect which assessment finding? 1) Increased blood pressure 2) Weak, rapid pulse 3) Moist mucous membranes 4) Jugular vein distention

2) Weak, rapid pulse

Which is a normal finding on auscultation of the lungs? 1. Tympany over the right upper lobe. 2. Resonance over the left upper lobe. 3. Hyperresonance over the left lower lobe. 4. Dullness above the left 10th intercostal space.

2. Resonance over the left upper lobe.

Which of the following nursing diagnoses is most appropriate for a client with a moss mass index (BMI) of 35? 1. Imbalanced Nutrition: Less Than Body Requirements 2. Imbalanced Nutrition: More Than body Requirements 3.Risk for Imbalanced Nutrition 4.Deficient Knowledge

2. Imbalanced Nutrition: More Than body Requirements

A parent calls the pediatrician's office frantic about the bottle of cleaner that her 2-year-old son drank. Which of the following is the most important instruction the nurse gives to this parent? A. Give the child milk. B. Give the child syrup of ipecac. C. Call the poison control center. D. Take the child to the emergency department.

C. Call the poison control center.

Your client has a Braden scale score of 17. Which is the most appropriate nursing action? 1. Assess the client again in 24h; the score is within normal limits. 2. Implement a turning schedule; the client is at increased risk for skin breakdown. 3. Apply a transparent wound barrier to major pressure sites; the client is at moderate risk for skin breakdown. 4. Request an order for a special low-air-loss bed; the client is at very high risk for skin breakdown.

2. Implement a turning schedule; the client is at increased risk for skin breakdown; A score ranging from 15 to 18 is considered at risk and a turning schedule is appropriate. Option 1 requires a score above 18 (normal and ongoing assessment indicated). Option 3, moderate risk, for which a transparent barrier would be appropriate, is applied to persons with scores of 13 to 14. Option 4, very high risk, is assigned for those with a score of 9 or less.

The nurse in a geriatric clinic collects the following information from an 82-year-old patient and her daughter, the family caregiver. The daughter explains that the patient is "always getting lost." The patient sits in the chair but gets up frequently and paces back and forth in the examination room. The daughter says, "I just don't know what to do because I worry she will fall or hurt herself." The daughter states that, when she took her mother to the store, they became separated, and the mother couldn't find the front entrance. The daughter works part time and has no one to help watch her mother. Which of the data form a cluster, showing a relevant pattern? (Select all that apply.) 1. Daughter's concern of mother's risk for injury 2. Pacing 3. Patient getting lost easily 4. Daughter working part time 5. Getting up frequently

2. Pacing 3. Patient getting lost easily 5. Getting up frequently

A nurse checks a patient's intravenous (IV) line in his right arm and sees inflammation where the catheter enters the skin. She uses her finger to apply light pressure (i.e., palpation) just above the IV site. The patient tells her the area is tender. The nurse checks to see if the IV line is running at the correct rate. This is an example of what type of assessment? 1. Agenda setting 2. Problem-focused 3. Objective 4. Use of a structured database format

2. Problem-focused

A nurse from home health is talking with a nurse who works on an acute medical division within a hospital. The home health nurse is making a consultation. Which of the following statements describes the unique difference between a nursing care plan from a hospital versus one for home care? 1. The goals of care will always be more long term. 2. The patient and family need to be able to independently provide most of the health care. 3. The patient's goals need to be mutually set with family members who will care for him or her. 4. The expected outcomes need to address what can be influenced by interventions.

2. The patient and family need to be able to independently provide most of the health care.

A nurse is assigned to a patient who has returned from the recovery room following surgery for a colorectal tumor. After an initial assessment the nurse anticipates the need to monitor the patient's abdominal dressing, intravenous (IV) infusion, and function of drainage tubes. The patient is in pain, reporting 6 on a scale of 0 to 10, and will not be able to eat or drink until intestinal function returns. The family has been in the waiting room for an hour, wanting to see the patient. The nurse establishes priorities first for which of the following situations? (Select all that apply.) 1. The family comes to visit the patient. 2. The patient expresses concern about pain control. 3. The patient's vital signs change, showing a drop in blood pressure. 4. The charge nurse approaches the nurse and requests a report at end of shift.

2. The patient expresses concern about pain control. 3. The patient's vital signs change, showing a drop in blood pressure.

A patient has been in the hospital for 2 days because of newly diagnosed diabetes. His medical condition is unstable, and the medical staff is having difficulty controlling his blood sugar. The physician expects that the patient will remain hospitalized at least 3 more days. The nurse identifies one nursing diagnosis as deficient knowledge regarding insulin administration related to inexperience with disease management. Which of the following patient care goals are long term? 1. Patient will explain relationship of insulin to blood glucose control. 2. Patient will selfadminister insulin. 3. Patient will achieve glucose control. 4. Patient will describe steps for preparing insulin in a syringe.

3. Patient will achieve glucose control.

Which of the following is true regarding the state of the science in regards to nursing diagnosis? 1) The original taxonomy has proven to be adequate in scope. 2) The organizing framework of the taxonomy is based on the work of Florence Nightingale. 3) More research is needed to validate and refine the diagnostic labels. 4) New diagnostic labels are approved by means of a vote of registered nurses.

3) More research is needed to validate and refine the diagnostic labels. Rationale: Diagnostic labels are continuously reviewed and revised as indicated by research—much more of which is needed. The original taxonomy has been replaced by Taxonomy II and is no longer based on a nurse theorist (options 1 and 2). New diagnoses are approved by NANDA International's Diagnostic Review Committee, not by a vote of nurses (option 4).

When initiating the implementation phase of the nursing process, the nurse performs which of the following phases first? 1) Carrying out nursing interventions. 2) Determining the need for assistance. 3) Reassessing the client. 4) Documenting intervention

3) Reassessing the client. Rationale: The first step of implementing is reassessing the client to determine that the activity is still indicated and safe. The next action would be to determine if assistance is required, and then implement the intervention (delegating if appropriate), and last document the intervention.

The nurse asks a patient, "Describe for me your typical diet over a 24-hour day. What foods do you prefer? Have you noticed a change in your weight recently?" This series of questions would likely occur during which phase of a patient centered interview? 1. Setting the stage 2. Gathering information about the patient's chief concerns 3. Collecting the assessment 4. Termination

3. Collecting the assessment

A nurse is talking with a patient who is visiting a neighborhood health clinic. The patient came to the clinic for repeated symptoms of a sinus infection. During their discussion the nurse checks the patient's medical record and realizes that he is due for a tetanus shot. Administering the shot is an example of what type of preventive intervention? 1. Tertiary 2. Direct care 3. Primary 4. Secondary

3. Primary An immunization is an example of a primary prevention aimed at health promotion.

The nurse enters a patient's room, and the patient asks if he can get out of bed and transfer to a chair. The nurse takes precautions to use safe patient handling techniques and transfers the patient. This is an example of which physical care technique? 1. Meeting the patient's expressed wishes 2. Indirect care measure 3. Protecting a patient from injury 4. Staying organized when implementing a procedure

3. Protecting a patient from injury A common method for administering physical care techniques appropriately includes protecting you and your patients from injury, which involves safe patient handling. Transferring a patient is a direct care measure. Organization is an aspect of physical care but not an example of this nurse's action. Although meeting patient needs is important, it is not a physical care technique.

Which statement, if made by the client or family member, would indicate the need for further teaching? 1. If a skin area gets red but then the red goes away after turning, I should report it to the nurse. 2. Putting foam pads under the heels or other bony areas can help decrease pressure. 3. If a person cannot turn himself in bed, someone should help them change position q4h. 4. The skin should be washed with only warm water (not hot) and lotion put on while it is still a little wet.

3. If a person cannot turn himself in bed, someone should help them change position q4h; Immobile and dependent persons should be repositioned at least every 2 hours, not every 4, so this client or family member requires additional teaching. Warm water and moisturizing damp skin are correct techniques for skin care. Red areas that do not return to normal skin color should be reported. It would also be correct to use a foam pad to help relieve pressure.

A patient tells the nurse during a visit to the clinic that he has been sick to his stomach for 3 days and he vomited twice yesterday. Which of the following responses by the nurse is an example of probing? 1. So you've had an upset stomach and began vomiting—correct? 2. Have you taken anything for your stomach? 3. Is anything else bothering you? 4. Have you taken any medication for your vomiting?

3. Is anything else bothering you?

A client has a pressure ulcer with a shallow, partial skin thickness, eroded area but no necrotic areas. The nurse would treat the area with which dressing? 1. Alginate 2. Dry Gauze 3. Hydrocolloid 4. No dressing indicated.

3. Hydrocolloid; Hydrocolloid dressings protect shallow ulcers and maintain an appropriate healing environment. Alginates (option 1) are used for wounds with significant drainage; dry gauze (option 2) will stick to granulation tissue, causing more damage. A dressing is needed to protect the wound and enhance healing.

In the following examples, which nurses are making nursing diagnostic errors? (Select all that apply.) 1. The nurse who listens to lung sounds after a patient reports "difficulty breathing" 2. The nurse who considers conflicting cues in deciding which diagnostic label to choose 3. The nurse assessing the edema in a patient's lower leg who is unsure how to assess the severity of edema 4. The nurse who identifies a diagnosis on the basis of a single defining characteristic

3. The nurse assessing the edema in a patient's lower leg who is unsure how to assess the severity of edema 4. The nurse who identifies a diagnosis on the basis of a single defining characteristic

What is the best indication of proper placement of a nasogastric tube in the stomach? 1.Client is unable to speak 2.Client gags during insertion 3.pH of the aspiration is less than 5 4.Fluid is easily instilled into the tube

3.pH of the aspiration is less than 5

Which of the following meals would the nurse recommend to the client as highest in calcium, iron, and fiber? 1. 3 ounces cottage cheese with 1/3 cup raisins and 1 Banana 2. ½ cup broccoli with 3 ounces chicken and ½ cup peanuts 3. ½ cup spaghetti with 2 ounces ground beef and ½ cup lima beans plus ½ cup ice cream 4. 3 ounces tuna plus 1 ounce cheese sandwich on whole-wheat bread plus a pear

4. 3 ounces tuna plus 1 ounce cheese sandwich on whole-wheat bread plus a pear

Black wounds are treated with debridement. Which type of debridement is most selective and least damaging? 1. Debridement with scissors 2. Debridement with wet to dry dressings 3. Mechanical debridement 4. Chemical debridement

4. Chemical debridement; Chemical debridement is either done with enzyme agents or autolytic agents. Answer 1 is a type of sharp debridement. Answers 2 and 3 are mechanical and less precise than chemical.

When a nurse properly positions a patient and administers an enema solution at the correct rate for the patient's tolerance, this is an example of what type of implementation skill? 1. Interpersonal 2. Cognitive 3. Collaborative 4. Psychomotor

4. Psychomotor Psychomotor skills require the integration of cognitive and motor activities to ensure safe intervention

A client's wound is draining thick yellow material. The nurse correctly describes the drainage as: 1. Sanguineous 2. Serous-sanguineous 3. Serous 4. Purulent

4. Purulent; Drainage is described as purulent. Sanguineous and Serous-sanguineous contain blood. Serous is clear and watery.

You are at the scene of an accident and find the victim has a bleeding lower leg wound. After flushing the wound with water and covering it with a clean dressing, you find the dressing has been saturated with blood. Which of the following would be the best action to take in this case? 1. Lower the extremity while applying pressure to the wound. 2. Take off the first dressing and apply another clean or sterile dressing. 3. Encircle the client's ankle with your hands and apply pressure. 4. Reinforce the first layer of dressing with a second layer of dressing.

4. Reinforce the first layer of dressing with a second layer of dressing; To control severe bleeding, apply direct pressure to the wound and elevate the extremity. If the dressing becomes saturated, apply a second layer. Removing the first dressing may disturb blood clots and increase the bleeding.

A goal specifies the expected behavior or response that indicates: 1. The specific nursing action was completed. 2. The validation of the nurse's physical assessment. 3. The nurse has made the correct nursing diagnoses. 4. Resolution of a nursing diagnosis or maintenance of a healthy state.

4. Resolution of a nursing diagnosis or maintenance of a healthy state. The success in meeting a goal is reflected in achieving expected outcomes—the physiological responses or behaviors that indicate that a nursing diagnosis has been resolved and the patient's health is improving

The nurse follows a series of steps to objectively evaluate the degree of success in achieving outcomes of care. Place the steps in the correct order. 1. The nurse judges the extent to which the condition of the skin matches the outcome criteria. 2. The nurse tries to determine why the outcome criteria and actual condition of skin do not agree. 3. The nurse inspects the condition of the skin. 4. The nurse reviews the outcome criteria to identify the desired skin condition. 5. The nurse compares the degree of agreement between desired and actual condition of the skin.

4. The nurse reviews the outcome criteria to identify the desired skin condition. 5. The nurse compares the degree of agreement between desired and actual condition of the skin. The evaluation of interventions examines two factors: the appropriateness of the interventions selected (whether the IV dressing was changed as the standard of care requires) and the correct application of the intervention (whether the dressing was in place and secure). Checking the IV infusion location in the left arm is an evaluation measure, and checking the type of IV solution is an assessment step to ensure that correct fluid is infusing

An Adult reports usually eating the following each day 3 cups dairy, 2 cups fruit, 2 cups vegetables, 5 ounces grains, and 5 ounces meat. The nurse would counsel the client to: 1. Maintain the diet; the servings are adequate 2. Increase the number of servings of dairy 3.Decrease the number of servings of vegetables 4.increase the number of servings of grains

4.increase the number of servings of grains

The nurse is planning a staff education conference about abdominal assessment. Which point is important for the nurse to include? A. The aorta can be felt using deep palpation in the upper abdomen near the midline. B. The patient should be sitting to best determine the contour and shape of the abdomen. C. Always wear gloves when palpating the skin on the patient's abdomen. D. Avoid palpating the abdomen if the patient reports any discomfort or feelings of fullness.

A

Which action is initially taken by the nurse to verify correct position of a newly placed small-bore feeding tube? A. Placing an order for x-ray film examination to check position B. Confirming the distal mark on the feeding tube after taping C. Testing the pH of the gastric contents and observing the color D. Auscultating over the gastric area as air is injected into the tube

A

The nursing assessment of an 80-year-old patient who demonstrates some confusion but no anxiety reveals that the patient is a fall risk because she continues to get out of bed without help despite frequent reminders. The initial nursing intervention to prevent falls for this patient is to: A. Place a bed alarm device on the bed. B. Place the patient in a belt restraint. C. Provide one-on-one observation of the patient. D. Apply wrist restraints.

A. Place a bed alarm device on the bed.

At 3 am the emergency department nurse hears that a tornado hit the east side of town. What action does the nurse take first? A. Prepare for an influx of patients B. Contract the American Red Cross C. Determine how to restore essential services D. Evacuate patients per the disaster plan

A. Prepare for an influx of patients

The nurse takes a medication to a patient, and the patient tells him or her to take it away because she is not going to take it. What is the nurse's next action? A. Ask the patient's reason for refusal B. Explain that she must take the medication C. Take the medication away and chart the patient's refusal D. Tell the patient that her physician knows what is best for her

A. Rationale: When patients refuse a medication, first ask why they are refusing it.

Hand-off communications that occur between the postanesthesia care unit (PACU) nurse and the nurse on the postoperative nursing unit should be done when a patient returns to the nursing unit. Select appropriate components of a safe and effective hand-off. (Select all that apply.) A. Vital signs, the type of anesthesia provided, blood loss, and level of consciousness B. Uninterrupted time to review the recent pertinent events and ask questions C. Verification of the patient using one identifier and the type of surgery performed D. Review of pertinent events occurring in the operating room E. (OR) while at the nurses' station

A. Vital signs, the type of anesthesia provided, blood loss, and level of consciousness B. Uninterrupted time to review the recent pertinent events and ask questions

13. Which factors should the nurse assess to determine a patient's ability to learn? a. Developmental capabilities and physical capabilities b. Sociocultural background and motivation c. Psychosocial adaptation to illness and active participation d. Stage of grieving and overall physical health

ANS: A Developmental and physical capabilities reflect one's ability to learn. Sociocultural background and motivation are factors in readiness to learn. Psychosocial adaptation to illness and active participation are factors in readiness to learn. Readiness to learn is related to the stage of grieving. Overall physical health does reflect ability to learn; however, because it is paired here with stage of grieving (which is a readiness to learn factor), this is a wrong answer.

A nurse is preparing to teach a kinesthetic learner about exercise. Which technique should the nurse use? a. Let the patient touch and use the exercise equipment. b. Provide the patient with pictures of the exercise equipment. c. Let the patient listen to a podcast about the exercise equipment. d. Provide the patient with a case study about the exercise equipment.

ANS: A Kinesthetic learners learn best while they are moving and participating in hands-on activities. Demonstrations and role playing work well with these learners. Patients who are visual learners learn best when you use pictures and diagrams to explain information. Patients who prefer auditory learning are distracted by pictures and prefer listening to information (e.g., podcasts). Patients who learn best by reasoning logically and intuitively learn better if presented with a case study that requires careful analysis and discussion with others to arrive at conclusions.

8. While preparing a teaching plan, the nurse described what the learner will be able to accomplish after the teaching session. Which action did the nurse complete? a. Developed learning objectives b. Provided positive reinforcement c. Implemented interpersonal communication d. Presented facts and knowledge

ANS: A Learning objectives describe what the learner will be able to do after successful instruction. Positive reinforcement follows feedback and involves the use of praise and acknowledgment of new attitudes, behaviors, or knowledge. Interpersonal communication is necessary for the teaching/learning process, but describing what the learner will be able to do after successful instruction constitutes learning objectives. Facts and knowledge will be presented in the teaching session.

20. Which learning objective/outcome has the highest priority for a patient with life-threatening, severe food allergies that require an EpiPen (epinephrine)? a. The patient will administer epinephrine. b. The patient will identify the main ingredients in several foods. c. The patient will list the side effects of epinephrine. d. The patient will learn about food labels.

ANS: A Once you assist in meeting patient needs related to basic survival (how to give epinephrine), you can discuss other topics, such as nutritional needs and side effects of medications. For example, a patient recently diagnosed with coronary artery disease has deficient knowledge related to the illness and its implications. The patient benefits most by first learning about the correct way to take nitroglycerin and how long to wait before calling for help when chest pain occurs. Thus, in this situation, the patient benefits most by first learning about the correct way to take epinephrine. "The patient will learn about food labels" is not objective and measurable and is not correctly written.

11. When the nurse describes a patient's perceived ability to successfully complete a task, which term should the nurse use? a. Self-efficacy b. Motivation c. Attentional set d. Active participation

ANS: A Self-efficacy, a concept included in social learning theory, refers to a person's perceived ability to successfully complete a task. Motivation is a force that acts on or within a person (e.g., an idea, an emotion, a physical need) to cause the person to behave in a particular way. An attentional set is the mental state that allows the learner to focus on and comprehend a learning activity. Learning occurs when the patient is actively involved in the educational session.

A patient with heart failure is learning to reduce salt in the diet. When would be the best time for the nurse to address this topic? a. At bedtime, when the patient is relaxed b. At lunchtime while the nurse is preparing the food tray c. At bath time, when the nurse is cleaning the patient d. At medication time, when the nurse is administering patient medication

ANS: B Appropriate times to talk about food/diet changes during routine nursing care are at breakfast, lunch, and dinner times or when the patient is completing the menu. Many nurses find that they are able to teach more effectively while delivering nursing care. For example, while hanging blood, you explain to the patient why the blood is necessary and the symptoms of a transfusion reaction that need to be reported immediately. In this situation, because the teaching is about food, coordinating it with routine nursing care that involves food can be effective. At bedtime would be a good time to discuss routines that enhance sleep. At bath time would be a good time to describe skin care and how to prevent pressure ulcers. At medication time would be a good time to explain the purposes and side effects of the medication.

9. A student nurse learns that a normal adult heartbeat is 60 to 100 beats/minute. In which domain did learning take place? a. Kinesthetic b. Cognitive c. Affective d. Psychomotor

ANS: B Cognitive learning includes all intellectual behaviors and requires thinking. In the hierarchy of cognitive behaviors, the simplest behavior is acquiring knowledge. The student nurse acquired knowledge, which is cognitive. Kinesthetic is a type of learner who learns best with a hands-on approach. Affective learning deals with expression of feelings and acceptance of attitudes, opinions, or values. Psychomotor learning involves acquiring skills that require integration of mental and muscular activities, such as the ability to walk or to use an eating utensil.

A patient has been taught how to change a colostomy bag but is having trouble measuring and manipulating the equipment and has many questions. What is the nurse's next action? a. Refer to a mental health specialist. b. Refer to an ostomy specialist. c. Refer to a dietitian. d. Refer to a wound care specialist.

ANS: B Resources that specialize in a particular health need (e.g., wound care or ostomy specialists) are integral to successful patient education. A mental health specialist is helpful for emotional issues rather than for physical problems. A dietitian is a resource for nutritional needs. A wound care specialist provides complex wound care.

1. A nurse is asked about the goal of patient education. What is the nurse's best response? The goal of educating others is to help people a. Meet standards of the Nurse Practice Act. b. Achieve optimal levels of health. c. Become dependent on the health care team. d. Provide self-care only in the hospital.

ANS: B The goal of educating others about their health is to help individuals, families, or communities achieve optimal levels of health. Although all state Nurse Practice Acts acknowledge that patient teaching falls within the scope of nursing practice, this is the nurse's standard, not the goal of education. Patient education helps patients make informed decisions about their care and become healthier and more independent, not dependent. Nurses provide patients with information needed for self-care to ensure continuity of care from the hospital to the home.

14. A nurse is preparing to teach a patient about heart failure. Which environment is best for patient learning? a. A darkened, quiet room b. A well-lit, ventilated room c. A private room at 85 F temperature d. A group room for 10 to 12 patients with heart failure

ANS: B The ideal environment for learning is a room that is well lit and has good ventilation, appropriate furniture, and a comfortable temperature. Although quiet is appropriate, a darkened room interferes with the patient's ability to watch your actions, especially when you are demonstrating a skill or using visual aids such as posters or pamphlets. A room that is cold, hot, or stuffy makes the patient too uncomfortable to focus on the information being presented. Learning in a group of six or fewer is more effective than in larger groups and avoids outburst behaviors.

4. Which statement indicates that the nurse has a good understanding of teaching/learning? a. "Teaching and learning can be separated." b. "Learning is an interactive process that promotes teaching." c. "Learning consists of a conscious, deliberate set of actions designed to help the teacher." d. "Teaching is most effective when it responds to the learner's needs."

ANS: D Teaching is most effective when it responds to the learner's needs. It is impossible to separate teaching from learning. Teaching is an interactive process that promotes learning. Teaching consists of a conscious, deliberate set of actions that help individuals gain new knowledge, change attitudes, adopt new behaviors, or perform new skills.

A nurse who is teaching a group of adults ages 20 to 40 years old about safety is going to ensure that which topic is a priority? Automobile crashes Drowning and firearms Falls Suicide and homicide

Automobile crashes Rationale: When educating a group of young to middle-aged adults on safety, it is important to instruct them on the leading cause of injuries in this group. The leading cause of injuries in this group is related to automobile use. Option 2 is the leading cause for school-age children. Option 3 is the leading cause for older adults, and option 4 relates to adolescents. Cognitive Level: Understanding. Client Need: Safe, Effective Care Environment. Nursing Process: Implementation.

The nurse is teaching a young female patient to practice good skin health. Which information is important for the nurse to include? A. Avoid sunbathing between 3 PM and 7 PM. B. Oral contraceptives and anti-inflammatories make the skin more sensitive to the sun. C. Call the health care provider for the presence of a mole on an arm or leg that appears uniformly brown. D. Wear sunscreen with an SPF of 30 or greater if using a sunlamp or tanning parlor.

B

The nurse sees the nursing assistive personnel (NAP) perform the following for a patient receiving continuous enteral feedings. What intervention does the nurse need to address immediately with the NAP? The NAP: A. Fastens the tube to the gown with tape. B. Places the patient supine while giving a bath. C. Performs oral care for the patient. D. Elevates the head of the bed 45 degrees.

B

The nurse suspects that the patient receiving parenteral nutrition (PN) through a central venous catheter (CVC) has an air embolus. What action does the nurse need to take first? A. Raise head of bed to 90 degrees B. Turn patient to left lateral decubitus position C. Notify health care provider immediately D. Have patient perform the Valsalva maneuver

B

You have been given the following postoperative patients to care for on your shift. Based on the information provided, which patient should you see first? A. A 75-year-old following hip replacement surgery who is complaining of moderate pain in the surgical site, with a heart rate of 92 B. A 57-year-old following hip replacement 6 hours earlier who is receiving intravenous patient-controlled analgesia (PCA) with a history of OSA. The pulse oximeter has been alarming and reading 85% C. A 36-year-old following bladder neck suspension who is 30 minutes late to receive her postoperative dose of antibiotic D. A 48-year-old following total knee replacement who needs help repositioning in bed

B A 57-year-old following hip replacement 6 hours earlier who is receiving intravenous patient-controlled analgesia (PCA) with a history of OSA. The pulse oximeter has been alarming and reading 85%

Nurses are legally required to document medications that are administered to patients. The nurse is mandated to document which of the following? A. Medication before administering it B. Medication after administering it C. Rationale for administering it D. Prescriber rationale for prescribing it

B. Medication after administering it

A nurse is working in the preoperative holding area and is assigned to care for a patient who is having a prosthetic aortic valve placed. The nurse inserts an intravenous (IV) line and obtains vital signs. The patient has a temperature of 39°C (102°F), heart rate of 120, blood pressure (BP) of 84/50, and an elevated white blood cell (WBC) count. The nurse immediately notifies the surgeon of the patient's vital signs because: A. They need to get the patient into the operating room (OR) quickly to start the surgery because of the low blood pressure. B. The surgery may need to be delayed to check the patient's WBC count and investigate the source of fever before surgery. C. The nurse anticipates the need for a fluid bolus to increase the patient's BP. D. The nurse anticipates an order for a sedative to help calm the patient and decrease the heart rate.

B. The surgery may need to be delayed to check the patient's WBC count and investigate the source of fever before surgery.

How should the patient be positioned to best palpate for lumps or tumors during an examination of the right breast? A. Supine with both arms overhead with palms upward B. Sitting with hands clasped just above the umbilicus C. Supine with the right arm abducted and hand under the head and neck D. Lying on the right side, adducting the right arm on the side of the body

C

The nurse assesses the skin turgor of the patient by: A. Inspecting the buccal mucosa with a penlight B. Palpating the skin with the dorsum of the hand C. Grasping a fold of skin on the back of the forearm and releasing D. Pressing the skin for 5 seconds, releasing, and noting each centimeter of depth

C

The nurse is teaching a young mother to palpate her 8-year-old child to quickly evaluate if the child has a fever. Which information is important for the nurse to include? A. Place the palm of the hand on the child's back. B. Lightly touch the child's forehead with the fingertips. C. Place the back of your hand against the child's forehead and then on the back of the neck. D. Use the pads of your fingers and press against the child's neck and over the thorax.

C

The patient is assessed for range of joint movement. He or she is unable to move the right arm above the shoulder. How should the nurse document this finding? A. Patient was not able to flex arm at shoulder. B. Extension of right arm is limited. C. Patient's abduction of right arm was limited to 100 degrees. D. Internal rotation of right arm is limited to less than 90 degrees.

C

The patient reports having a sore throat, coughing, and sneezing. While performing a focused assessment, which finding supports the patient's reported symptoms related to upper respiratory infection? A. Buccal mucosa is moist and dark pink. B. Respiratory rate is 18, rhythm is even. C. Retropharyngeal lymph nodes are enlarged and firm. D. Inspection with a tongue depressor on the posterior tongue causes gagging.

C

Which statement made by an adult patient demonstrates understanding of healthy nutrition teaching? A. I need to stop eating red meat. B. I will increase the servings of fruit juice to four a day. C. I will make sure that I eat a balanced diet and exercise regularly. D. I will not eat so many dark green vegetables and eat more yellow vegetables.

C

A nurse is recovering a patient who received conscious sedation for cosmetic surgery. Which of the following is an advantage that conscious sedation has over general anesthesia? A. Loss of sensation at the surgical site B. Reduction of fear and anxiety and need for assistance with airway patency and ventilation C. Amnesia and relief of pain D. Monitoring in phase I recovery

C Amnesia and relief of pain

A nurse is caring for a client who was just admitted to the unit after falling at a nursing home. This client is oriented to person, place, and time and can follow directions. Which of the following actions by the nurse are appropriate to decrease the risk of a fall? (Select all that apply.) A. Place a belt restraint on the client when he is sitting on the bedside commode. B. Keep the bed in low position with full side rails up. C. Ensure that the client's call light is within reach. D. Provide the client with nonskid footwear. E. Complete a fall-risk assessment.

C. Ensure that the client's call light is within reach. D. Provide the client with nonskid footwear. E. Complete a fall-risk assessment.

A nurse observes smoke coming from under the door of the staff lounge. Which of the following is the priority action by the nurse? A. Extinguish the fire. B. Pull the fire alarm. C. Evacuate the clients. D. Close all open doors on the unit.

C. Evacuate the clients.

The nurse's first action after discovering an electrical fire in a patient's room is to: A. Activate the fire alarm. B. Confine the fire by closing all doors and windows. C. Remove all patients in immediate danger. D. Extinguish the fire by using the nearest fire extinguisher.

C. Remove all patients in immediate danger.

Put the following steps for removal of protective barriers after leaving an isolation room in order: A) Untie top, then bottom mask strings and remove from face. B) Untie waist and neck strings of gown. Allow gown to fall from shoulders and discard. Remove gown, rolling it onto itself without touching the contaminated side. C) Remove gloves. D) Remove eyewear or goggles. E) Perform hand hygiene.

C. Remove gloves. D. Remove eyewear or goggles. B. Untie waist and neck strings of gown. Allow gown to fall from shoulders and discard. Remove gown, rolling it onto itself without touching the contaminated side. A. Untie top, then bottom mask strings and remove from face. E. Perform hand hygiene.

Which client meets the criteria for selection of the apical site for assessment of the pulse rather than a radial pulse? A client who is in shock. The pulse changes with body position changes. A client with an arrhythmia. It is less than 24 hours since a client's surgical operation.

Correct Answer: A client with an arrhythmia. Rationale: The apical rate would confirm the rate and determine the actual cardiac rhythm for a client with an abnormal rhythm; a radial pulse would only reveal the heart rate and suggest an arrhythmia. For clients in shock, use the carotid or femoral pulse (option 1). The radial pulse is adequate for determining change in orthostatic heart rate (option 2). The radial pulse is appropriate for routine postoperative vital sign checks for clients with regular pulses (option 4). Cognitive Level: Understanding. Client Need: Health Promotion and Maintenance. Nursing Process: Planning.

The component that should receive the highest priority before a physical examination is: A. Preparation of the equipment B. Preparation of the environment C. Physical preparation of the patient D. Psychological preparation of the patient

D

The nurse is assessing a patient receiving enteral feedings via a small-bore nasogastric tube. Which assessment findings need further intervention? A. Gastric pH of 4.0 during placement check B. Weight gain of 1 pound over the course of a week C. Active bowel sounds in the four abdominal quadrants D. Gastric residual aspirate of 350 mL for the second consecutive time

D

The nurse is assessing a patient who returned 3 hours ago from a cardiac catheterization, during which the large catheter was inserted into the patient's femoral artery in the right groin. Which assessment finding would require immediate follow-up? A. Palpation of a femoral pulse with a heart rate of 76 B. Auscultation of a heart murmur over the left thorax C. Identification of mild bruising at the catheter insertion site D. Palpation of a right dorsalis pedis pulse with strength of +1

D

The nurse plans to assess the patient's abstract reasoning. Which task should the nurse ask the patient to perform? A. "Tell me where you are." B. "What can you tell me about your illness?" C. "Repeat these numbers back to me: 7...5...8." D. "What does this mean: 'A stitch in time saves nine? ' "

D

Because an older adult is at increased risk for respiratory complications after surgery, the nurse should: A. Withhold pain medications and ambulate the patient every 2 hours. B. Monitor fluid and electrolyte status every shift and vital signs with temperature every 4 hours. C. Orient the patient to the surrounding environment frequently and ambulate the patient every 2 hours. D. Encourage the patient to turn, deep breathe, and cough frequently and ensure adequate pain control.

D Encourage the patient to turn, deep breathe, and cough frequently and ensure adequate pain control.

You are a nurse in the postanesthesia care unit (PACU), and you note that your patient has a heart rate of 130 beats/min and a respiratory rate of 32 breaths/min; you also assess jaw muscle rigidity and rigidity of limbs, abdomen, and chest. What do you suspect, and which intervention is indicated? A. Infection: Notify surgeon and anticipate administration of antibiotics. B. Pneumonia: Listen to breath sounds, notify surgeon, and anticipate order for chest radiography. C. Hypertension: Check blood pressure, notify surgeon, and anticipate administration of antihypertensives. D. Malignant hyperthermia: Notify surgeon/anesthesia provider immediately, prepare to administer dantrolene sodium (Dantrium), and monitor vital signs frequently.

D Malignant hyperthermia: Notify surgeon/anesthesia provider immediately, prepare to administer dantrolene sodium (Dantrium), and monitor vital signs frequently.

A charge nurse is designating room assignments for clients who will be admitted to the unit. Based on the nurse's knowledge of fall prevention, which of the following clients should be assigned to the room closest to the nurses' station? A. A 43-year-old client who is postoperative following a laparoscopic cholecystectomy B. A 61-year-old client being admitted for telemetry to rule out a myocardial infarction C. A 50-year-old client who is postoperative following an open reduction internal fixation of the ankle D. A 79-year-old client who is postoperative following a below-the-knee amputation

D. A 79-year-old client who is postoperative following a below-the-knee amputation

A couple is with their adolescent daughter for a school physical and state they are worried about all the safety risks affecting this age. What is the greatest risk for injury for an adolescent? A. Home accidents B. Physiological changes of aging C. Poisoning and child abduction D. Automobile accidents, suicide, and substance abuse

D. Automobile accidents, suicide, and substance abuse

You are caring for a patient who has diabetes complicated by kidney disease. You need to make a detailed assessment when administering medications because this patient may experience problems with: A. Absorption. B. Biotransformation. C. Distribution. D. Excretion.

D. Excretion

The nursing assessment on a 78-year-old woman reveals shuffling gait, decreased balance, and instability. On the basis of the patient's data, which one of the following nursing diagnoses indicates an understanding of the assessment findings? A. Activity intolerance B. Impaired bed mobility C. Acute pain D. Risk for falls

D. Risk for falls

A nurse is working in an ambulatory care setting and is ready to discharge a patient who is wheelchair dependent. The patient underwent dilation of an esophageal stricture. Her postanesthesia recovery score for ambulatory patients (PARSAP) score is 16. Her family is ready to go and eager to make the long road trip home. In determining if it is safe for the patient to be discharged at this time, the nurse should decide the following: A. The PARSAP score must be 18 or higher before being discharged. B. The patient's family is capable to care for her, and she understands her discharge instructions; thus the nurse proceeds with discharge. C. Since the patient hasn't been drinking much, the nurse is not concerned that she is unable to void and proceeds with discharge. D. Since the patient was admitted to the surgical center in a wheelchair, she can be discharged with a lower PARSAP score.

D. Since the patient was admitted to the surgical center in a wheelchair, she can be discharged with a lower PARSAP score.

In the postanesthesia care unit (PACU) the nurse notes that the patient is having difficulty breathing and suspects an upper airway obstruction. The nurse would first: A. Suction the pharynx and bronchial tree. B. Give oxygen through a mask at 4 L/min. C. Ask the patient to use an incentive spirometer. D. Position the patient on one side with the face down and the neck slightly extended so the tongue falls forward.

DPosition the patient on one side with the face down and the neck slightly extended so the tongue falls forward.

The nurse, at change-of-shift report, learns that one of the clients in his care has bilateral soft wrist restraints. The client is confused, is trying to get out of bed, and had pulled out the IV line, which was subsequently reinserted. Which action(s) by the nurse is appropriate? Select all that apply. Document the behavior(s) that require continued use of the restraints. Ensure that the restraints are tied to the side rails. Provide range-of-motion exercises when the restraints are removed. Orient the client. Assess the tightness of the restraints.

Document the behavior(s) that require continued use of the restraints. Provide range-of-motion exercises when the restraints are removed. Orient the client. Assess the tightness of the restraints. Rationale: Standards require documentation of the necessity for restraints. The implementation of range-of-motion exercises prevents joint stiffness and pain from disuse. Orienting the client helps the nurse determine the necessity of the restraint. Option 2 is inappropriate because it may cause injury if the side rail is lowered without untying the restraint. Cognitive Level: Applying. Client Need: Safe, Effective Care Environment. Nursing Process: Implementation.

Medication errors can place the client at significant risk. Which practice(s) will help decrease the possibility of errors? Select all that apply. Hire only competent nurses. Improve the nurse's ability to multitask. Establish a reporting system for "near misses." Communicate effectively. Create a culture of trust.

Establish a reporting system for "near misses." Communicate effectively. Create a culture of trust. Rationale: Reviewing near misses could identify flaws in the system or practices that placed the client at risk. Communication among staff and with clients will increase the efficiency and create an atmosphere where nurses are willing to discuss errors openly so that the flaws in the system can be corrected. Options 1 and 2 are inappropriate answers. A competent nurse may make medication errors. Also, evidence is needed to support these conclusions. Cognitive Level: Understanding. Client Need: Safe, Effective Care Environment. Nursing Process: Planning.

The client has a urinary health problem. Which procedure is performed using indirect visualization? Intravenous pyelography (IVP) Kidneys, ureter, bladder (KUB) Retrograde pyelography Cystoscopy

Kidneys, ureter, bladder (KUB) A KUB is an x-ray of the kidneys, ureters, and bladder. This does not require direct visualization. Option 1 is an IVP, an intravenous pyelogram, which requires the injection of a contrast media. Option 3 is a retrograde pyelography, which requires the injection of a contrast media. Option 4 is a cytoscopy, which uses a lighted instrument (cystoscope) inserted through the urethra, resulting in direct visualization.

Which of the following activities can you delegate to Nursing Assistive Personnel (NAP)?

Measuring Oral Intake and Urine Output; Reporting an I.V. Container that is Low in Fluid

A client has a streptococcal throat infection. The White Blood Cell count is elevated. When looking at the differential, the nurse expects which type of white blood cell to be elevated? Open Hint for Question 6 in a new window. Eosinophils Monocytes Lymphocytes Neutrophils

Neutrophils Neutrophil count is elevated when a client has a streptococcal infection. Eosinophil count is elevated in allergic reactions Monocyte count is elevated in chronic inflammatory disorders. Lymphocyte count is elevated in viral infections.

A client is being admitted to the hospital because of a seizure that occurred at his home. The client has no previous history of seizures. In planning the client's nursing care, which of the following measures is most essential at this time of admission? Select all that apply. Place a padded tongue depressor at the head of the bed. Pad the bed with blankets. Inform the client about the importance of wearing a medical identification tag. Teach the client about epilepsy. Test oral suction equipment.

Pad the bed with blankets. Test oral suction equipment. Rationale: Options 2 and 5 are measures needed to keep the client safe in the event of another seizure. Option 1 is incorrect because the current nursing literature states to not put anything in the client's mouth during a seizure. Options 3 and 4 are more relevant after the cause of the seizure is known. Seizures are not all classified as epilepsy. Cognitive Level: Applying. Client Need: Safe, Effective Care Environment. Nursing Process: Planning.

A primary care provider is going to perform a thoracentesis. The nurse's role will include which action? Place the client supine in the Trendelenburg position. Position the client in a seated position with elbows on the overbed table. Instruct the UAP to measure vital signs. Administer an opioid analgesic.

Position the client in a seated position with elbows on the overbed table.

A nurse cares for a client following a liver biopsy. Which nursing care plan reflects proper care? Position in a dorsal recumbent position, with one pillow under the head Bed rest for 24 hours, with a pressure dressing over the biopsy site Position to a right side-lying position, with a pillow under the biopsy site Neurological checks of lower extremities every hour

Position to a right side-lying position, with a pillow under the biopsy site

Which noninvasive procedure provides information about the physiology or function of an organ? Angiography Computerized tomography (CT) Magnetic resonance imaging (MRI) Positron emission tomography (PET)

Positron emission tomography (PET) Rationale: This type of nuclear scan demonstrates the ability of tissues to absorb the chemical to indicate the physiology and function of an organ. Option 1 is an invasive procedure that focuses on blood flow through an organ. Options 2 and 3 provide information about density of tissue to help distinguish between normal and abnormal tissue of an organ.

A client with poor nutrition enters the hospital for treatment of a puncture wound. An appropriate nursing diagnosis would be_______________ .

Risk for Infection Rationale: Because a malnourished client with a wound is less able to resist an infection, Risk for Infection is the most likely nursing diagnosis. Others may include Pain or Imbalanced Nutrition but they are less focused on the immediate health risk. Cognitive Level: Applying. Client Need: Safe, Effective Care Environment. Nursing Process: Diagnosing.

During a patient's routine annual physical, she tells you that she has noted that her heart feels like it's " racing, " usually in the later morning, early afternoon, or just before she goes to bed. Her radial pulse rate is 68 beats/min and regular; her blood pressure is 134/82 mm Hg. What additional information is helpful in evaluating the patient's racing heart? (Select all that apply.) a) Dietary habits b) Medication list c) Exercise regimen d) Age, weight, and height

a) Dietary habits b) Medication list Dietary habits may include caffeine fluids and foods that stimulate heart rate. Medication list may include pharmacological agents that increase or decrease heart rate.

For a patient who has a muscle sprain, localized hemorrhage, or hematoma, which wound care product helps prevent edema formation, control bleeding, and anesthetize the body part? a) Binder b) Ice bag c) Elastic bandage d) Absorptive diaper

b) Ice bag An ice bag helps to constrict excess fluid in tissues, which prevents edema. The blood vessels become constricted, help to control bleeding, and can decrease pain where the ice bag is placed.

A patient is admitted for dehydration caused by pneumonia and shortness of breath. He has a history of heart disease and cardiac dysrhythmias. The nursing assistant tells you his admitting vital signs. Which measurement should you reassess? (Select all that apply.) a) Right arm BP: 120/80 b) Radial pulse rate: 72 and irregular c) Temporal temperature: 37.4°C (99.3°F) d) Respiratory rate: 28 e) Oxygen saturation: 99%

b) Radial pulse rate: 72 and irregular d) Respiratory rate: 28 An irregular pulse may be the result of a complication of heart disease and requires the assessment of the apical rate. A respiratory rate of 28 is abnormal, yet the oxygen saturation is normal. Both oxygen saturation and respiratory rate would be expected to be outside of the acceptable range

A 55-year-old widowed patient was in a motor vehicle accident and is admitted to a surgical unit after repair of a fractured left arm and left leg. She also has a laceration on her forehead. An intravenous (IV) line is infusing in the right antecubital fossa, and pneumatic compression stockings are on the right lower leg. She is receiving oxygen via a simple face mask. What sites do you instruct the nursing assistant to use for obtaining the patient's blood pressure and temperature? a) Right antecubital and tympanic membrane b) Right popliteal and right axillae c) Left antecubital and oral d) Left popliteal and temporal artery

b) Right popliteal and right axillae The only extremity that does not have a compromised artery to auscultate is the right lower leg after the sequential device is removed. The tympanic membrane and temporal artery are affected by facial surgery and oxygen mask.

A postop client who had abdominal surgery is holding a pillow against his abdomen during deep-breathing and coughing exercises. What term does the nurse use to describe this technique?

splinting

When caring for a single client during one shift, it is appropriate for the nurse to reuse only which of the following personal protective equipment? Goggles Gown Surgical mask Clean gloves

Goggles Rationale: Unless overly contaminated by material that has splashed in the nurse's face and cannot be effectively rinsed off, goggles may be worn repeatedly (option 1). Since gowns are at high risk for contamination, they should be used only once and then discarded or washed (option 2). Surgical masks (option 3) and gloves (option 4) are never washed or reused. Cognitive Level: Understanding. Client Need: Safe, Effective Care Environment. Nursing Process: Implementation.

The nurse evaluates the chart of a 65-year-old client and concludes that which immunizations are current? (Select all that apply.) Last tetanus booster was at age 50. Receives a flu shot every year. Has not received the hepatitis B vaccine. Has not received the hepatitis A vaccine. Has not received the herpes zoster vaccine.

Receives a flu shot every year. Has not received the hepatitis B vaccine. Has not received the hepatitis A vaccine. Rationale: Flu shots are recommended for all adults over age 50. Only adults at risk need to receive hepatitis B and A vaccine (note, this is different than for children). Options 1 and 5 are incorrect because all adults should receive a tetanus booster every 10 years (or sooner if injured) and adults over age 60 should receive the herpes zoster vaccination. Cognitive Level: Remembering. Client Need: Safe, Effective Care Environment. Nursing Process: Assessment. Rationale: Flu shots are recommended for all adults over age 50. Only adults at risk need to receive hepatitis B and A vaccine (note, this is different than for children). Options 1 and 5 are incorrect because all adults should receive a tetanus booster every 10 years (or sooner if injured) and adults over age 60 should receive the herpes zoster vaccination. Cognitive Level: Remembering. Client Need: Safe, Effective Care Environment. Nursing Process: Assessment.

A practitioner orders a wound to be packed with a wet-to-damp gauze dressing. What should the nurse explain to the client is the primary reason for this type of dressing?

Packing the wound with wet-to-damp dressings allows epidermal cells to migrate more rapidly across the bed of the wound surface than dry dressings, thereby facilitating healing. Wet-to-damp dressings will also wick exudate up and away from the base of the wound and help to increase resistance to a wound infection.

An 87-year-old man is admitted to the hospital for cellulitis of the left arm. He ambulates with a walker and takes a diuretic medication to control symptoms of fluid retention. Which intervention is most important to protect him from injury? Leave the bathroom light on. Withhold the client's diuretic medication. Provide a bedside commode. Keep the side rails up.

Provide a bedside commode. Rationale: The placement of the bedside commode next to his bed will assist in decreasing the number of steps he is required to ambulate. This will assist in protecting him from injury due to falls. Option 1: Leaving the light on would assist the client in locating the bathroom, but would not reduce the risk of fall when rushing to the bathroom. Option 2: The nurse cannot withhold a client's medication without consulting with the primary care provider. Option 4: If the client has orders to be up with assistance and the side rails are up, he is at risk for falls as well as falling from a greater distance. Cognitive Level: Applying. Client Need: Safe, Effective Care Environment. Nursing Process: Implementation.

Review the following list of nursing diagnoses and identify those stated incorrectly. (Select all that apply.) 1. Acute pain related to lumbar disk repair 2. Sleep deprivation related to difficulty falling asleep 3. Constipation related to inadequate intake of liquids 4. Potential nausea related to nasogastric tube insertion

1. Acute pain related to lumbar disk repair 2. Sleep deprivation related to difficulty falling asleep 4. Potential nausea related to nasogastric tube insertion

A nurse is caring for patients with a variety of wounds. Which would will most likely heal by primary intention? 1. Cut in the skin from a kitchen knife 2. Excoriated perineal area 3. Abrasion of the skin 4. Pressure ulcer

1. Cut in the skin from a kitchen knife; A cut in the skin by a sharp instrument with minimal tissue loss can heal by primary intention when the wound edges are lightly pulled together (approximated). Excoriations, abrasions, and pressure ulcers heal by secondary, not primary. Secondary intention healing occurs when wound edges are not approximated because of full-thickness tissue loss; the wound is left open until it fills with new tissue. Abrasions and excoriations are injuries to the surface of the skin.

The nurse wishes to determine the client's feelings about a recent diagnosis. Which interview question is most likely to elicit this information? 1) "What did the doctor tell you about your diagnosis?" 2) "Are you worried about how the diagnosis will affect you in the future?" 3) "Tell me about your reactions to the diagnosis." 4) "How is your family responding to the diagnosis?"

3) "Tell me about your reactions to the diagnosis." Rationale: Eliciting feelings requires an open-ended question that does more than seek factual information (option 1) and cannot be answered with a single word (option 2). The family can provide indirect information about the client, but is not most likely to provide the most accurate information (option 4).

The nurse is answering questions after a presentation on sleep at a local senior citizens center. A woman in her late 70s asks for an opinion about the advisability of allowing her husband to nap for 15 to 20 minutes each afternoon. Which is the nurse's best response? 1) "Taking an afternoon nap will interfere with his being able to sleep at night. If he's tired in the afternoon, see if you can interest him in some type of stimulating activity to keep him awake." 2) "He shouldn't need to take an afternoon nap if he's getting enough sleep at night." 3) "Unless your husband has trouble falling asleep at night, a brief afternoon nap is fine." 4) "Encourage him to consume coffee or some other caffeinated beverage at lunch to prevent drowsiness in the afternoon."

3) "Unless your husband has trouble falling asleep at night, a brief afternoon nap is fine."

The nurse is to administer 0.75 mL of medication subcutaneously in the upper arm to a 50-year-old 300-pound client. The nurse can grasp approximately 2 inches of the client's tissue at the upper arm. Which is the most appropriate for the nurse to use? 1) A tuberculin syringe, #25-#27 gauge, 1/4- to 5/8-inch needle 2) Two 3-mL syringes, #20-#23 gauge, 1 1/2-inch needle 3) 2-mL syringe, #25 gauge, 5/8-inch needle 4) 2-mL syringe, #20-#23 gauge, 1-inch needle

3) 2-mL syringe, #25 gauge, 5/8-inch needle Rationale: The type of syringe for subcutaneous injections depends on the medication to be given. This situation does not indicate that the medication is insulin and, thus, another syringe is needed. Generally a 2-mL syringe is used for most subcutaneous injections. Generally, a #20- to #23-gauge needle is used for IM injections. Needle size and length are based on the client's body mass, the intended angle of insertion, and the site of the injection. Generally, a #25-gauge, 5/8-inch needle is used for adults of normal weight and the needle is inserted at a 45-degree angle. Because 2 inches of tissue can be grasped or pinched at the site of the injection, the nurse should administer the medication at a 90-degree angle to ensure the medication reaches subcutaneous tissue.

Which test is the best resource for determining the preoperative status of a client's liver function? 1) Serum electrolytes 2) Blood urea nitrogen (BUN), creatinine 3) Alanine amino transferase (ALT), aspirate amino transferase (AST), bilirubin 4) Serum albumin

3) Alanine amino transferase (ALT), aspirate amino transferase (AST), bilirubin

In the validating activity of the assessing phase of the nursing process, the nurse performs which of the following? 1) Collects subjective data. 2) Applies a framework to the collected data. 3) Confirms data is complete and accurate. 4) Records data in the client record.

3) Confirms data is complete and accurate. Rationale: In validating, the nurse confirms that data is complete and accurate. Subjective data is collected in the collecting activity (option 1), a framework is applied to the data in the organizing activity (option 2), and data is recorded in the documenting activity (option 4).

After having received 0.2 mg of naloxone (Narcan) intravenous push (IVP), a patient' s respiratory rate and depth are within normal limits. The nurse now plans to implement the following action: 1) Discontinue all ordered opioids 2) Close the room door to allow the patient to recover 3) Administer the remaining naloxone over 4 minutes 4) Assess patient's vital signs every 15 minutes for 2 hours

4) Assess patient's vital signs every 15 minutes for 2 hours

A patient returning to the nursing unit after knee surgery is verbalizing pain at the surgical site. The nurse's first action is to: 1) Call the patient's health care provider. 2) Administer pain medication as ordered. 3) Check the patient's vital signs. 4) Assess the characteristics of the pain.

4) Assess the characteristics of the pain.

The use of a conceptual or theoretical framework for collecting and organizing assessment data ensures which of the following? 1) Correlation of the data with other members of the health care team. 2) Demonstration of cost-effective care. 3) Utilization of creativity and intuition in creating a plan of care. 4) Collection of all necessary information for a thorough appraisal.

4) Collection of all necessary information for a thorough appraisal. Rationale: Frameworks help the nurse be systematic in data collection. Other members of the health care team may use very different conceptual organizing frameworks so data may not correlate (option 1). Cost-effective care (option 2) is more likely to occur with systematic application of the Nursing Process, but use of a framework for assessment alone may not accomplish this goal. Because the framework is structured and because of the nature of client needs/problems, creativity and intuition in care planning are not assured (option 3).

A client has a history of sleep apnea. Which is the most appropriate question for the nurse to ask? 1) Do you have a history of cardiac irregularities? 2) Do you have a history of any kind of nasal obstruction? 3) Have you had chest pain with or without activity? 4) Do you have difficulty with daytime sleepiness?

4) Do you have difficulty with daytime sleepiness?

The nurse is developing a plan of care for a patient experiencing narcolepsy. Which intervention is appropriate to include on the plan? 1) Instruct the patient to increase carbohydrates in the diet 2) Have patient limit fluid intake 2 hours before bedtime 3) Preserve energy by limiting exercise to morning hours 4) Encourage patient to take one or two 20-minute naps during the day

4) Encourage patient to take one or two 20-minute naps during the day

The client has an order of morphine 2.5 to 5.0 mg intravenous (IV) every 4 hours. He received 2.5 mg IV 4 hours ago for pain rated at 3 on a scale of 0 to 10. He is now watching television and visiting with family members. When asked about his pain, he rates it as a 5. His vital signs are stable. What nursing intervention is the most appropriate? 1) Give morphine 3.5 mg IV and inform him to continue watching TV because it is a distraction from the pain. 2) Give 2.5 mg of morphine IV to avoid the client becoming addicted. 3) Give nothing at this time because he is not exhibiting any signs of pain. 4) Give morphine 5.0 mg IV and reassess in 20 minutes.

4) Give morphine 5.0 mg IV and reassess in 20 minutes.

Which action by the nursing assistant at bedtime requires the nurse to intervene? 1) Giving the patient a back rub 2) Turning on quiet music 3) Dimming the lights in the patient's room 4) Giving a patient a cup of coffee

4) Giving a patient a cup of coffee

3. A nurse provides teaching about coping with long-term impaired functions. Which situation serves as the best example? a. Teaching a family member to give medications through the patient's permanent gastric tube b. Teaching a woman who recently had a hysterectomy about her pathology reports c. Teaching expectant parents about physical and psychological changes in childbearing women d. Teaching a teenager with a broken leg how to use crutches

ANS: A Not all patients fully recover from illness or injury. Many have to learn to cope with permanent health alterations. New knowledge and skills are often necessary for patients and/or family members to continue activities of daily living. Teaching family members to help the patient with health care management (e.g., giving medications through gastric tubes, doing passive range-of-motion exercises) is an example of coping with long-term impaired functions. Injured and ill patients need information and skills to help them regain or maintain their levels of health. Some examples of this include teaching a woman who recently had a hysterectomy about her pathology reports and expected length of recovery and teaching a teenager with a broken leg how to use crutches. In childbearing classes, you teach expectant parents about physical and psychological changes in the woman and about fetal development; this is part of health maintenance.

A patient had a stroke and must use a cane for support. A nurse is preparing to teach the patient about the cane. Which learning objective/outcome is most appropriate? a. The patient will walk to the bathroom and back to bed using a cane. b. The patient will understand the importance of using a cane. c. The patient will learn how to use a cane. d. The patient will know the correct use of a cane.

ANS: A Outcomes often describe a behavior that identifies the patient's ability to do something on completion of teaching such as will empty colostomy bag, or will administer an injection. Understand, learn, and know are not behaviors that can be observed or evaluated.

The client is only comfortable lying on the right side or left side (not on the back or stomach). List at least four potential sites of pressure ulcers the nurse must assess.

These are important areas to assess. Potential ulcer sites for side-lying clients include: 1. Ankles 2. Knees 3. Trochanters 4. Ilia 5. Shoulders 6. Ears


Kaugnay na mga set ng pag-aaral

Lifespan Chap 32: Concepts of Care for Patients with Cardiac Problems

View Set

Combo of NY BAR Review cards by smjr1920

View Set

Chapter 26: EMT: Behavioral Emergencies Post study Questions

View Set

Chapter 29 The High-Risk Newborn: Problems Related to Gestational Age and Development

View Set

Public Speaking DE - Ch 12 Using Language

View Set

CWTS-6-Radio Frequency Fundamentals for Wireless LAN Technology

View Set